Download as pdf or txt
Download as pdf or txt
You are on page 1of 124

ING BAHASA INGGRIS

DAFTAR ISI

Halaman

BAHASA INGGRIS
01. PERSIAPAN UJIAN MASUK PTN (1) ............................................................................................... 1

02. PERSIAPAN UJIAN MASUK PTN (2) ............................................................................................... 8

03. PERSIAPAN UJIAN MASUK PTN (3) ............................................................................................... 15

04. PERSIAPAN UJIAN MASUK PTN (4) ............................................................................................... 21

05. PERSIAPAN UJIAN MASUK PTN (5) ............................................................................................... 27

06. PERSIAPAN UJIAN MASUK PTN (6) ............................................................................................... 32

07. PERSIAPAN UJIAN MASUK PTN (7) ............................................................................................... 38

08. PERSIAPAN UJIAN MASUK PTN (8) ................................................................................................... 44

09. PERSIAPAN UJIAN MASUK PTN (9) ................................................................................................... 49

10. PERSIAPAN UJIAN MASUK PTN (10) ................................................................................................... 55

11. PERSIAPAN UJIAN MASUK PTN (11) ................................................................................................... 61

12. PERSIAPAN UJIAN MASUK PTN (12) ................................................................................................... 67

13. PERSIAPAN UJIAN MASUK PTN (13) ................................................................................................... 72

14. LATIHAN UJIAN MASUK PTN (1) ......................................................................................................... 78

15. LATIHAN UJIAN MASUK PTN (2) ......................................................................................................... 82

16. LATIHAN UJIAN MASUK PTN (3) ......................................................................................................... 88

17. LATIHAN UJIAN MASUK PTN (4) ........................................................................................................ 92

18. LATIHAN UJIAN MASUK PTN (5) ........................................................................................................ 95

19. INTERNATIONAL EXAM (1) ................................................................................................................. 98

20. INTERNATIONAL EXAM (2) ................................................................................................................. 106

21. INTERNATIONAL EXAM (3) ................................................................................................................. 115


BAHASA INGGRIS
PERSIAPAN UJIAN MASUK PTN (1)

Text 1

Vast populations of microbes live between four and six miles above the Earth's surface in an atmospheric
zone considered at best a pretty unpleasant location for life. They might be living at those altitudes and feasting on
carbon compounds that help warm the planet, or perhaps they were launched up there by air currents, according to a
new study.
Researchers found 17 different bacterial taxa. On average, 20 percent of the small particles in the upper
atmosphere are living bacterial cells. Bacteria greatly outnumber fungi in the atmosphere. The bugs in the air seem
to mirror the type of bugs on the surface. When the aircraft flew over the ocean; the filters caught marine bacteria,
and when they were over land, they found terrestrial microbes. The bacteria likely reach such great heights through
the same processes that send sea salt and dust into the air.
Some of the bacteria use carbon compounds in the atmosphere, suggesting they might be able to survive
there long-term. What is especially interesting about this is the potential impact microscopic creatures may have on
our weather. Clouds are collections of liquid or frozen droplets that condense around a nucleus, usually a piece of
dust or a grain of salt. But nuclei could be made from bacteria, too. Some types of bacteria promote the formation of
ice droplets or of freezing.

01. The sentence “The bacteria likely reach such 03. It is assumed that bacteria in the atmosphere …
great heights through the same processes that
send sea salt and dust into the air” (paragraph 2) (A) determine the pollution level
is most probably meant to …
(B) lead to the formation of clouds
(A) explain how light materials like bacteria and (C) signal the bad quality of the air
dust can easily reach the sky
(D) can be found in all places
(B) conclude the journey made by bacteria that
enables them to live in the space (E) affect the weather of an area
(C) emphasize the condition that bacteria and
dust can stay in the atmosphere
(D) show a similar method by bacteria and other
04. The points mentioned in paragraph 3 of the
materials to stay in the air
passage mainly explain that …
(E) speculate the path taken by bacteria and
other materials to the sky
(A) kinds of bacteria in the air are linked to the
local weather
02. The following pairs of associated words are (B) bacteria have long polluterd the air more
relevant with the idea on how bacteria in the air than we think
are linked to the formation of ice droplets
(C) prevalence of bacteria in the air depends on
described in the passage, except …
area types
(A) smoke – polluting air (D) small particles flying in the air are
(B) heavy rain – flooding microorganisms
(C) roots – deepening
(D) heat – expanding (E) the atmosphere contains a varied form of
(E) low temperature – freezing microbes

-1- BTA / Ing / S. Intensif 2023


Text 2

A.D 830: A storm sends an Indonesian trading ship drastically off course. Months later, dozens of ragged
survivors make landfall on an island off the southeast coast of Africa, more than 3,000 miles from home. Today,
Murray Cox, a computational biologist at New Zealand Masses University, says a scenario like this may describe the
gloomy origins of the first permanent settlement on Madagascar, home to about 22 million people today.
Genetic and linguistic studies suggest the island’s native Malagasy people are mainly of Indonesian
descent. The idea of early Indonesians traveling 3,000 miles to the island intrigued Cox. It’s a surprisingly long
distance to come, he says. So he used computer modeling to parse the clue, running through 40 million settlement
simulations. Cox soon pinpointed one that would explain the DNA pattern evident in Madagascar today.
Surprisingly, the current populations descends primarily from just 30 or so Indonesian women who arrived 12
centuries ago. His conclusion is supported by prior finding that about 30 percent of Malagasy have the same
mitochondrial DNA, which is passed from mother to child-far less diversity than in typical human populations,
which share less than 2 percent. This suggests rapid, recent growth from a very small founded populations, Cox
says.
It is unclear how Madagascars founding mothers (and the fathers who must have been with them) arrived.
Cox proposed seafaring merchants throws off course, or refugees fleeing political strive, the latter could explain why
women, usually not found on trade ships, were on board. Now, Cox plans to explore whether small founding groups
are characteristics of other early island settlement, including Hawaii. There may be general rules for settling islands,
he says.

05. The passage implies that Indonesian settlers in 07. The paragraph following the passage will likely
Madagascar most probably … talk about …

(A) migrated due to political reasons (A) the life of Hawaiian people
(B) were the first settlers on the island
(B) political aspects of the journey
(C) traveled to the island on purpose
(D) landed on the island by accident (C) further similar studies on other islands
(E) found it hard to live on the island
(D) DNA analyses of the island’s population
(E) the life in Madagascar compared to that in
06. Which of the following is the restatement of the Hawaii
sentence “The idea of early Indonesians traveling
3,000 miles to the island intrigued Cox.”
(paragraph 2)?
08. In presenting the ideas, the author starts by …
(A) Cox proposed a theory of nomadic life of
Indonesian people. (A) introducing an expert
(B) Cox investigated the intrigues behind the (B) describing the sea voyage
migration of Indonesians.
(C) Cox was eager to make a long journey like (C) presenting a historical fact
Indonesians to the island. (D) theorizing the origin of Indonesians
(D) Cox was curious so as to why Indonesians
had a long distance journey. (E) providing the result of a biologist’s study
(E) Cox’s ideas of Indonesian’s long journey to
the island are still in question.

-2- BTA / Ing / S. Intensif 2023


Text 3

Passage A
For those of you now eyeing your cell phones suspiciously, it's worth noting that both the National Cancer
Institute (NCI ) and the World Health Organization (WHO) say there isn't evidence to support the assertion that cell
phones are a public-health threat. But a number of scientists are worried that there has been a dangerous rush to
declare cell phones safe, using studies they feel are inadequate and too often weighted toward the wireless industry's
interests. An analysis published by University of Washington neurologist Henry Lai determined that far more
independent studies than industry-funded studies have found at least some type of biological effect from cell-phone
exposure.
A strong link between mobiles and cancer could have major public-health implications. As cell phones
make and take calls, they emit low-level radio-frequency (RF) radiation. Stronger than FM radio signals, these RF
waves are still a billionth the intensity of known carcinogenic radiation like X-rays.
(www.time.com)

Passage B
A study published in the journal Bioelectromagnetics reported no statistically significant change in the
incidence of brain cancers in men and women in England between 1998 and 2007, a time when a cell phone use
increased dramatically.
But now, new work published this week in the Journal of the American Medical Association says there is
an identifiable effect of cell phone use in the brain, but it is really too early to tell what, if anything, that effect
means health-wise.
There has been a lot of controversy of whether cell phones could increase the temperature of the brain,
which in turn could affect energy requirements. Based on our study, we really cannot infer whether this is a bad or
could even have potentially good applications so that – our finding does not illuminate or enlighten that very
important question of whether cell phone exposure could have detrimental effects.

09. The topic discussed in both passages above is … 11. Which of the following reflects opinions
mentioned in both passages?
(A) the increase in the brain cancer incidents by
the mobile phone radiation (A) There was no increase of brain cancer
(B) inconclusive empirical findings on the patients in men and women between 1998-
health impact of the use of mobile phones 2007.
(C) recent research findings on radiation by (B) Statements by NCI and WHO on safe use of
mobile phones on brain damage cell phones might lead to a dangerous rush.
(D) arguments against frequent uses of mobile (C) There are some types of biological effects
phones for health reasons from mobile phone exposure.
(E) issues on the impact of using mobile phones (D) Radiation emitted by X-rays is carcinogenic.
on one’s state of health (E) Mobile phones emit deadly radiation.

12. Information in both passages may lead to a


10. Both passages are similar in content in terms of hypothesis that …
addressing …
(A) uncontrolled use of mobile phones will
(A) relation between mobile phones and health increase carcinogenic stuff in the user’s
brain
(B) recent research evidence on cell phone uses
(B) frequent use of mobile phones will increase
(C) potential effects of mobile phone use on the radiation accumulation in the brain
users’ brain (C) the increasing use of mobile phones will
result in detrimental health effects of their
(D) mobile phone use in triggering the brain
users
cancer
(D) careless use of mobile phones will increase
(E) findings of research on the use of mobile levels of brain energy consumptions
phones (E) regular use of mobile phones will increase
biological defects not just in the brain

-3- BTA / Ing / S. Intensif 2023


Text 4

Parents send their children to school with the best of intentions, believing that formal education is what
kids need to become productive, happy adults. Many parents do have qualms about how well schools are
performing, but the conventional wisdom is that these issues can be resolved with more money, better teachers,
more challenging curricula, or more rigorous tests. But what if the real problem is school itself? The unfortunate fact
is that one of our most cherished instituations is, by its very nature, failing our children and our society.
Children are required to be in school, where their freedom is greatly restricted, far more than most adults
would tolerate in their workplaces. In recent decades, we've been compelling them to spend even more time in this
kind of setting, and there's strong evidence that this is causing psychological damage to many of them. And as
scientists have investigated how children naturally learn, they've realized that kids do so most deeply and fully, and
with greatest enthusiasm, in conditions that are almost opposite to those of school.
Compulsory education has been a fixture for our culture now for several generations. The former President
Obama and Secretary of education Arne Duncan are so enamored of it that they want even longer school days and
years. Most people assume that the basic design of today's schools emerged from scientific evidence about how
children learn. But nothing could be further from the truth.
Schools as we know them today are a product of history, not of research. The blueprint for them was
developed during the Protestant Reformation, when schools were created to teach children to read the bible, to
believe Scripture without questioning it, and to obey authority figures without questioning them. When schools were
taken over by the state, made compulsory, and directed toward secular ends, the basic structure and methods of
teaching remained unchanged. Subsequent attempts at reform have failed because they haven't altered the basic
blueprint. The top-down, teach-and-test method, in which learning is motivated by a system of rewards and
punishments rather than by curiosity or by any real desire to know , is well designed for indoctrination and
obidience training but not much else. It's no wonder that many of the world's greatest entrepreneurs and innovators
either left school early (like Thomas Edison) or said they hated school and learned despite it, not because of it (like
Albert Enstein).

(D) To review how compulsory education the


13. What is the topic of the text above? USA has met parent's expectation.
(E) To describe how American children learn at
(A) Parent's expectation on reformation in
school and in real-life settings.
American school system.
(B) Doubts on the effectiveness of America
15. Which of the following is closest in meaning to
school systems.
the word “qualms” (paragraph 1)?
(C) Restrictions on children's freedom at the US
school. (A) remarks
(D) Regulations for American children to stay (B) requests
longer at schools.
(C) doubts
(E) Absence of a research-based school system
(D) views
in the USA.
(E) beliefs
14. What is the purpose of the text?

(A) To discuss if the American school system is


truly effective to educate children.
(B) To remind American parents that the formal
school is basically a product of culture.
(C) To tell the readers that formal school in the
USA have been constatly developed for a
long time.

-4- BTA / Ing / S. Intensif 2023


Text 5

Ecotourism is defined as 'purposeful travel that creates an understanding of cultural and natural history,
while safe guarding the integrity of the ecosystem and producing economic benefits that encourage conservation'.
The definition recognized that ecotourism is an important educational tool. Real life exposure to a natural situation
in the accompaniment of an experienced guide leads to a greater increase in knowledge than real life exposure
without a guide, or exposure to a knowledge able guide in an artificial setting.
More than 50 years ago, Swiss psycologist Jean Piaget studied the development of human cognitive
capability and its dynamic relationship with the physical world. He viewed cognitive development as an interaction
between physical maturation of the brain and environmentally induced changes in learning. He observed that, as
children grow, they proceed through a series of increasingly abstract thinking styles. Piaget demonstrated that a
primary motivation for learning is resolution of cognitive conflict, which he described as `disequilibrium'.
Borrowing upon principles of cognitive psycology, my colleagues and I have developed an interpreted
mode for presenting information about marine mammals and their ocean environment during whale-watch in Hawaii
and Australia. The model has more recently been extended to include snorkeling excursions to coral reef areas near
Maui, Hawaii.
Its application can be examined in the context of a typical 2.5 h commercial whale-watch trip aboard a
Pacific Whale Foundation passanger vessel to observe humpback whales in Hawaii. Each whale-watch trip is a
different venture, controlled by such variable factors as the number and type of passangers weather conditions, what
the whales choose to do (or not to do, as the case may be), the type of vessel, and the experience of the captain.
Nonetheless, it is possible to view the trip as a structured experince, and to guide participants through an education
sequence that has very clear goals and objectives that can be monitored and evaluated over time.

16. The purpose of the text is to ... 19. What is the topic of the text?

(A) describe the development of an interpretive (A) Definition of ecotourism


model of ecotourism (B) Learning through excursions
(B) explain the influence of Piaget's cognitive (C) Ecotourism as learning resources
theory of learning on ecotourism (D) Influences of environment in learning
(C) show that ecotourism can be interpreted induction
contextually as excursions (E) Learning controlled by environmental
(D) discuss the relevance of a learning theory variable
with ecotourism as education
(E) define ecotourism from theories of learning 20. Paragraph 4 implies that …
and their application
(A) an education is for fun
(B) the guide is the instructor
17. The word “its” (paragraph 4) refers to (C) the captain organizes the trip
(D) learning is through experiencing
(A) interpretive model (E) participants are evaluated at the end of the
(B) extended model trip
(C) marine mammal trip
(D) whale-watch program 21. The text mentions all of the following, EXCEPT
(E) environment-based model …

(A) Additional important benefits of ecotourism


18. Which of the following can best replace the word (B) Impact of ecotourism on environment
“venture” (paragraph 4)? understanding
(C) Excursions because of various independent
(A) effort variables
(B) course (D) Brain and environment interaction
(C) activity influencing cognitive growth
(D) pleasure (E) The interpretive model covering both waters
(E) experience and terrestrial excursion

-5- BTA / Ing / S. Intensif 2023


Text 6

Population growth occurs because there are currently three births for every death. In the past, the crude
birth rates were only slightly higher than crude death rates, but with improvements in medicine and economic
growth, the death rate fell more than birth rates. Much of the world's population growth is occuring in less-
developed countries, which are unable to support such growth. The causal effect between poverty and population
growth can be looked at in two ways. First, population growth causes poverty as the limited resources are depleted
and there are too many people for the available goods, resulting in poverty. The other view is that poverty causes
high population growth. Also, parents believe that having many children will ensure that several of them will
survive to take care of them in their old age. Cultures in which children are a form of security encourage high
population growth, as children reach reproductive age and have large families.
Much of the growth is happening disproportionately in urban areas. Cities are environmentally harmful
because they import many resources for the people that live there, and they export their wastes. They also have an
impact on the local and regional meteorology and are centers for social problems such as crime, homelessness, and
unemployment to name a few. Cities do offer many amenities such as cultural opportunities, jobs, and education.
There are some good environmental aspects to cities, such as the promotion of efficiency in transportation, housing,
utilities; the provision of necessary goods and services; and the accomodation of large numbers of people within a
relatively small space. The biggest problem with cities is that people want the benefits of a city while still living in
the country. This leads to urban sprawl and suburban.

22. The purpose of the text is to ... 25. The writer organizes the ideas in the text by ...

(A) show negative impacts of population growth (A) showing the causes of population growth
(B) descibe factors contributing to poverty in and their effect
cities
(B) defining population growth followed with
(C) analyze causes of economic growth to
several examples
population
(D) compare population growth in the past and (C) arguing against population growth, followed
the present with a case in urban places
(E) discuss population problems in urban areas
(D) describing a historical account of population
of developing countries
growth in the past and the present
(E) comparing effects of population growth in
23. The pronoun "they" (paragraph 2) refers to ... countries of different development

(A) people
(B) areas
(C) resources 26. Which of the following is true according to the
(D) cities text?
(E) families
(A) High population growth results in a problem
where cities cannot provide enough jobs
24. It can be inferred from the text that ... (B) Parents now tend to have few children in
less-developed countries
(A) life quality of cities is worse than that of
suburban (C) Large families are tolerated in less-
(B) poverty in a country is linked to its rapid developed countries due to their predominat
birth rates belief
(C) the less developed a country is the worse its (D) Economic growth gives little contribution to
economic growth is population growth
(D) less developed countries tend to show a
higher rate of population growth (E) In the past there were three births for every
(E) the more children parents have, the better death in less-developed countries
their economy will be

-6- BTA / Ing / S. Intensif 2023


27. In which parts does the author assume people’s
expectation of village life with city convenience?

(A) The causal effect … in two ways.


(paragraph 1)
(B) Cultures in which … large families.
(paragraph 1)
(C) They also have an impact … and education.
(paragraph 2)
(D) There are some good … small space.
(paragraph 2)
(E) The biggest problem with … and suburban.
(paragraph 2)

-7- BTA / Ing / S. Intensif 2023


BAHASA INGGRIS
PERSIAPAN UJIAN MASUK PTN (2)
Text 1

A panic attack is a sudden feeling of terror. Usually it does not last long, but it may feel like forever. The
cause can be something as normal as driving over a bridge or flying in an airplane. It can happen even if the person
has driven over many bridges or flown many times before. The signs of panic disorder include a fast heartbeat,
sweaty hands, difficulty breathing, and a lightheaded feeling.
At the first a person may have no idea what is wrong. The first appearance is between the ages of eighteen
and twenty-four. In some cases, it develops after tragedy. For example, the death of a loved one or some other
difficult situation. In the United States, the National Institute of Mental Health says more than two million people
are affected in any one-year period. The American Psychological Association says that it happens more to women
than men. It can last anywhere from a few months to lifetime.
Some people who suffer a panic attack develop a phobia, a deep fear of ever repeating the activity that
brought on the attack. But experts say panic disorder can be treated. Doctors might suggest anti-anxiety or
antidepressant medicines. Talking to a counsellor could help a person learn to deal with or avoid a panic attack.
There are breathing methods, for example, that might help a person calm down.
Panic disorder is include among what mental health professionals call anxiety disorders. A study published
last week reported a link between anxiety disorders and several physical diseases. It says these include thyroid
disease, lung and stomach problems, arthritis, migraine headaches, and allergic conditions.

01. What is the topic of the passage? 03. Based on paragraph 4, what did the study reveal
about anxiety disorders?
(A) Mental hazards
(B) Physical diseases (A) The need for anti-depressant medicines for
(C) Emotional problems those suffering a panic attack
(D) Panic disorders
(B) The connection between the disorder and
(E) Anxiety disorders
some physical disease
(C) The need for talking to a counsellor to deal
02. The word “lightheaded” in line paragraph 1 with a panic attack
means ...
(D) The possibility of developing a panic attack
to a phobia
(A) sad
(B) dizzy (E) The possible treatment of a panic attack
(C) excited
(D) sorrowful
(E) distressed

Text 2

The application, games, and websites that are promoted as ‘educational’ are not always the best ones for
supporting learning. The fact that they are interactive does not necessarily mean that they are much better than an
old-fashioned workbook with its right and wrong answers. Children may enjoy these products for a while but then
get a bit bored, and we now know that they are not the most appropriate or engaging way to learn.
Treat so-called ‘free’ application with caution. Some will expose your child to advertisement. Others are
designed to wait until your child is engaged in a game or storyline and then demand payment before they can go any
further. This can lead to frustration as young children do not understand why they can not continue. Sometimes it is
better to make a small payment in advance if the application promises no further purchases. However, some
applications are free to download and completely free of advertisement or in-app purchases.

-8- BTA / Ing / S. Intensif 2023


Choosing an app needs the same kind of thought and care you would put into buying anything else for your
child. Do not rely only on the star rating. Instead, read the user reviews and check the privacy policy if you are
worried about the personal information that the app might be collecting. If you want your child to enjoy learning, to
develop curiosity, and think about things creatively, provide them with a range of games and apps. Open-ended
games have become progressively more challenging and encourage children to explore and have fun. They are,
therefore, more likely to establish a love of learning and to lay the foundation for their future development. Physical
activity, reading, and other more ‘traditional’ activities continue to play a very important part in children’s
development. But, most parents do rely on screen devices from time to time to engage their child while they are
busy with something else. This is not a problem in itself just as long as children’s time is made up of balanced range
of activities.

04. The author reminds readers to be more careful 06. The author would apparently agree that ...
with commercialism in free applications in
(A) open-ended games can stimulate children’s
paragraph(s) ...
love for learning
(B) some games and applications are boring
(A) 1
because they are too easy
(B) 2 (C) there is no convincing prediction for the
future of the games and apps
(C) 3
(D) it is better to pay for games in advance so
(D) 1 and 4 that children can learn freely
(E) apps star rating provides users with reviews
(E) 2 and 3
on the good things of games and apps

05. How does the idea in sentence 6 relate to the 07. Regarding busy parents reliance on screen device
other ideas in paragraph 2? to help keep their children entertained, the author
assumes that ...
(A) Sentence 6 elaborates the other ideas in
(A) it is the parents responsibility to make their
paragraph 2
children less engaged with screen devices
(B) Sentence 6 is the result of the other ideas in (B) children whose parents are busy with their
paragraph 2 activities must use screen devices less
(C) parent need to be aware of their children’s
(C) Sentence 6 strongly contradicts the other
need for varied activities
ideas in paragraph 2
(D) such parent's reliance is acceptable as long
(D) Sentence 6 is the implementation of the as the children are given the right proportion
theory discussed in paragraph 2 of the other activities
(E) it is not tolerable as their children need to
(E) Sentence 6 provides another type of free
have more physical activities
applications discussed in paragraph 2

Text 3
Light pollution is the change of natural light levels in the night environment produced by introduction of
artificial light. Because of the continuous growth of the nighttime artificial lighting, this problem is increasingly
debated. In fact, many localities have developed regulations to restrict the wasteful loss of light into the sky and
environment.
The expanding use of light at night is because humans are diurnal animals. They are trying to extend
activities into the usually dark hours. This increasing use is driven by what seems common sense, and by the lighting
industry with justifications that at first may seem correct. With few exceptions, everything we build is lit at night.
This includes street, roads, bridges, airports, commercial and industrial building, parking lots, sport centers and
homes. Outdoor lighting continues to expand as more infrastructure are built. Lighting levels in public areas are
often set high with one more secondary objectives in mind. For instance, building exteriors are often lit for a merely
beautiful effect. Shopping centers are typically lit to attract shoppers. They also create a lively environment to
stimulate spending. Lighting levels in public areas often set high to prevent crime, even though studies have not
proven this to have any effect on crime rates. Indeed, the cores of our urban centers are bathed in light. The resulting
light pollution can extend more than a hundred kilometers out from the city’s edge.

-9- BTA / Ing / S. Intensif 2023


Reliable evidence shows that this artificial of the day produce serious and risky consequences to human
health and environment. Ciszano, Falchi and Elvidge (2001) have studied the impact of light pollution on the night
sky. The result indicates that more than 60% of world population live under light polluted skies (99% of the
population of USA and Europe) and almost one-fifth of world terrain is under light polluted skies. In regards to
human, there are no doubts that exposure to light at night (LAN) decreases pineal melatonin (a kind of hormone)
production and secretion and are not only a source for phase shift in daily rhythms.

08. Which of the following best restates the sentence 10. What does the author imply in the second
“.. humans are diurnal animals” in paragraph 2? paragraph?

(A) Humans are day light active creature (A) Providing light could increase economic
(B) Humans require light to live like animals activities
(C) Humans and animals equally need light to (B) Both humans and animals rely on light for
live survival
(D) Both humans and animals are active during (C) Using more light may mean increasing light
the day pollution
(E) Humans and animals try to prolong their (D) Light should be provided according to
activities at night people's needs
(E) Building infrastructure mainly used for
09. The author introduces the problems of light public facilities
pollution followed by ...
11. Which of the following obviously shows the
(A) research evidence indicating the danger of author's bias in the passage?
light pollution on human health
(A) This artificial extension of the day produces
(B) further explanation about why and how
serious and risky consequence to human
people use more light
(B) Many localities have developed regulation
(C) more examples of light and causes that to restrict the wasteful loss of light
might (C) Exposure to light at night decreases pineal
melatonin production and secretion
(D) the use of regulation to minimize the effect
(D) More than 60% or world population lives
of light
under light polluted skies
(E) details on areas requiring light for business (E) Building exteriors are often lit for a merely
beautiful effect

Text 4

The second advantage of herbal medicine is the absence of long term effect. Repeated and long-term use of
herbal remedies does no result in long-term effects on the body. Certain pharmaceutical drugs like paracetamol
when used for long periods results in permanent body damage and long-term ill effect. Herbal remedies on the other
hand are not harmful to the body in the long-term. One can use most herbal remedies repeatedly and regularly
without any worries.
Another benefit of using herbal remedies is that they are completely natural. They do not have chemical
composition or drugs. Due to their natural nature they can be consumed regularly. These natural remedies also result
in a better immune system.
Furthermore, herbal remedies have multipurpose benefits. For instance, ginger is usually used as an herbal
remedy for colds and digestive disorders, but one can also use it in their tea, beverages, and daily home cooked food
for flavoring. Similarly, herbal extracts for peppermint can not only add spice and flavor to your kitchen recipe, but
can also be used as anti-depressant.
Herbal remedies are absolutely inexpensive and sometimes even absolutely free of cost. Unlike
pharmaceutical drugs, herbal remedies have no manufacturing costs, making them far cheaper. Some herbal
remedies like ginseng, peppermint and ginger can even be grown in your backyard.
Herbal remedies are easily accessible. Certain herbal remedies like ginger, peppermint and basil leaves can
be acquired anywhere in the world. One can even visit any local drug store and purchase herbal remedies without
any trouble. Besides this you can even grow some of the remedies in your garden.

- 10 - BTA / Ing / S. Intensif 2023


Another excellent benefit of herbal remedies is that, they can be used to cure people of all ages. Children,
adults and even elderly people can resort to herbal remedies to get rid of their ailment. Due to their natural nature,
herbal remedies most often do not cause an allergic reaction in the body. Several people are allergic to certain
chemical that are part of pharmacheutical drugs. Herbal remedies on the other hand create no allergic reaction.

12. The paragraph preceding the passage most likely 15. What is the best summary of the passage?
discusses ...
(A) Some advantages can be enjoyed when
(A) disadvantages of long term treatment using someone uses herbal medicine. They do not
herbal medicines result in long term effects and allergic
(B) the use of herbal medication for a long term reaction. They are natural and are useful for
treatment many purposes. They can be obtained easily
(C) long term effect of herbal remedies on the at a low price, or even for free. Herbal
body remedies can be used by people of different
(D) herbal medication and its advantages age groups.
(E) one advantage of herbal as medicine (B) One of the advantages of using herbal
medication is its use for several purpose.
Ginger can be used for health treatment as
13. What is the author's attitude toward the topic of well as for meal flavouring. Similarly,
the passage? peppermint gives fresh taste on your food
and serves as an anti-depressant. It is then
(A) Neutral very useful to grow herbs in your garden.
(B) Negative (C) Being useful for some purpose
(C) Concerned simultaneously make herbs great choices.
(D) Approving People can use them, not only for flavouring
(E) Uninterested food, but also for medication. They are
cheap or even free of charge. People can
grow them easily at home.
14. Based on the passage, the more complicated the (D) The best positive side of herbal medicine is
manufacturing process, the ... its natural nature. It is not only useful for
adults, but also for young people. The price
(A) better the drug quality is affordable. Moreover, they can be used
(B) more effective the drugs for several purposes simultaneously.
(C) cheaper the price of drugs (E) Herbal remedies are useful for some
(D) higher the price of medicines reasons. The main reasons is its low price.
(E) less frequent the allergic reaction People can even have it without payment.
Some are useful for multiple purpose.

Text 5

Social norms are the implicit social rules that govern behavior within a community. Norms are not directly
established; instead, they develop over time as people go about their daily behaviors, sense people’s reactions to
those behaviors, and observe what other people are doing. Social norms differ, depending on the group of people,
community, or culture. What is considered fully normal, even admirable, in one group may be met with disapproval
in another (e.g.. offices where casual dress is normal vs. offices where everyone must wear a suit). According to
psychologist Robert Cialdini, people are constantly looking for “social proof” to guide their own behavior. They
look for clues to what other people are doing in order to understand the appropriate behavior in a given situation.
Social proof of what is acceptable is a particularly powerful guide to behavior when the situation is ambiguous or
new.
In relation to social norms, one of the big challenges we face is creating a sustainable culture where so
many unsustainable actions are considered perfectly normal and even something to strive for: driving alone, living in
a very large home, eating foods that have traveled long distances, eating meat at every meal, having a weed-free
green grass lawn, and continuously shopping for new consumer goods. Sustainable behaviors, such as buying
second-hand or taking short shower, on the other hand, are often seen as lower or undesirable in status.

- 11 - BTA / Ing / S. Intensif 2023


Social norms create opportunities for change: for one thing, they are dynamic and constantly shifting (just
think about fashion trends). The goal for those of us who want to bring about a more sustainable society is to quickly
bring sustainable behaviors into the realm of normal, acceptable one, and something people aspire to. How do we
accomplish this? We need to give people evidence, social proof, that sustainable behavior is acceptable and
desirable.

16. The passage is mainly concerned with ... 18. According to the author, how is sustainable
behavior acceptable and desirable?
(A) social proof
(B) social disorder (A) By aspiring people
(C) daily behavior
(B) By making it dynamic
(D) sustainable behavior
(E) acceptable culture (C) By establishing norms
(D) By providing proof to the society
17. Based on the text, the word “disapproval” in
paragraph 1 means ... (E) By giving opportunities for change

(A) a bed feeling


(B) an unacceptable feeling
(C) an unsustainable behavior
(D) a different opinion
(E) a negative opinion

Text 6
Technology tools can help educators make and strengthen home-school connections. With technology as a
means of sharing information and communicating with one another, early childhood educators have an opportunity
to build stronger relationship with parents. Early childhood educators always have had a responsibility to support
parents and families by sharing knowledge about child development and learning. Technology tools such as
smartphones, mobile device, and apps offer affordable ways for busy family members to communicate with their
child's teachers and caregivers. Internet-based communication tools offer new opportunities for video calling and
conferencing when face-to-face meetings are not possible: it can connect children to other family members who live
at the distance. Educators have a responsibility to parents and families to model affective uses of technology, media,
and social media that are safe, responsible, and ethical.
Teachers can use digital portfolios that include photographs as well as audio and video recordings to
archive a child's accomplishment and developmental progression with families in face-to-face conferences, or
through social media tools. Displaying photos in the classroom of children’s drawings or block buildings can help
families understand the critical role of play in early childhood development. Sending weekly, monthly, or even daily
updates through social media or e-mail can help families feel more connected. Inviting children to take a picture of
something they have done and helping them upload the photo to a file that can be e-mailed also contribute to their
learning more about the functions of reading and writing.
Most educators understand the value of writing down by using e-mail, educational texting, or educational
communications tools. This activity demonstrates the concept about communication and helps to build digital
literacy skill at the same time. If information is stored on a computer, the photos and notes can be printed and given
to families. Modelling the effective use of technology and interactive media for parent communication and family
engagement also creates opportunities to help parents themselves become better informed, empowers them to make
responsible choices about technology use and screen time at home, engages them as teachers encourage co-viewing
co-participation, and joint media engagement.

19. How are ideas in paragraphs 1 an 2 related? (C) Paragraph 2 is an introduction to idea in
paragraph 1
(A) Paragraph 2 explains further the idea (D) Paragraph 2 provides examples of
presented in paragraph 1 comparisons presented in paragraph 1
(B) Paragraph 2 contradicts the idea in (E) Paragraph 2 explains the effect of the idea
paragraph 1 stated in paragraph 1

- 12 - BTA / Ing / S. Intensif 2023


20. The author will apparently agree that ... (D) sending weekly or monthly emails to parents
to share the development of a child is quite
(A) busy family members may afford difficult to be done by educators.
smartphones and mobile devices to stay in (E) taking a picture of a child may not
touch with their colleagues. contribute to the success of a learning
(B) educators have a responsibility to process if it is not shared with parents.
demonstrate the effective use of technology
and interactive media parent-teacher 22. Which paragraph(s) ilustrate(s) the effective use
communications. of technology and media to provide parents with
(C) technology tools are necessary to connect information of what is going on in a class?
children to other family members who are
separated thousand miles away. (A) 1
(D) educators have no responsibility to (B) 2
guarantee that technology tools they use are (C) 1 and 2
safe, secure, healthy, and responsible. (D) 3
(E) the effectiveness of technology and media (E) 3 and 4
use in class by educators is only factor
determining the success of parent-teacher
communications.

21. The author holds an assumption that ...

(A) all technology tools are capable of


measuring and recording children’s learning
activities.
(B) all parents have accesses to technology tools
to helps build good communications
between parents and educators.
(C) displaying photos of children’s drawing and
block buildings can help parents understand
why children should play a lot in schools.

Text 7
The green movement is catching on in many pockets of the world. This is especially true in the construction
industry. Today’s buzzwords, which include global warming and zero emission, are causing everyday people to look
for ways to reduce their carbon footprint. Purchasing environmentally-friendly property is a good investment for
those who are concerned about their own health and the well-being of the earth. Based on this trend, entire districts,
known as eco-communities, are being designed with green initiatives in mind. Dockside Green in Victoria, British
Columbia, Canada is one of these communities. Its goal is to become the world’s first zero-emissions neighborhood.
Builders of Dockside Green have the environment in mind with every choice they make. They ensure
proper ventilation, and guarantee residents 100% fresh indoor air. Interior and exterior building materials, such as
paints and wood, are natural and non-toxic. Eco-conscious builders use bamboo wherever possible because it is
durable and does not require pesticides to grow.
Energy efficiency is one of the top priorities in eco-communities, such as Dockside Green. Not only do
energy-efficient appliances and light fixtures reduce the environmental impact of heating and hot water, they also
save residents and business owners money. Dockside Green claims that homeowners will use 55% less energy than
average residents in Canada. Though they are sharing space by investing in condo-style living, residents will have
individual utility metres. Studies show that people use approximately 20% less energy when they are billed for
exactly what they use. In addition, water is treated at Dockside Green and reused on site for flushing toilets.
Planners of eco-communities such as Dockside Green must take the future into account. Dockside Green
will reuse 90% of its construction waste. They also plan to continue using local suppliers for all of their transport
and maintenance needs. This is a great way to reduce emissions. Dockside residents will be encouraged to make use
of a mini transit system and buy into the community’s car share program. Finally, plans are in the works for a high-
tech heating system that will use renewable biomass instead of fossil fuels.

- 13 - BTA / Ing / S. Intensif 2023


23. By saying “Today’s buzz words … to reduce 25. The false idea that the author may hold about
their carbon footprint.” in paragraph 1, the author Dockside Green is that ...
implies that ...
(A) it provides all eco-friendly properties
(A) environmental issues have made people (B) it is a good asset to buy a property in it
more cautious of their actions (C) it encourages all house to utilize efficient
(B) global warming and zero emissions become energy
everyone's concern (D) it reduces global warming and emissions
(C) people have to take care of their (E) it is where more natural livings can exist
greenhouses carefully
(D) eco-friendly environmental programs should
be strongly enforced 26. The author organizes the ideas in the passage by
(E) all people around the world are aware of ...
global warming
(A) describing a new housing trend and
24. Which of the following best restates the sentence providing a detailed example
“The green movement is catching on in many (B) discussing an environmental issue and
pocket of the world.” in line 1? providing ways to solve the issue
(C) identifying a community problem and
(A) Many people consider it crucial to have eco- discussing possible solutions to the problem
friendly environments. (D) explaining an environmentally friendly
(B) People who are aware of environmental initiative and discussing the impacts of the
issues love safe and healthy surroundings initiative
(C) House designers should consider the (E) establishing a new community initiative that
importance of green materials is eco-friendly and explaining the ideas
(D) Everyone should understand the effect of behind the initiative
global warming and zero emissions well
(E) Environmentalists are concerned with the
green environment

- 14 - BTA / Ing / S. Intensif 2023


BAHASA INGGRIS
PERSIAPAN UJIAN MASUK PTN (3)
Text 1

By the age of 5, most triplets are on par with their peers in mental and emotional development, but those born at
the lowest weights may still lag behind, according to a study published in July 2009. Israeli researchers found that
among the 126 singletons, twins and triplets they followed from birth to age 5, triplets generally trailed behind their
peers in cognitive development over the first two years of life. By age 5, however, many triplets had bridged the gaps in
both IQ and social development, the researchers report in the journal Pediatrics. The exception was triplets who’d been
particularly growth-restricted in the womb  those who, at birth, had weighed more than 15 percent less than the sibling
with the highest birth weight. At age 5, these children were still developmentally behind both their siblings and peers.
Until now, there had been no well-designed studies following the development of triplets over the first few years
of life. And the catch-up seen among most triplets in this study is “excellent news,” lead researcher Dr. Ruth Feldman,
of Bar-Ilan University, told Reuters Health. Parents of triplets, she said, should be aware that their children may be
slower to reach developmental milestones in infancy, but most of them are likely to close that gap during the preschool
years. Children who were born substantially smaller than their siblings may not catch up, however.
At age 5, the study found, these children typically scored at the lower end of the normal range for intellectual,
emotional and social development. For instance, Feldman explained, average verbal IQ was about 95, which, while
within normal range, would make it difficult for a child to get through standard schooling.
Growth restriction was common among the 21 sets of triplets in the study. In 65 percent, one sibling was born
weighing more than 15 percent less than the heaviest sibling. The findings on development point to the importance of
giving these children extra attention from infancy onward, according to Feldman. “Knowing that these children respond
to parental investment already in the first months of life tells parents to be especially sensitive and responsive to these
children,” Feldman explained. In addition, she said, the children’s development during infancy and preschool should be
continuously monitored, and parents and children should receive extra help when needed  such as interventions to help
children regulate their emotions and cultivate social skills, or to improve their attention and concentration abilities.
More studies are also needed, the researchers note, to see whether the developmental gaps persist into later childhood
and adolescence.

01. An appropriate title for the text is .... 04. According to the text, which triplet-baby would
most probably be developmentally behind her
(A) Intellectual, Emotional and Social
peers?
Development of Triplets
(B) Triplets, Twins, and Singletons: A
(A) Born with less than 1.5 kg in weight.
Comparison Study
(C) Parental Guides to The Development of (B) Born weighing 20 percent less than her
Babies siblings.
(D) The Reason for the Slow Growth of Triplets
(C) Born with heaviest weight.
(E) The Correlation between Birth-Weight and
Intellectual Development (D) Born weighing 21 percent more than her
siblings.
02. The word “persist” in paragraph 4 can be best
(E) Born with lowest weight.
replaced by ....
(A) vanish
(B) elapse
(C) stay 05. The text would most probably be found in ....
(D) change
(A) an advertisement of a children health center
(E) keep
(B) an article in a medical journal
03. According to the text, when do triplets typically
(C) an article in a popular science magazine
have the biggest gap in development?
(D) an article in a Bar-Ilan University publication
(A) During infancy.
(B) When they reach the age of five. (E) a preface of a medical book
(C) After they reach the age of five.
(D) During school years
(E) During preschool years
- 15 - BTA / Ing / S. Intensif 2023
Text 2

Delivering medicine to the world’s poorest people is a challenge. Hot, poor places such as Tanzania have many
microbes but microscopic health budgets. Dangerous myths deter many sick rural folk from seeking medical help. Even
if they do seek help, it is often unavailable, for they do not have the money to pay for it, and their government rarely has
the money to give it to them for free. Because they cannot afford adequate health care, poor people are sick a lot of the
time. And because they are sick a lot of the time, they find it hard to put in the long hours of productive labor that might
make them less poor.
All hope is not lost, however. A recent experiment in Tanzania has shown that a small health budget can go a
long way, provided that the money is spent with care. With the help of a Canadian charity, the Tanzanian health
ministry set up a health project in two rural districts, with a combined population of about 700,000. Five years ago,
annual health spending in Tanzania was about 8 a head. This figure included an estimate for the annual cost of trained
staff. The charity added 2 a head to the pot, on condition that it was spent rationally. By this, the donors meant that the
amount of money spent on fighting a particular disease should reflect the burden that disease imposed on the local
population.
This may sound obvious; however, in this region, no one had a clue which diseases caused the most trouble, so
the first task was to find out. Researchers were sent out to carry out a door-to-door survey, asking representative
households whether anyone had been ill or died recently, and if so with what symptoms. These raw numbers were then
crunched to produce a ‘burden of disease’ profile for the two districts. In other words, researchers sought to measure
how many years of life were being lost to each disease, including the damage done to families when breadwinners die.
They then compared their results with the amount spent by the local health authorities on each disease and found
that it bore no relation whatsoever to the harm which the disease inflicted on local people. Some diseases were horribly
neglected, such as malaria, which accounted for 30% of the years of life loss but only 5% of the health budget. Other
conditions, meanwhile, attracted more than their fair share of cash. Tuberculosis, which accounted for less than 4% of
years of life loss, received 22% of the budget.
This tiny infusion of cash from the Canadians, in the form of an extra 2 a head, was enough to allow the districts
health authorities to make their spending reflect the disease burden. The results of all this were stunning. Infant
mortality fell by 28% between 1999 and 2000 and the proportion of children dying before their fifth birthday dropped
by 14%.

06. The most suitable title for the text is .... 08. The term a ‘burden of disease’ in paragraph 3
means
(A) A Survey on the Health Condition in Tanzania
(A) a disease is burdensome for the poor
(B) The General Health Condition of Tanzanian
People (B) the worse the disease the more the burden
(C) the relative effects of different diseases on a
(C) The Effects of an Aid Package from a
society
Canadian Charity
(D) a disease affects not only the sick but also the
(D) Deadly and Virulent Diseases Found among breadwinner
the Tanzanian
(E) each society and family has its own burden
(E) The Difference a Small Increase in a Tiny caused by disease
Health Budget can Make
09. Which of the following statements about the text is
07. What can be inferred from paragraph 1? FALSE?

(A) People in African countries do not go to (A) The amount of Canadian budget allocated to
doctors due to perilous myth. each disease depended on how harmful a
disease was.
(B) The budget for health in Tanzania is low as
there aren’t many diseases. (B) The budget allocated for each person included
the training cost for the medical staff.
(C) When people in Tanzania are sick, they have
no hope to recover. (C) The presence of myth in Tanzania may have
discouraged people to go to doctors.
(D) In Tanzania, medical treatment is accessible (D) The additional amount donated by the
for the poor.
Canadian charity was carefully spent.
(E) Poverty and illness in Tanzania are like a (E) A serious disease probably affected not only
vicious cycle. the patient but also the family.
- 16 - BTA / Ing / S. Intensif 2023
10. The purpose of the writer in writing this article is (C) persuade other countries to copy the
probably to .... Tanzanian model
(D) show how the money is spent is more
(A) invite donors to donate money to countries in important than how much is spent
need of cash
(E) explain the types of diseases people can find
(B) show how generous foreign institutions can be in Tanzania
in helping other countries

Text 3

Identifying ingredients in old paintings can help curators decide how best to maintain, display and restore them.
The 17th-century Dutch artist, Rembrandt, was found to use wheat, according to a new advanced analysis of two of his
works. It is the first study to identify wheat starch in any of Rembrandt’s work, even though scientists have performed
numerous analyses on more than 150 of his.
In Rembrandt’s time, artists mixed their own paints, which they then spread onto canvas in layers. Often,
individual layers of the same piece of art contained different binding agents, pigments, varnishes and other ingredients.
Besides color, each layer was mixed to just the right level of thickness, glossiness, texture, evenness on the surface,
drying time and more.
However, paint layers are tough to analyze because they are spread so incredibly thin. The thinnest ones rise just
a thousandth of a millimeter above the layer below them. Using a variety of chemical and physical analytical methods,
along with old written records, scientists have been able to identify pigments and other inorganic materials in many
ancient paintings.
For the new study, Sanyova and colleagues used some of the most high-tech equipment around to look at the
“Portrait of Nicolaes van Bambeeck,” which Rembrandt van Rijn painted in 1641. First the researchers took a cross-
section from a miniscule section of the painting. Then they used a variety of methods to probe the layers, including a
technique called Time of Fly –– Secondary Ion Mass spectrometry (ToF-SIMS). This technique involves sending a
focused, high-energy beam of ions at the layered sample, then observing the ions that bounce back. By analyzing the
energy and chemical nature of the ejected ions, scientists can deduce detailed information about the types of elements
and chemical bonds held within.
For the second grayish layer of paint on the “Portrait of Nicolaes van Bambeeck,” the scan showed, Rembrandt
mixed oil and a small amount of lead with wheat flour. It’s not clear yet whether Rembrandt used wheat earlier or
continued to use the ingredient after painting the “Portrait of Nicolaes van Bambeeck,” who was a rich wool merchant.
But Sanyova and colleagues also found wheat in the “Portrait of Agatha Bas,” the merchant’s wife.

11. What is the main idea of paragraph 3? 13. How does the author organize the ideas in the
passage?
(A) There are some advantages of using paint layers.
(B) There are inorganic materials in ancient (A) By arguing other theories of old painting’s
paintings. ingredients.
(C) Scientists have found the thinnest layer of (B) By comparing one Rembrandt’s works to his
painting. other works.
(D) There are some methods to crack layer-painting (C) By discussing possible ingredients used at
mystery. Rembrandt’s time.
(E) The chemical ingredients for layer painting (D) By presenting the research’s findings and
contain different pigments. describing its process.
(E) By discussing the importance of knowing what
ingredients Rembrandt used.
12. According to paragraph 1, which of the following
words can best describe scientists’ effort to identify 14. What would the paragraph following the passage be
ingredients in Rembrandt’s paintings? likely to discuss?

(A) Glorious. (A) The ToF-SIMS.


(B) Effortless. (B) Paintings of other 17th century artists.
(C) Involuntary. (C) The painting “Portrait of Agatha Bas”.
(D) Lengthy (D) Written evidence of the use of wheat flour.
(E) Laborious. (E) Other techniques used in the research.

- 17 - BTA / Ing / S. Intensif 2023


15. What does the word ‘they’ in “... own paints, which
they then ...” (par. 2) refer to?

(A) Layers.
(B) Artists.
(C) Paints.
(D) Pigments.
(E) Ingredients.

Text 4

Floods are second only to fire as the most common of all natural disasters. They occur almost everywhere in the
world, resulting in widespread damage and even death. Consequently, scientists have long tried to perfect their ability to
predict floods. So far, the best that scientists can do is to recognize the potential for flooding in certain conditions. There
are a number of conditions, from deep snow on the ground to human error, that cause flooding.
When deep snow melts, it creates a large amount of water. Although deep snow alone rarely causes floods, when
it occurs together with heavy rain and sudden warmer weather, it can lead to serious flooding. If there is a fast snowmelt
on top of frozen or very wet ground, flooding is more likely to occur than when the ground is not frozen. Frozen ground
or ground that is very wet and already saturated with water cannot absorb the additional water created by the melting
snow. Melting snow also contributes to high water levels in rivers and streams. Whenever rivers are already at their full
capacity of water, heavy rains will result in the rivers overflowing and flooding the surrounding land.
Rivers that are covered in ice can also lead to flooding. When ice begins to melt, the surface of the ice cracks and
breaks into large pieces. These pieces of ice move and float down the river. They can form a dam in the river, causing
the water behind the dam to rise and flood the land upstream. If the dam breaks suddenly, then the large amount of
water held behind the dam can flood the areas downstream too.
Broken ice dams are not the only dam problems that can cause flooding. When a large human-made dam breaks
or fails to hold the water collected behind it, the results can be devastating. Dams contain such huge amounts of water
behind them. If sudden break occurs, the destructive force of the water is like a great tidal wave. Unleashed dam water
can travel tens of kilometers, cover the ground in meters of mud and debris, and drown and crush every thing and
creature in their path.
Although scientists cannot always predict exactly when floods will occur, they do know a great deal about when
floods are likely, or probably, going to occur. Deep snow, ice-covered rivers, and weak dams are all strong conditions
for potential flooding. Hopefully, this knowledge of why floods happen can help us reduce the damage they cause.

16. The best title of the passage is .... 18. The following statement is TRUE according to the
2nd paragraph ....
(A) Damages Caused by Flood
(A) Flooding occurs when melting snow makes
(B) Prevention of Potential Flooding
the level of a river rise.
(C) Scientists’ Ability to Predict Floods (B) Pieces of ice move and float down the river,
making the water too cold.
(D) The most Common Natural Disaster (C) Melting snow causes pieces of ice to block the
(E) Potential Circumstances for Flooding river, which causes flooding.
(D) Melting deep snow together with heavy rain
rarely cause flooding.
(E) Ground saturated by water will easily absorb
17. The purpose of the writer is .... melting snow.
(A) to criticize the scientists 19. The word “debris” in paragraph 4 is closest in
(B) to clarify the impacts of flooding meaning to ....
(C) to ask for actions to help flooding victims (A) residue
(D) to inform the readers on how to stop flooding (B) leftovers
(C) wreckage
(E) to explain the conditions that can cause (D) leavings
flooding (E) garbage

- 18 - BTA / Ing / S. Intensif 2023


20. It can be concluded from the text that the scientists (C) have sufficient knowledge to recognize
potential flooding conditions
(A) are required to provide some help when
(D) have come very close to knowing when floods
flooding occurs
will occur
(B) believe that flooding is the most common
(E) are trying to reduce damage caused by floods
natural disaster

Text 5

We all know that mobile phones, cell phones, hand-phones, whatever we want to call them (and shouldn’t we all
be calling them the same thing?) are changing our lives. But it takes a good old-fashioned survey to wake us up to the
glaring reality: they have changed who we are. The mobile phone has indeed changed the way we behave. But perhaps
we don’t realize how much we have become its slave. Consider other elements of the Siemens Mobile Survey: With the
exception of Australia, in every country surveyed the majority polled said they would go back for their phone if they left
it at home (in Australia it was a respectable 30%). If you’ve endured the traffic in Indonesia, the Philippines, and India,
you’ll know what kind of sacrifice some two-thirds of those surveyed are making. I can’t think of anything I would go
back for – except my wallet, maybe, or my clothes.
And even if we remember to bring it, we’re still not happy. Many of us get anxious if it hasn’t rung or a text
message hasn’t appeared for a while (a while being about an hour). Once again of those surveyed, Indonesians (65%)
and Filipinos (77%) get particularly jittery. Australians are more laid back about this (20%), but every other user in Asia
seems to be glancing at the phone every few second. This statistic, I have to say, is highly believable, and the instinct
highly annoying. There’s nothing worse than chatting to someone who constantly checks his or her hand-phone.
Then there’s the fact that mobile phones are not only enslaving the user, they’re trampling the rights of everyone
else. Around a third of folk surveyed acknowledge they get so engrossed in mobile conversations that they’re often
unaware of speaking loudly while discussing their private lives in public. At least most of us agree on one thing: with
the exception of China, Hong Kong and Taiwan, the increasing use of mobile phones has led to a decline in courtesy
and considerate behavior.
The bottom line here is that we are more than a little bit out of control. Mobile phones are great: but if we allow
them to dominate our lives to this extent – interrupting conversations with those around us to take a call, staring at our
phones rather than relating to the world and people around us, sending flirty text messages to random numbers – then I
can only assume that in another 10 years, society as we know it will no longer exist. All we’ll see is a blur of digital data
going out and having all the fun, socializing, falling in love and taking sneaky pictures of each other.

21. The main purpose of the writer is to inform the 23. What makes Asian users of cell phones different
readers about .... from Australian ones?

(A) the advantages of using cell phones in many (A) They mostly use their cell phones for chatting.
Asian countries (B) The number of users in Asia is greater than in
(B) the effects of cell phones on the behavior of Australia.
users (C) They become angry when nobody calls or
(C) the percentage of cell phone users in Asia sends messages.
(D) the popularity of communication technology (D) Their cell phones ring every hour throughout
(E) the various kinds of hand-phones available in the day.
the market (E) They are much more dependent on their cell
phones.
22. Which of the following statements is NOT TRUE
about cell phones?
24. Cell phones have not only enslaved the users but
(A) People become upset when they forget to have also ....
bring their cell phone.
(B) By using cell phones people can change their (A) disturbed other people’s conversation in
lives. public
(C) Cell phones have made users quite enslaved (B) eliminated the existence of social life
(D) The writer is not one who is enslaved by cell (C) made the users behave impolitely
phones. (D) caused the user to become quite considerate
(E) The change in people’s behavior may be (E) prevented people from socializing with each
caused by cell phone other

- 19 - BTA / Ing / S. Intensif 2023


25. The phrase “trampling the rights of everyone else”
in lines 14 means ....

(A) interfering with somebody else’s affairs


(B) prohibiting somebody else’s right to talk
(C) ignoring the rights of other people
(D) rejecting to acknowledge one’s ownership
(E) making use of other people’s property

- 20 - BTA / Ing / S. Intensif 2023


BAHASA INGGRIS
PERSIAPAN UJIAN MASUK PTN (4)
Text 1

Coming out of a period where I struggled with bad health, it feels slightly strange to write an article about the
benefits of living a healthy lifestyle. The reasons I find it strange is that the benefits of a healthy lifestyle should be
obvious to everyone, aren’t they? Surely we all know that having a healthy lifestyle brings extraordinary benefits,
like lots of energy, good mental health, healthy weight, and strengthened defenses against disease and illness. Yet,
as so few actually practice keeping a healthy lifestyle, it is obviously important to put in front of them the key
benefits.
One thing I know for sure is good health is not something you can buy over-the-counter. There is no magical
pill you can take that will deliver you to brilliant health. The only way of doing it is to adopt a healthy lifestyle
including doing physical activity on a regular basis, good nutrition, good rest, and relaxation in order to exist at
optimum level.
“Life is not merely being alive, but being well”, said Marcus Valerius Martialis. It may not be easy to adopt and
then sustain a healthy lifestyle, but the value of doing so will be significant as there are countless benefits associated
with living a healthy lifestyle. Let me give you some examples.
One of the benefits of living a healthy lifestyle is the increased amount of energy. When you eat a healthy diet
including whole grains, lean meats, low-fat dairy products, and fruits and vegetables, your body has the fuel that it
needs to manage your energy level. Having more energy is so important because it enables you to do so much more,
and I know everyone of us can testify to how much better it feels being full of energy rather than feeling lethargic
and empty.
Another of the many benefits of living a healthy lifestyle is that it is good for our mental health. There is
evidence showing that good mental health is very much supported by having a healthy lifestyle. So, things like a
lack of self-esteem, depression, anxiety, and negative state of mind can be reduced by eating right and exercising.

01. Which of the following reflects the author’s 03. The passage can be best summarized as ....
attitude toward the topic of the passage?
(A) living a healthy lifestyle is easy to adopt
(A) Concerned (B) living a healthy lifestyle is difficult to adopt
(B) Neutral (C) there are numerous advantages of living a
(C) Critical healthy lifestyle
(D) the main advantage of living a healthy lifestyle
(D) Indifferent
is good mental health
(E) Impartial (E) the main advantage of living a healthy lifestyle
is the increased amount of energy

02. The paragraph following the passage will likely 04. What will happen when our body has the fuel to
talk about .... manage our energy level?

(A) more facts on living a healthy lifestyle (A) We will feel satisfied.
(B) more stories on living a healthy lifestyle (B) We will feel optimistic.
(C) more benefits of living a healthy lifestyle (C) We will feel contented.
(D) more evidence of living a healthy lifestyle (D) We will feel energized.
(E) more examples of living a healthy lifestyle (E) We will feel fatigue.

- 21 - BTA / Ing / S. Intensif 2023


Text 2
Eating one avocado a day as part of a heart healthy, cholesterol-lowering moderate-fat diet can help improve
bad cholesterol levels in overweight and obese individuals, according to new research published in the Journal of the
American Heart Association.
Researchers evaluated the effect avocados had on traditional and novel cardiovascular risk factors. They
replaced saturated fatty acids from an average American diet with unsaturated fatty acids from avocados. Forty-five
healthy, overweight or obese patients between the ages of 21 and 70 were put on three different cholesterol-lowering
diets. Participants consumed an average American diet consisting of 34 percent of calories from fat, 51 percent
carbohydrates, and 16 percent protein for two weeks before they started one of the following three cholesterol
lowering diets: lower fat diet without avocado, moderate-fat diet without avocado, and moderate-fat diet with one
avocado per day. The lower fat diet provided 24 percent of calories as fat (11 percent from MUFAs). Meanwhile,
the last two moderate fat diets both provided 34 percent of calories as fat (17 percent of calories from
monounsaturated fatty acids/MUFAs). Each participant consumed each of the three test diets for five weeks.
Participants were randomly sequenced through each of the three diets.
Researchers found that, compared to the baseline average American diet, low-density lipoprotein (LDL)–the so
called ‘bad cholesterol’–was 13.5 mg/dL lower after consuming the moderate fat diet that included an avocado. LDL
was also lower on the moderate fat diet without the avocado (8.3 mg/dL lower) and the lower fat diet (7.4 mg/dL
lower), although the results were not as striking as the avocado diet. In addition, several additional blood
measurements were also more favorable after the avocado diet versus the other two cholesterol-lowering diets as
well.
These measurements are all considered to be cardio-metabolic risk factors in ways that are independent of the
heart-healthy fatty acid effects, said a distinguished professor at Pennsylvania State University. “This was a
controlled feeding study, but not the real-world. So, it is a proof-of-concept investigation. We need to focus on
getting people to eat a heart-healthy diet that includes avocados and other nutrient-rich food sources of better fats.”
“In the United States avocados are not a mainstream food yet. They can be expensive, especially at certain times
of the year. Also, most people do not really know how to incorporate them in their diet except for making
guacamole. Guacamole is typically eaten with corn chips, which are high in calories and sodium. Avocados,
however, can also be eaten with salads, vegetables, sandwiches, and lean protein foods like chicken or fish or even
whole.”
In addition to MUFAs, avocados also provide other bioactive components that could have contributed to the
findings such as fiber, phytosterols, and other compounds. The Mediterranean diet includes fruits, vegetables, whole
grains, fatty fish, and foods rich in monounsaturated fatty acids—like extra-virgin olive oil and nuts. Like avocados,
some research indicates that these contain not only better fats but also certain micronutrients and bioactive
components that may play an important role in reducing risk of heart disease.

05. In relation to paragraph 2, what does paragraph 3 07. About avocados described in the passage, the
inform? author has the same belief as the researchers that
(A) Procedures in carrying out the research
(B) Results of the treatment described in (A) research needs to reveal accurately their
paragraph 2 contents
(C) Cholesterol contents that are discussed in (B) combined diets involving avocados need
paragraph 2 further studies
(D) Conditions of the respondents when applying
the diets (C) the current research on avocados has
(E) Relationships between the diets and conclusive findings
cholesterol contents (D) people will take the diet regardless the price of
avocados
06. Based on the passage, the relationship between the
Mediterranean diet and heart disease is like that (E) the fruit nutrients are equal to those of the
between .... Mediterranean diet
(A) fossil fuel and sports car
(B) bush fire and black smoke
(C) severe hunger and big meal
(D) air conditioner and warm room
(E) distressed patient and male doctor
- 22 - BTA / Ing / S. Intensif 2023
08. Which paragraphs of the passage; most effectively 10. Another simple way to restate ideas in the last
illustrate the power of avocados to deal with heart paragraph of the passage is ....
disease?
(A) elements in avocados, similar to those in the
(A) 2 and 6 Mediterranean diet, can lower the threat of
(B) 3 and 6 heart disease
(C) 3 and 5
(B) avocados, rich in useful nutrients like the
(D) 4 and 5
Mediterranean diet, are used to treat people
(E) 5 and 6
with heart disease
09. The author of the passage implies that .... (C) avocados and the Mediterranean diet are more
effective when they are consumed together
(A) rather than the amount, the appropriate diet of with other fruits
avocados cures heart disease
(D) patients’ heart disease can effectively be
(B) avocados are closely linked to the high rate of
heart disease treated by having either the Mediterranean diet
(C) eating avocados than other fruits is a better or avocado feeding
cure for heart disease (E) micronutrients in avocados and bioactive
(D) heart disease happens due to lack of contents in the Mediterranean diet alike heal
consuming avocados heart disease effectively
(E) consuming avocados likely lowers LDL levels
among overweight persons

Text 3

Different forms of energy sources have helped us in many ways from running our vehicles to cooking at home
or generating electricity. There are two types of energy sources–renewable and non-renewable energy resources.
The former is where the energy resources do not get depleted and they can be used repeatedly, whereas non-
renewable energy resources cannot be used again and again.
We cannot even imagine a life without the existence of fossil fuels like crude oil, coal, and natural gas. But
usage of these energy resources creates lots of problems. Global warming, increase in prices of fossil fuels, and the
threat of peak oil are major concerns around the globe but what steps are we taking to overcome all these issues? If
we stop consuming fossil fuels, we can stop global warming and save non-renewable energy resources for our future
generations to deploy. But, at the same time, we need to seek an alternative to meet our requirements. We come
across these topics on television, Internet, and magazines and there is a lot of debate going on with regards to these
topics. Is it possible for us to overcome this crisis? Only alternative energy solutions can help us, as well as, our
environment to some extent.
Industries that deal with alternative energy solutions like solar and wind power thrive hard to save our
environment. Even though modern technologies are being implemented to extract crude oil from places, which were
not reachable earlier, it is still not possible to meet the growing demand. The demand for fossil fuels is increasing
due to the rapid increase in population growth. Alternative energy solutions have lots of benefits when compared to
non-renewable energy resources. Industries have started investing their time and money in researching as well as
setting up power plants.
Certain home users hesitate to go for alternative energy solutions, as initial investment needs to be made.
However, once the equipment is installed, there will be no maintenance costs or other repetitive costs. Apart from
the set up cost, energy from the sun and the wind will always be free in any part of the country and there is no need
for being dependent upon other countries. On the other hand, the prices of renewable energy resources keep
increasing and certain countries have to be dependent upon major oil producing countries. Home users who consume
alternative energy resources can cut down on their electricity bills.
Every system has its own pros and cons but the best and wise thing to do is to weigh the options that are
available and choose the energy resource that offers long-term benefits. When it comes to alternative energy
solutions, setting up solar plants and building windmills is a huge and complicated process but once they are set up,
they offer endless benefits. In case of fossil fuels, oil companies need to locate the proven reserves, dig oil wells,
extract oil, and convert into a more usable form so that it can reach the end consumer.
- 23 - BTA / Ing / S. Intensif 2023
11. The author’s bias regarding pollution is that .... 14. The paragraph preceding the passage most likely
deals with ....
(A) different forms of energy sources have helped
prevent it from happening (A) modern technologies used for energy solutions
(B) industries making use of alternative energy (B) investment needs for equipment installation
contribute the least of it (C) introduction to current energy sources
(C) energy from the sun and wind will produce the (D) alternative energy solutions
minimum level of it (E) sun as an energy source
(D) it is alternative energy that merely offers
solutions to overcome it 15. The ideas in the passage may be best summarized
(E) any natural source of energy cannot avoid that ....
from generating it
(A) the government must support attempts for new
12. In presenting the ideas, the author starts by .... energy installation
(B) solar and wind sources are examples of
(A) describing the types of energy sources alternative energy solutions
(B) referring to non-renewable energy resources (C) setting up solar plants and building windmills
is a complicated process
(C) showing evidence of the roles of energy
(D) initial investment and maintenance costs
sources
alternative energy considerably
(D) stating the needs for alternative energy (E) alternative energy solutions can keep our
solutions environment clean and green
(E) arguing the possibility of using various forms
of energy 16. Based on the passage, it can be hypothesized that
we cannot stop global warming and save non-
13. The author’s attitude towards alternative energy renewable energy resources for our future
may be best described as being .... generations if ....

(A) assertive (A) the prices of renewable energy resources are


decreasing
(B) critical
(B) we keep using fossil-based energy at the
(C) supportive present rate
(D) pessimistic (C) alternative energy is out of industrial solutions
(E) sympathetic (D) present life-styles shift from alternative energy
(E) fewer modern technologies are employed

Text 4

Meteorites are among the rarest materials that exist on earth–far less common than gold, diamonds, or even
emeralds. Therefore, the chances of discovering a new example are slim–even for those who make their living
hunting for and studying meteorites.
A specimen that is thought to be a meteorite, but turns out to be a common earth rock is affectionately and
humorously dubbed a meteor-wrong. The surface of our planet is rich in terrestrial iron oxides such as magnetite and
hematite (many of which will stick to a magnet), dark black rocks such as basalt, and many different types of man-
made metallic by-products such as run off (slag) from old smelters and cast off iron implements. All of these
materials are frequently mistaken for meteorites. Identification of a genuine meteorite takes a practiced eye, but
there are a number of simple tests that can help hopeful rock hounds determine if they have stumbled across a rare
space rock, or just a common earthbound stone.
Meteorites tend to look different from the ordinary terrestrial rocks around them. They do not contain the
common earth mineral quartz, and in general do not contain vesicles. When gas escapes from cooling molten
material, it creates small pinprick holes in a rock’s surface. The volcanic rock pumice, often used in skin care for the
removal of calluses, contains vesicles, which is one of the reasons it is very light in weight. If a suspected meteorite
looks like a sponge, with lots of tiny holes, it is probably volcanic rock or slag of earthly origin.

- 24 - BTA / Ing / S. Intensif 2023


17. The most appropriate title for this passage is ... 19. According to the text, which of the following
statements is FALSE?
(A) Visual Identification of meteor-wrongs
(A) Most meteorites lack substances usually found
(B) The Physical Structure of Kinds of Rocks
in rocks
(C) Distinguishing Meteorites from Other Earthly (B) There are professionals who look for
Minerals meteorites
(C) We can hardly find a meteorite around us
(D) Distinguishing Rare Outer Space Rocks on
(D) A meteorite is less likely to have cavities
Earth (E) Vesicles make a stone hefty
(E) The Difficulties of Finding the Right Method
of Identification
20. The tone of the author is best described as ...

(A) callous
18. What is the author’s purpose in writing this text? (B) playful
(C) acerbic
(A) To explain to the readers how a rock can be (D) pensive
mistaken for a meteorite (E) pragmatic
(B) To show how people use humor and science to
make sense of the world
21. This passage would probably be assigned reading
(C) To show readers that celestial objects are not in which of the following course?
easily found on earth
(D) To provide the readers with the basic (A) Geology
knowledge of identifying a meteorite (B) Geography
(C) Geophysics
(E) To educate the readers to have a trained eye to (D) Geochemistry
recognize meteorites and rocks (E) Geomorphology

Text 5
For decades now, people have been observing the rapid changes in the climate that alarms the world. They have
heard things such as carbon footprint and the melting of the ice caps. Somewhere along the line, they might have
considered adjusting their lifestyle to a more sustainable one. Scientists are still debating whether the rising
temperature of the planet will harm humans or will everyone be able to adapt. Some researchers are even looking
into the positive effects of the climate change.
According to ecological and environmental scientists at Sweden’s Umea University, global warming will not
cause extinction among Arctic species. On the contrary, it could allow species to expand their ranges. The study
began concentrating on the effects of climate change among species in Arctic and sub-arctic Europe. The scientists
created models showing the effects of global warming to these species. The results are surprising, indicating that,
irrespective of the scenario, most species (43 out of 61) will expand and shift their ranges, mostly in a north-easterly
direction, in response to expected climate change if we assume that species are able to colonize all areas that become
climatically suitable, the scientists observed.
Some aquatic species may also benefit from the warming ocean. Research by McMaster University biologist
Graham Scott and University of St. Andrew’s Ian Johnston suggests that certain species of fish growing up at
warmer temperature may adapt quickly to the changing climate. The study shows that fish are hardier after being
raised in warm water. In the study, Zebra fish, a native of Southern Asia, were observed growing in different
temperatures. The fish in the study were raised across temperatures they would naturally live in. Scott and Johnson
found that embryos raised in warm waters could swim faster and developed muscles better suited for aerobic
exercises. These fishes were also able to adjust to colder temperatures well compared to other fishes raised in colder
water.

- 25 - BTA / Ing / S. Intensif 2023


Agriculture may also benefit from the changing mechanics of the atmosphere. Carbon dioxide may aid growth
in plants while their need for water is reduced. For the first time in three years, carbon dioxide concentration
exceeded 400-parts per million. Australian researchers say that partial benefits from the climate change may offset
some of its negative effects. Areas with annual rainfall below 400 millimeters can benefit the most from the
increasing carbon dioxide in the atmosphere. However, the plants still need other key nutrients such as phosphorus
and nitrogen to grow effectively.
As with any story, there are two sides that need to be considered. It is undeniable that global warming has its
possible advantages in the future. However, significant negative impacts are already felt around the globe,
ecologically and economically. The small positive effects cannot offset the greater calamities and problems brought
by global warming. Right now people still have a choice: create a better future environment for the coming
generations, or equip them to adapt in a harsher planet. However, if they delay, they may not have this option.

22. The author’s bias regarding global warming is (C) people’s reasons for refuting the belief of
global warming positive impacts
(A) people have plenty of time to deal with global
(D) arguments for supporting the necessity of
warming
having a more sustainable lifestyle
(B) animals can adapt to natural changes better
than humans (E) the dangers of delaying training for humans to
(C) global warming is also advantageous, but not live in a rough planet condition
most people think so
(D) all planets in the universe are affected by
global warming phenomena 26. The best summary of the passage would most
(E) the Earth can sustain more global warming likely be ....
negative effects in the future
(A) global warming has more advantages than
disadvantages
23. In presenting the ideas, the author starts by ....
(B) some plants can resist the damaging impacts
(A) explaining a global warming theory of climate change
(B) giving examples of global warming research (C) animals and plants have been shown to suffer
(C) stating the natural causes of global warming living in harsh environments
(D) showing global warming as a new (D) unlike what some people believe, global
phenomenon warming have several disadvantages
(E) suggesting possible favorable impacts of (E) global warming can impact positively on the
global warming distribution of some flora and fauna

24. The author’s attitude towards the research results 27. Based on the passage, it can be hypothesized that
in the passage is ....
....
(A) critical
(A) if global warming effects continue, humans
(B) neutral
will have a smaller space to live on land
(C) denying
(D) objective (B) certain animals and plants will adapt
(E) supportive themselves to impacts of global warming
(C) climate changes will not harm humans if
25. The paragraph following the passage will most
humans move to north east areas
likely talk about ....
(D) if global warming lasts for a long time, most
(A) researchers opposing the studies of global species are likely to survive
warming positive effects
(E) animals and plants will not survive the global
(B) the convenience of thinking about global warming effects
warming as something natural

- 26 - BTA / Ing / S. Intensif 2023


BAHASA INGGRIS
PERSIAPAN UJIAN MASUK PTN (5)
Text 1

Passage 1
People still collect books as valuable antiques or for a hobby, but you get virtually all the information you need
from the viewscreen of your home computer system. The computer is linked to a library — not a library of books but an
electronic library where information on every subject is stored in computer memory banks.
Having this service at your fingertips is like having a huge brand-new encyclopedia in your homes at all times. The
computer can tell you anything you want to know, and the information is always the very latest available. There need be
only one central library to which computers in homes, offices, schools and colleges are connected. At the library experts
are constantly busy, feeding in the very latest information as they receive it. In theory one huge electronic library could
serve the whole world!

Passage 2
E-books have not spelled the demise of the local library in New York. In fact, according to a new report from the
Center for an Urban Future, 40.5 million people visited the city’s public libraries, more than all of the city’s
professional sports teams and major cultural institutions combined.
The report “Branches of Opportunity,” looks at the changing role of the city’s libraries in the digital age. It finds
that while public libraries are serving more New Yorkers than ever, they are “undervalued by policymakers and face
growing threats.” New York City’s library system is a unique hybrid. Three organizations — the New York Public
Library, along with the Brooklyn and Queens libraries— operate 206 local branches throughout the five boroughs.

01. Which idea in Passage 2 is different from that in 03. Based on the information in both passages, it can be
Passage 1? hypothesized that ….

(A) E-libraries require sophisticated IT expertise. (A) conventional libraries will continue despite of
the threats
(B) Access to information in e-libraries is
unlimited. (B) policies should be made to conserve common
libraries
(C) Collections of e-libraries are regularly updated.
(C) threats to book publishers becomes more
(D) E-libraries function as a huge information
serious
bank.
(D) IT-based libraries will gain much popularity
(E) In reality most people are still e-library
illiterate. (E) unpopularity of ordinary libraries is obvious

02. Which of the following statements is true according


to both passages? 04. The topic discussed in both passages is ….

(A) People would rather go to the library than (A) the electronic library establishment in the
watch sporting events. digital era
(B) People still dream of using libraries in spite of (B) the importance of libraries in the computer era
time and space.
(C) the preference to choosing public libraries
(C) Access to information in an e-library requires a
(D) the advanced technology in managing libraries
good IT system.
(E) the number of public library visitors in New
(D) Libraries provide quick access to free e-books
York
and newspapers.
(E) The role of library changes fast due to
advanced IT technology.

- 27 - BTA / Ing / S. Intensif 2023


Text 2

A sanctuary may be defined as a place where man is passive and the rest of nature active. Till quite recently nature
had her own sanctuaries, where man either did not go at all or only as a tool-using animal in comparatively small
numbers. But now, in this machinery age, there is no place left where man cannot go with overwhelming forces at his
command. He can strangle to death all the nobler wild life in the world today. Tomorrow he certainly will have done so,
unless he exercises due foresight and self-control in the meantime.
There is not the slightest doubt that birds and mammals are now being killed off much faster than they can breed.
And it is always the largest and noblest forms of life that suffer most. The whales and elephants, lions and eagles, go.
The rats and flies, and all mean parasites, remain. This is inevitable in certain cases. But it is wanton killing off that I am
speaking of tonight. Civilized man begins by destroying the very forms of wild life he learns to appreciate most when he
becomes still more civilized. The obvious remedy is to begin conservation at an earlier stage, when it is easier and better
in every way, by enforcing laws for close seasons, game preserves, the selective protection of certain species, and
sanctuaries.
I have just defined a sanctuary as a place where man is passive and the rest of nature active. But this general
definition is too absolute for any special case. The mere fact that man has to protect a sanctuary does away with his
purely passive attitude. Then, he can be beneficially active by destroying pests and parasites, like bot-flies or
mosquitoes, and by finding antidotes for diseases like the epidemic which periodically kills off the rabbits and thus
starves many of the carnivora to death. But, except in cases where experiment has proved his intervention to be
beneficial, the less he upsets the balance of nature the better, even when he tries to be an earthly providence.

05. The author implies that his first definition of a 07. It can be inferred that the passage is
sanctuary is
(A) part of an article in a scientific journal
(A) totally wrong (B) extracted from the minutes of a nature club
(B) somewhat idealistic (C) part of a speech delivered to an educated
(C) unhelpful audience
(D) indefensible (D) a speech delivered in a court of law
(E) immutable (E) from a polemical article published in a
magazine
06. The author’s argument that destroying bot-flies and
mosquitoes would be a beneficial action is most 08. The purpose of the final paragraph is
weakened by all of the following except
(A) to sum up the main points of the author’s
(A) parasites have an important role to play in the argument
regulation of populations (B) to urge a solution to an increasingly pressing
(B) the elimination of any species can have problem
unpredictable effects on the balance of nature (C) to qualify the author’s definition of an important
(C) the pests themselves are part of the food chain term
(D) these insects have been introduced to the area (D) to propose a program
by human activities (E) to suggest that man should not intervene in
(E) elimination of these insects would require the natural environments
use of insecticides that kill a wide range of
insects

Text 3

Along a lush river delta in what is now northeastern Spain, a herd of dinosaurs munched on ferns and conifers
similar to modern-day cypresses 125 million years ago. These creatures stood out from the others in this Cretaceous
Period landscape by virtue of the unusual sail-like structure on their backs, and experts today can only hypothesize about
its function.
Scientists announced on Wednesday the discovery ___11___ that they named Morelladon, a four-legged herbivore
that measured 6 meters long, near the town of Morella in Spain’s Castellon Province. Morelladon is known from a
partial skeleton including the spines, other vertebrae, pelvic bones, a thighbone and teeth.

- 28 - BTA / Ing / S. Intensif 2023


___12___ from its back was a series of bony spines that formed the sail-like structure that stood about 60
centimeters tall. The sail could help in heat exchange or thermoregulation. It was focused on releasing excess body heat
into the environment, like the ears of the modern-day elephants, or as a storage place for fat to be used during periods of
low food supply. The structure also could have served a display role in attracting mates.
Another plant-eating dinosaur called Ouranosaurus with similarities to Morelladon lived about the same time in
Africa. The biggest sail-backed creature was Spinosaurus, which lived a semi-aquatic lifestyle 95 million years ago in
Africa. At 15 meters and 7 tons, it was the grandest dinosaur predator on record, ___13___. Millions of years before the
rise of the dinosaurs, there were other sail-backed creatures including the carnivorous reptile Arizonasaurus, the
amphibian Platyhystrix and the distant mammal relatives Dimetrodon and Edaphosaurus.
Northeastern Spain during Morelladon’s time alternated between wet and dry periods, with strong temperature
variations ranging from 4 to 5 degrees Celsius. The main predator in the area was Baryonyx, a relative of Spinosaurus,
and there ___14___ other plant-eating dinosaurs around as well as crocodilians and the flying reptiles called pterosaurs.

09. The sentence “Sail-like structures appeared 12. (A) To protrude


periodically in the evolutionary history of
(B) Protruding
vertebrates, often in animal groups not closely
related to one another.” should be in ... (C) Protruded
(D) Being protruded
(A) the first sentence of paragraph 1
(B) the first sentence of paragraph 2 (E) Protrude
(C) the first sentence of paragraph 3
(D) the first sentence of paragraph 4
(E) the first sentence of paragraph 5
13. (A) which was pale in comparison to
10. The italic phrase in the first paragraph means... Ouranosaurus
(B) in its time sitting at the top of the food chain
(A) despite
(B) owing to (C) and considered the most intimidating
(C) looking like
(D) with the exception of the crocodiles
(D) apparent from
(E) characterized by (E) larger even than Tyrannosaurus rex

11. (A) of a dinosaur fossil of the medium-sized


remains
14. (A) have been
(B) of a medium-sized fossil of the dinosaur
remains (B) had been
(C) of the fossil remains of a medium-sized
(C) will be
dinosaur
(D) of the medium-sized remains of a dinosaur (D) were
fossil
(E) are
(E) of the dinosaur remains of a medium-sized
fossil

Text 4

For the Pacific bluefin tuna, sitting at the popular kids’ table surely is not paying off. The stock of the fish is at
historically low levels and is being dangerously overfished, a new report shows. Fishery scientists estimate that the
Pacific bluefin population has declined from its unfished level by more than 96 percent. The report warns that stock
levels likely will not improve by extending the current fishing levels. All the world’s scrombrids – a family that
includes tunas and mackerels – are on the endangered list.
One problem is that the majority of bluefins that fishermen are snagging are under a year old, further hindering the
species’ chance to procreate. But the extreme lack of supply is not deterring many buyers. If anything, low supplies of
the fish have caused it to become a premium commodity, worth buying at extreme prices. A Pacific bluefin was sold for
$1.78 million at an auction in Tokyo.

- 29 - BTA / Ing / S. Intensif 2023


The director of the Pew Environment Group has said “the most responsible course of action is to immediately
suspend the fishery until significant steps are taken to reverse this decline.” She called on the main countries responsible
for Pacific bluefin fishing – Japan, Mexico, South Korea and the U.S. – to take conservational action. So far, there has
been one minor step forward: In June 2012, the Inter-American Tropical Tuna Commission set a quota for the tuna
catch in the eastern Pacific for the first time ever. Some of the other actions were preventing fishing on bluefin
spawning grounds in the northern pacific and creating size limits to reduce the number of juvenile bluefin caught.

15. The points provided in Paragraph 1 of the passage 17. With reference to the passage, an environmentalist
explain that …. will most likely ….

(A) fish such as tunas and mackerels have been (A) agree that the Pacific bluefins should be
badly endangered conserved
(B) catching tunas and their species has been
(B) suggest that the Pacific bluefins be the most
interesting activities
expensive food
(C) scientists think of the need to increase the
population of tunas (C) ban the catching of all bluefins completely all
(D) stock levels of bluefins in fishery have shown a over the world
radical decrease
(E) tunas and tuna-like species are mostly found in (D) advise the fishermen not to do the fishing
the North Pacific activities in the eastern Pacific
(E) suggest bluefins as sacred fish to the Inter-
16. Paragraph 2 is linked to Paragraph 3 in that the American Tropical Tuna Commission
former ….

(A) presents the policy of preventing bluefins from 18. What does the author assume regarding the Pacific
becoming rare; the latter discusses the laws for bluefin tuna?
bluefin conservation
(B) gives a warning to the irresponsible fishermen; (A) The family of bluefin tunas could be extinct
the latter outlines rules for a limited number of because of illegal fishing.
bluefins hunting
(C) discusses a problem and effect of catching too (B) The fish will be well sold over the world if
young bluefins; the latter suggests successfully promoted.
conservations (C) A greater number of fishermen have caught the
(D) talks about efforts to increase the population of fish in the Pacific.
bluefins; the latter proposes the need for free-
zone fishing (D) The fishery of the bluefin has been suspended
(E) mentions the possible regulations for hunting from year to year.
bluefins; the latter presents an issuance of a (E) The bluefin tunas show a gradual annual
bluefin tuna fishing ban decrease in number.

Text 5

Oregon is known for its wonderful diversity of natural landscapes including deserts, deep river canyons, high
mountains, flat well-watered valleys, and a coastline with coves and headlands. Unavoidably, however, the breathtaking
scenery goes hand in hand with geologic processes that can be responsible for recurring and destructive hazards.
In the Pacific Northwest, natural geologic catastrophes may be placed into five categories: floods, landslides,
earthquakes, volcanic eruptions, and tsunamis. All five of these catastrophes have occurred in Oregon within the past
century. Quite often the effect of two or more events ___20___ simultaneously greatly accentuates the destructiveness
of the episode. Floods are nearly always accompanied by landslides and a major quake following a flood results in a
multitude of large and small landslides.
The long view of geology recognizes that ___22___ remarkably slow and that all of these features can be explained
by ongoing natural events. Oregon’s landscape is being continuously shaped by ___23___, heavy winter rainfall, and
ocean storms.

- 30 - BTA / Ing / S. Intensif 2023


Are hazardous geologic occurrences increasing in frequency? There is a tendency to suggest that this is the case. In
a headlong rush for news items, the media will often pump up any event to catastrophic levels even when no deaths and
only minimal property damage have occurred. Additionally, the wonder of modern communication is such that news
stories are pulled in from remote corners of the globe, while 50 years ago they ___24___ missed or rated only a line in a
newspaper. On the other hand, it is true that the increase in population and the dispersal of populations into some areas
previously considered marginal or unsafe dictate that more of these natural disasters will be witnessed than before.
Humans themselves often aggravate disastrous situations by placing themselves in harm’s way, and activities such
as redirecting rivers or clear-cutting may create problems. Moreover, development of highways is so widespread that
natural disasters are much more likely to impact mankind and cause loss of life and property than at any time in the past.

19. What does the italic phrase in the first paragraph 22. The phrase that can be used to complete the sentence
mean in this context? is ....

(A) Something is passed from one hand to another (A) the topography shaping most geologic
hand. processes
(B) One event is immediately followed by another
(B) the processes shaping most geologic
one.
topography
(C) One condition is accompanied by another one.
(D) Two things frequently happen simultaneously. (C) most geologic processes shaping the
(E) Two things are held by two hands. topography

20. (A) occur (D) the shaping processes most geologic


(B) to occur topography
(C) occurring (E) most topography shaping the geologic
(D) occurred processes
(E) to be occurring

21. The concluding sentence of paragraph 2 is .... 23. (A) extreme fishing activities
(B) building constructions
(A) Earthquakes in coastal areas frequently precede (C) agricultural plantation
tsunamis. (D) massive deforestation
(B) Shock wave can also happen in some (E) crustal plate movement
earthquake-prone areas.
(C) The death toll caused by landslides can reach
hundreds of people. 24. (A) had to be
(D) Muddy-debris flows in slopes cause severe (B) will have been
damage to infrastructures. (C) must have been
(E) Landslides can occur in underwater, coastal (D) should have been
and onshore environments. (E) would have been

- 31 - BTA / Ing / S. Intensif 2023


BAHASA INGGRIS
PERSIAPAN UJIAN MASUK PTN (6)

Text 1

1 Although it seems like the spread of spams --unwanted junk e-mails sent to millions of people each day-- is
a recent problem, spam has been around as long as the internet has. In fact, the first documented case of spam
occurred in 1978, when a computer company sent out 400 e-mails via the Arpanet, the precursor to the modern
internet. Now, spam e-mails account for more than two-thirds of all the e-mails sent over the internet, and for some
unlucky users, spam makes up 80 percent of the messages they receive. And despite technological innovations such
as spam filters and even new legislation designed to combat spam, the problem will not go away easily.
7 The reason spammers --the people who and businesses that spread spam-- are difficult to stop is that spam
is so cost effective. It costs a spammer roughly one-hundredth of a cent to send spam, which means that a spammer
can still make a profit even with an extremely low response rate, as low as one sale per 100,000 e-mails sent. This
low rate gives spammers a tremendous incentive to continue sending out millions and millions of e-mails, event if
the average person never purchases anything from them. With so much at stake, spammers have gone to great
lengths to avoid or defeat spam blockers and filters.
13 Most spam filters rely on fairly primitive “fingerprinting” system. In this system, a program analyzes
several typical spam messages and identifies common features in them. Any arriving e-mails that match these
features are deleted. But the fingerprinting defense proves quite easy for spammers to defeat. To confuse the
program, a spammer simply has to include a series of random characters of numbers. The additions to the spam
message change its “fingerprint” and thus allow the spam to escape detection. And when programmers modify the
fingerprint software to look for random strings of letters, spammers respond by including nonrandom content, such
as sports scores or stock prices, which again defeats the system.
20 A second possible solution takes advantage of a computer’s limited learning abilities. So called “smart
filters” use complex algorithms, which allow them to recognize new versions of spam messages. These filters may
be initially fooled by random characters or bogus content, but they soon learn to identify these features.
Unfortunately, spammers have learned how to avoid these smart filters as well.

01. The following statements describe spam, except 03. Smart filters are superior to fingerprinting
.... systems because smart filters ....

(A) It affects millions of internet users. (A) are eventually able to recognize new version
(B) It totals more than 80% of e-mails. of spam
(B) have the ability to learn from their previous
(C) It is beneficial to the general public. mistakes
(D) It is an unwanted message for mass (C) do not need to find common features to
audience. detect spam
(E) It may be a good source of income for (D) are not fooled by random characters or
spammers. content
(E) take advantages of a computer’s limited
learning abilities
02. The best title for the above text is ....
04. The word ‘program’ in line 18 refers to ....
(A) The Development of Spam and Spammers
(B) The Success of the Development of Spam (A) a spam message
Filters (B) a character or a number
(C) The Disadvantages of Using e-mails (C) a type character of spam filter
(D) How to Make Profit through Spamming (D) a common feature
(E) Spam: Problems and Solutions (E) a fingerprinting

- 32 - BTA / Ing / S. Intensif 2023


05. From the text we can conclude that spammers .... (D) make a big profit from the combat against
spam blockers
(A) have always managed to get responses from (E) always send 100,000 e-mails a day to make
internet users profit
(B) produce spam that can never be detected by
spam blockers
(C) have been able to sell cheap products to
users

Text 2

You probably know that calcium and vitamin D are needed to build strong bones. But what you may not
know is that there are certain elements in your diet that can actually reduce bone density, increasing your risk for
osteoporosis.
You need protein to build strong bones, but when you eat too much protein, your body produces chemicals
called sulphates that can cause calcium to leach out of the bones. This effect on bones is more likely to occur with
animal protein than vegetable protein. In the recent Nurses Health Study II, conducted by Harvard Medical School,
116,686 women were followed for 10 years. The researchers found that women who ate red meat at least five times
a week were more likely to have a bone fracture than women who ate red meat only once a week.
In a recent study of 31,527 Swedish women ages 40 to 76, conducted by the Swedish Department of
Toxicology’s National Food Administration, researchers found that women who drink 330 milligrams of caffeine or
more a day - the equivalent of about four cups of coffee - have an increased risk of bone fractures. This risk was
especially noted in women who had a lower consumption of calcium. The researchers did not find an association
between tea consumption and an increased risk for bone fractures. One reason could be that the caffeine content of
tea is typically half that of coffee.
The recent Framingham Osteoporosis Study measured the bone mineral density in the spines and hips of
1.413 women and 1,125 men against the frequency of their soft drink consumption. The researchers concluded that
cola and diet cola beverages (though not other carbonated drinks) may cause bone loss in women and may involve
not just the caffeine, but the phosphorus in colas, too. “It may be that the connection between colas and bone loss is
due in part to the substitution of soda for milk, decreasing calcium intake,” says Kristine Cuthrell, RD. research
nutritionist and project coordinator, Cancer Research Center of Hawaii, University of Hawaii in Honolulu.

06. The author’s attitude toward the topic might best 08. Which of the following best summarizes the
be described as... passage?

(A) concerned (A) In addition to the knowledge on how to


(B) skeptical build strong bones, people also need to
(C) scientific know the causes of bone fractures to prevent
(D) serious osteoporosis.
(E) optimistic
(B) Too much protein reduces bones density as
it generates unsafe chemicals causing the
07. The paragraph following the passage will most
bones to lose calcium.
likely discuss ....
(C) Some elements in our diet such as animal
(A) medical prevention of osteoporosis protein, caffeine, and phosphorus in soft
drinks may cause bone loss and lead to
(B) another element of diet that reduces bone
osteoporosis.
density
(D) Excessive calcium and vitamin D
(C) advantages of consuming more calcium and
consumption builds weaker bones and
vitamin D
decreases the risk of osteoporosis.
(D) an explanation about osteoporosis and its
(E) Drinking coffee is more harmful to human
impact
bones compared to drinking tea as coffee
(E) sources of high calcium and vitamin contains caffeine twice than tea does.

- 33 - BTA / Ing / S. Intensif 2023


09. What will happen if people consume less animal 10. The phrasal verb “leach out” as used in paragraph
protein? 2 can be replaced with

(A) It will increase the chance of osteoporosis. (A) leak out


(B) It will stop spines and hips mineral density
(B) drain
loss.
(C) The protein will be substituted by calcium (C) run out
and vitamin D.
(D) spoil
(D) Human body will produce more calcium to
strengthen the bones. (E) reduce
(E) It will decrease sulphates production, which
decreases the risk of calcium leach.

Text 3

1 Bad luck always seems to strike at the worst possible moment. A man about to go for an interview for his
job gets stuck in traffic. A law student taking her final exam wakes up with a headache. A runner twists his ankle
minutes before a big race, perfect examples of cruel fate.
4 Or are they? Psychologists who study such common mishaps now believe that in many instances, they may
be carefully orchestrated schemes of the subconscious mind. In their book, two psychologists, Berglas and
Baumeister, contend that people often engage in a form of self-defeating behavior known as self-handicapping - or,
in plain terms, excuse making. It’s a simple process: By taking on a crippling handicap, a person makes it more
likely that he or she will fail at an endeavor. Though it seems like a crazy thing to do, Berglas and Baumeister say it
is actually a clever trick of the mind, one that sets up a win-win situation by allowing a person to save face when he
or she does fail.
11 A classic self-handicapper is the French chess champion Deschapelles who lived during the 18th century.
Deschapelles was a phenomenal player who quickly became champion of his region. But when competition grew
tougher, he adopted a new condition for all matches: He would compete only if his opponent would remove one of
Deschapelle’s pawns and make the first move, increasing the odds that Deschapelles would lose. If he did lose, he
could blame it on the other player’s advantage and no one would know the true limits of his ability; but if he won
against such odds, he would be all the more revered for his amazing talents. Psychologists now use the term
“Deschapelles coup” to refer to acts of self-sabotage rampant in today’s world.
18 Overall, men are more likely than women to make excuses. Several studies suggest that men feel the need
to appear competent in all realms, while women worry only about the skills in which they’ve invested heavily. Ask a
man and a woman to go scuba diving for the first time, and the woman is likely to jump in, while the man is likely to
first make it known that he’s not feeling too well.
22 In fact, the people most likely to become chronic excuse makers are those obsessed with success, says
Berglas. Such people are so afraid of being labelled a failure at anything that they constantly develop one handicap
or another in order to explain away failure. Furthermore, over the long run, excuse makers fail to live up to their true
potential, thwart their own goals and lose the status they care so much about. And despite their protests to the
contrary, they have only themselves to blame.

11. Which of the following is the topic of the whole 12. Which of the following sentences is TRUE?
essay?
(A) Self-handicapping is ultimately a destructive
(A) Avoiding failure by using self-handicapping behavior.
(B) Chronic excuse making can be a sign of
(B) Self-handicapping as a cause of depression
depression.
(C) The life of a classic self-handicapper, (C) Self-handicapping is used by women to save
Dechapelle their face.
(D) Self-handicapping is usually used by men to
(D) A comparison between the ways men and
save their face
women avoid failure
(E) People who commit self-handicapping do
(E) Performing self-handicapping as a way to not care about success.
improve status

- 34 - BTA / Ing / S. Intensif 2023


13. Which of the following is an example of self- 15. It can be inferred from the passage that a student
handicapping? who wanted to engage in self-handicapping
would ....
(A) Leaving in plenty of time for an important
meeting (A) join study groups for difficult subjects
(B) Taking a good care of our physical condition (B) go to bed early the night before an exam
prior to a game (C) study as hard as possible for a big exam
(C) Having a good preparation before meeting a (D) eat plenty of spicy food hours before an
client exam
(D) Going to bed early before a big examination (E) try to cheat on a test to get a high score
(E) Practicing running a few minutes before a
race

14. The word “rampant” in line 17 is closest in


meaning to ....

(A) interesting
(B) widespread
(C) occasional
(D) plausible
(E) intermittent

Text 4

Most of us have heard the good advice that we need to eat less sugar, and rightly so. However, despite the
numerous warnings by health authorities of the ill effects of sugar, the majority of the population is still consuming
sugar on a daily basis in some form or other.
We do not have to consume white, refined sugar to be consuming sugar. Sugar includes glucose, fructose (as in
fruit sugar), lactose (as in milk), sucrose (as in table sugar), maltose or malts (as in rice malt and honey), jam
(contains concentrated juice, which is high in fruit sugar), maple syrup, corn syrup, palm sugar (traditionally used in
macrobiotic cooking), and the very deceiving organic brown sugar, which is not all that different from white sugar.
Even alcohol is a sugar. All of these sugars are problematic in many different ways.
The body changes sugar into 2 to 5 times more fat in the bloodstream than it does starch. With 146 proven
reasons why sugar is bad for us, is there perhaps one single reason as to why we might need it? The only interesting
thing about sugar is that it tastes good and makes us temporally feel good. This is an area worth exploring.
According to Traditional Chinese Medicine, a 5000-year-old wisdom of self-contained knowledge of healing, we all
need sweetness in our life. We need six tastes: sweet, sour, salty, astringent, bitter and pungent to stimulate the taste
buds on our tongue at main meals, in order to experience satiety.
Many people really try hard to avoid sugar, and do not sweeten their tea or coffee, yet they crave sugar in some
other form, such as chocolates, cakes, ice cream or even fruit - dates and figs. Dates are 99% sugar, in the form of
fructose. When a person is in metabolic balance, they do not crave sugar. If they do, it is a sign of a metabolic
imbalance and it can be corrected without having to consume sugar.

16. What is the author’s attitude toward the topic of 17. The paragraph following the passage will likely
the passage? talk about …

(A) Considerate (A) a way of remedying metabolic inequality


(B) details of a person with metabolic problems
(B) Determined
(C) a comparison of sugar contents in tea and
(C) Persistent cakes
(D) other sweetener types for replacing sugar
(D) Thoughtful
(E) effects of consuming too much sugar
(E) Cautious

- 35 - BTA / Ing / S. Intensif 2023


18. The passage can be best summarized as .... (D) tastes in the food complement each other
(E) our body suffers from metabolic imbalance
(A) all types of sugar are problematic to human
health 20. The word “satiety” as used in paragraph 3 most
(B) humans actually need sugar every day in the nearly means
form of food
(C) the only function of sugar is to satisfy our (A) Completion
need of sweet taste (B) Binge
(D) given the ill effect of sugar, people should (C) Bulge
avoid consuming it (D) Appetite
(E) over consumption of sugar will impact both (E) Satisfaction
physically and psychologically

19. Based on the passage, it can be hypothesized that


we will feel full if ....

(A) there is sugar in the food


(B) we eat or drink something sweet
(C) alcohol is served during meal time

Text 5

A forgotten issue in urbanism is land use during the nighttime, with problems such as noise and dirt, which
could be improved with information from Twitter. At least this is what Enrique and Vanessa Frias-Martinez believe,
computer science researchers at Telefonica Research and the University of Maryland (USA) respectively, who have
suggested using geolocalized tweets for urban planning and land use.
Enrique Frias-Martinez explained that geolocalized tweets can be a very useful source of information for
planning since it is an activity carried out by a large number of people who provide information on where they are at
a specific time and what they are doing. The researcher points out that thanks to the increased use of smartphones,
social networks like Twitter and Facebook have made it possible to access and produce information ubiquitously.
These networks generate tags with the event’s geolocation. The new technique “automatically determines land
uses in urban areas by grouping together geographical regions with similar patterns of Twitter activity,” says the
researcher. Using aggregate activity of tweets, the Frias-Martinez siblings have studied land use in Manhattan,
Madrid and London. In the first two cases they identified four uses: residential, business, daytime leisure (mainly
parks and tourist areas) and nightlife areas. The study has determined that, in Madrid, nighttime tweet activity is
concentrated on weekends and in Manhattan, on weekdays. On the other hand, London is characterized by its
tweeting activity in daytime leisure areas.

21. Which of the following is the main idea of the 22. The author’s purpose in writing the passage is to
passage? ....

(A) Many people like to use geolocation tags on (A) compare Facebook and Twitter users in
Twitter. terms of geolocation tags.
(B) Information from social networks can be (B) argue that social networks do not necessarily
accessed universally. reflect users’ activities.
(C) Geolocalized tweets can be useful for urban (C) persuade people to stop using geolocation
planning. tags to protect their privacy.
(D) Complex patterns of Twitter activities can (D) inform the possible use of Twitter data for
finally be identified. studying and planning land use.
(E) Problems concerning urbanism are often (E) illustrate the urgency of solving various
neglected and forgotten. problems concerning urban planning.

- 36 - BTA / Ing / S. Intensif 2023


23. According to the passage, which of the following 25. This text would probably be found in ....
is NOT TRUE about the study?
(A) a popular science magazine
(A) It shows that people in London mostly tweet (B) a brochure
in daytime leisure areas. (C) an academic journal
(B) The focus of the study is land use in (D) a newsletter
Manhattan, Madrid and London. (E) a tabloid
(C) Residential, business, daytime leisure and
nightlife areas could be identified.
(D) Manhattan and Madrid’s nighttime tweet
activities are concentrated differently.
(E) The researchers could identify four land uses
in Manhattan, Madrid and London.

24. The tone of this passage is ....

(A) harsh
(B) neutral
(C) critical
(D) didactic
(E) satirical

- 37 - BTA / Ing / S. Intensif 2023


BAHASA INGGRIS
PERSIAPAN UJIAN MASUK PTN (7)
Text 1

The present study sought to document the word reading and comprehension levels attained by children who
were implanted by 5 years of age. It was hyphothesized that the improved speech perception abilities acquired with
cochlear implantation would promote phonological coding skills. (01) ...
Three subtest from diagnostic reading assessment batteries standardized on hearing children were
administered to 181 children between 8 years 0 month and 9 years 11 month of age who had 4 to 6 years of implant
experience. (02) ... it included a lexical decision task, a rhyme task and the digit span subtest of the Wechsler
Intelegence Scale for Children.
Over half of the children scored within the average range for their age compared with the normative data
for hearing children. (03) ... they were higher nonverbal intelligence, higher family socio-economic status, female
gender and later onset of deafness (between birth and 36 months).
(Adapted from journal of ear and hearing)

01. Which option best completes (01)? (D) Consequently, the assessment was used to
evaluate the reading skills.
(A) The finding showed that the hypothesis was (E) Instead, an interview consisting of many
accepted different tasks was conducted
(B) The implantation was eventually shown to
be successful 03. Which option best completes (03)?
(C) The following paragraph would discuss the
findings of the study (A) The results were categorized based on the
(D) The objective of the study was to see the subject's status.
subjects reading skills
(B) Data were collected on the basis of several
(E) It would also facilitate the acquisition of
different items.
beginning reading skills
(C) Reading competence was associated with
02. Which option best completes (02)? three aspects.
(D) The subjects consisted of different age
(A) Likewise, the children were obliged to take a
children.
reading test.
(B) In addition, a battery of processing measures (E) Findings showed various levels of reading
was administered. skills.
(C) However, standard measurement was
applied to test the subjects.

Text 2

These are amazing advances, but while all this scientific back-slapping is going on, the dark cloud on the
horizon is the emerging Ebola epidemic in West Africa and the warning undercurrent that comes with it. At the time
of writing at least 7000 people have been infected and half of those have died. The CDC in America also estimate
that, because the level of reporting is so poor, the numbers can, in all likelihood, be doubled or even tripled. And
because the rates of infection appear to be growing exponentially, tens of thousands, or even millions, might
ultimately be affected.
To put the scale of the present situation into perspective, since the first recorded case of Ebola in the
Democratic Republic of Congo 38 years ago there have been fever than 2.500 death documented in total. So this
single present outbreak is already three times larger than the entire Ebola death toll ever. It's also no longer just an
African problem. The West has had its own wake-up call this week as the US and Spain, countries previously
regarded as immune to the threat thanks to modern medicine, have reported imported cases of the condition and,
despite strict infection-control guidelines and practices, onward transmissions of Ebola on their home soil.

- 38 - BTA / Ing / S. Intensif 2023


What is remarkable though is that, while Ebola is terrifying and dramatic in its impact when it causes an
outbreak, it appears to be a relatively easy agent to fight. Experimental vaccines tested so far on animals have been
impressively effective, protecting against even injection of the live Ebola virus. But because they are at a test stage,
these agents, which will be critical if we're to nip this outbreak in the bud, are nowhere near ready for mass
production. Trials are only now getting underway of human versions of the vaccines in Oxford, UK, and the US.
"Way too late," many are saying, to prevent the inevitable.
So why is it that, nearly 40 years after Ebola first surfaced, the world finds itself in a state of panic, and up
to ten thousand people are dead, owing to a bug that's probably preventable thanks to scientific research done
decades ago? The answer is that Ebola was regarded as someone else's problem. It was a tropical disease of low
importance and (presumed to be) constrained by geography and climate to a part of the world that held little
economic interest to the rest of us. But there in lies a salutary lesson: because if even a tiny fraction —less than 1%
— of what the present outbreak is now costing the world in terms of lost productivity, humanitarian aid and human
lives lost had been spent 20 years ago to develop an Ebola vaccine, we probably wouldn't be in this position now.
It's easy to dismiss tropical diseases as an issue that won't affect the West, but the present situation is a warming shot
across our bows that we ignore at our peril.

04. How are the ideas in paragraphs 1 and 2 related? 07. Which sentences most effectively illustrate the
current Ebola prevalence?
(A) Paragraph 1 and 2 highlight the horror of the
Ebola oubreak. (A) At the time of writing … be affected.
(B) Paragraph 1 contradicts the explanation of (paragraph 1)
Ebola in paragraph 2. (B) To put the scale … Ebola death toll ever.
(C) Paragraph 1 and 2 argue Ebola can be (paragraph 2)
prevented with the latest vaccines. (C) Experimental vaccines tested … for mass
(D) Paragraph 1 states the causes of Ebola and production. (paragraph 3)
paragraph 2 explains the effects. (D) So why is it that, nearly … as someone
(E) Paragraph 1 defines an epidemic disease and else's problem. (paragraph 4)
paragraph 2 gives the examples. (E) But there in lies … at our peril. (paragraph
4)
05. Which of the following is most relevant with the
idea of Ebola outbreak described in the passage? 08. Paragraph 3 implies that ...

(A) Scientists are developing a new drug, but (A) ebola vaccines are very complex and time
they are running out of fund. consuming to create.
(B) A mother does not comfort her crying son (B) ebola vaccines should have been tested on
who merely wants her attention. humans much earlier.
(C) A diabetic man ignored a slight wound on (C) it is in appropriate for ebola vaccines to be
his toe, then he got infected horribly safely tested on humans.
(D) Students choose not to study for an (D) the UK and US are countries that are in
upcoming test since the test was book-open urgent needs of ebola vaccines.
(E) A recently launched book has caused a (E) testing ebola vaccines on animals takes
problem due to its controversial content many years before their use on humans.

06. The assumption the author has about the West is, 09. Which of the following best restates ideas of
that ... paragraf 4?

(A) they act fast to prevent a disease only if it is (A) Ebola has been a major problem for 40
important for them years, so it will be over soon
(B) tropical disease are considered exotic and (B) Ebola only affects the countries with little
dangerous by the people economic and political power
(C) it is difficult to convince them that they need (C) The number of patients dying from Ebola
to send medical help to Africa will likely to remain the same
(D) they are really interested in developing ebola (D) Scientists are developing vaccines that can
vaccines for mass production be used for animals and humans
(E) they intentionally postpone developing ebola (E) Had its vaccines been seriously created
vaccines because it is expensive earlier, Ebola could have been cured

- 39 - BTA / Ing / S. Intensif 2023


Text 3

As spring arrives, albeit quite late this year for many of us, we notice the changing of seasons by the
weather, but also by a host of natural events. Depending on where you live, the crocuses may poke through the
snow, the killdeer may be back, or the cherry trees may bloom. There is an orderly sequence of events that seems to
occur, with various spring flowers appearing in order, red maple buds bursting into new leaves, or the old lilac by
the barn scenting the air. This seasonal cycle of natural phenomena is called phenology. Global climate change
appears to be interfering with the phenology of many species, at the very heart of species interactions.
In temperate regions like the northern half of the United States, there is comparatively little biological
activity in winter. Most plants are dormant, and so are the insects feeding on them. In turn, animals that rely on these
insects, such as bats and birds, are hibernating or spending the cold months in more southerly locations. Ectotherms
like reptiles and amphibians, which take their body warmth from their environment, also have active phases tied to
the seasons. This long winter period constrains all the growing, breeding, and dispersing activities that plants and
animals do to a short favorable window. That's what makes spring so vibrant, with plants flowering and putting on
new growth, insects emerging breeding, and birds flying back to take advantage of this short-lived bounty. The
onsets of each of these activities add up to so many phenological markers.
Different organism respond to different cues to initiate seasonal activities. Many plants will start growing
leaves again after a set period of dormancy, which very roughly dictates the leaf-out window. Cue that more
precisely determine when the buds break can be soil temperature, air temperature, or water availability. Similarly,
temperature cues can promote the beginning of insects activity. Day length itself can be the operative trigger for
some seasonal events. It is only when there are a sufficient number of day light hours that reproductive hormones
will be produced in many bird species.
The most energy-demanding period in the life of most animals is when they reproduce. For that reason, it is
to their advantage to coincide breeding (and for many, the raising of young) during a period when food is most
abundant. Caterpillars should hatch just as the young tender leaves of oak tree emerge, before they harden and
becomes less nutritious. Breeding songbirds need to time the hatching of their young just during that peak in
caterpillar activity. So they can take advantage of this rich source of protein to feed their offspring. Many species
have evolved to exploit peaks in resource availability, so all these seemingly independent phenological events are
indeed part of a complex web of precise interactions. Disruptions in seasonal events can have profound effects on
ecosystems.

10. The author's bias regarding spring phenology is 12. The author's attitude towards sensational cycle of
that ... natural events is ...

(A) spring arrives late (A) assertive


(B) spring is so enthusiastic (B) objective
(C) various spring flowers appear in order (C) optimistic
(D) birds are flying back to take advantage of (D) supportive
short-lived bounty (E) sympathetic
(E) many species have evolved to exploit peaks
in resource availability
13. The paragraph following the passage most likely
deals with ...
11. In presenting the ideas, the author starts by ...
(A) the interference of global climate change to
(A) showing evidence of why scientists are species phenology
concerned with phenology
(B) reasons for scientists to be concerned with
(B) describing the natural characteristics and
phenology
definition of phenology
(C) stating the needs for triggering phenological (C) processes in climate change to affect
events phenology
(D) arguing how climate change is affecting
(D) the seasonal cycle of natural phenomena
phenology
(E) referring to global climate change (E) factors triggering phenological events

- 40 - BTA / Ing / S. Intensif 2023


14. The ideas in the passage may be best 15. Based on the passage, it can be hyphothesized
summarized that ... that reproductive hormones in many birds'
species will not be produced if ...
(A) in winter plants are dormant and animals are
hibernating (A) there is an inadequate number of day light
(B) seasons change as the natural events do, hours
following them (B) seasons as well as local weather change a bit
(C) winter constrains plants and animals from (C) those feeding on insects are hibernating
growing and breeding (D) no orderly sequence of event occur
(D) different organisms respond to different cues (E) global climate change happens
for seasonal activities
(E) phenological events represent a complex
chain of natural interactions

Text 4
Many people now get news from social media, websites and networks. It can often be difficult to tell
whether stories are credible or not. Thus, there has been an increase in false information. False information is news,
stories or hoaxes created to deliberately deceive readers. Usually, these stories are created to either influence
people's views, push a political agenda, or cause confusion.
There are four types of fake information. First, clickbait stories use sensationalist headlines to grab
attention at the expense of truth or accuracy. Second, propaganda are created to mislead audiences. They are also
aimed to promote a biased point of view or particular political cause. The third type is satire. Lots of websites and
social media accounts publish fake news stories for entertainment and parody. Fourth, there is sloppy journalism.
Sometimes reporters or journalists may publish a story with unreliable information or without checking all of the
facts. All of these types need to be aware of and avoided by readers.

16. According to the passage, false information is 18. To produce reliable information, journalist ...
created to deliberately ... readers. double check all of the facts they have.

(A) mislead (A) would rather


(B) betray (B) could
(C) blame (C) might
(D) dismiss (D) must
(E) disappoint (E) had to

17. Based on the passage, the word “sloppy” in


paragraph 2 means ...

(A) firm
(B) slippery
(C) careless
(D) ignorant
(E) convincing

Text 5
Inflammation is actually a defense mechanism in our body, and helps in fighting against injuries and
infections. However, chronic inflammation is known to result in weight gain and disease, in addition to making us
prone to cancer and rheumatoid arthritis. There are several anti-inflammatory foods we can eat to keep disease at
bay.
Broccoli is one to mention. It is extremely nutritious vegetable known to lower the risk of developing
cardiovascular disease. It is also packed with calcium, vitamin A, vitamin C, and potassium. We can incorporate
broccoli in soups and salads, or stir fry it with other veggies and seasons them up with salt and pepper for filling
dish.

- 41 - BTA / Ing / S. Intensif 2023


It may not be familiar to us that eating berries helps our body produce natural killer cells. Also, berries are
loaded with vitamins, minerals and fiber. Blackberries, blueberries, strawberries, raspberries are easy to eat, and are
powerful anti-inflammatory foods: they are also good for our heart health.
1
Olive oil is beneficial for our health in multiple ways. 2It is known for its cheap price. 3Being a key
ingredient in Mediterranean cooking, olive oil is known to lower cholesterol and promote heart health. 4This is
because it is rich in monosaturated fats. 5These types of fats are good for our body.
Fatty fish are an excellent source of omega-3 fats. We can have salmon, sardines and even anchovies. Plus,
they also give us a protein boost and keep kidney disease, heart disease and diabetes at bay. The taste does not seem
to be a significant factor. They are amazing anti-inflammatory foods and are quite filling too.

19. The author would apparently agree that the 21. The author describes anti-inflammatory foods
relationship between inflammation and disease in that can reduce the amount of food taken in
paragraph 1 is similar to the phenomenon of ... paragraphs ...

(A) salt and high blood pressure (A) 1 and 2


(B) anemia and iron deficiency (B) 1 and 3
(C) water and dehydration (C) 2 and 3
(D) iron and heart disease (D) 2 and 5
(E) sugar and energy (E) 3 dan 5

20. Which sentences is NOT relevant to the topic


discussed in paragraph 4?

(A) Sentence 1
(B) Sentence 2
(C) Sentence 3
(D) Sentence 4
(E) Sentence 5

Text 6

Digital scent technology refers to the online communication using the human nose. It is a crucial
application for electronic nose (e-nose). E-nose is a device that analyses the chemical content of a specific odour and
identifies it. Digital scent technology enables the transmission of odour over the internet. The technology works with
the help of olfactometers and electronic noses. Digital scent technology provides its users with several benefits. One
of the benefits is they can smell the product before buying it online. For instance, a California-based company,
digiscents Inc. has created a small device called iSmell. It has its own driver. It can be connected through a personal
computer via serial ports.
Digital scent technology works by coding aromas. Such aromas are then downloaded in computers as
audible sounds. This technology enables the users to produce and modify their personal fragrances. High cost of
electronic nose and delays between successive smell tastes are some restraints faced by the global digital scent
technology market. For instance, digital scent technology used in the field of defence and clinical diagnosis is
relatively expensive. This acts as a barrier for further expansion of the market. Unpredictability of air flows is
another major factor that hinders the transmission of digitalized smell. This problem also restrics the usage of digital
scent technology.
The global digital scent technology market can be classified based on its type, application, and end-user
industry. In terms of type, the market can be segmented into scent synthesizer and electronic nose. The electronic
nose market segment is expected to hold substantial share of the market because it has been extensively used in the
defence and healthcare sectors. In terms of application, the digital scent technology market can be segmented into
mobile phones, music and video games, smelling screens, medical diagnostic products, and so forth. In terms of end-
user industry, one of the sectors enjoying this technology is the healthcare sector. Digital scent technology is used in
aromatheraphy. This may cure certain diseases by employing different types of smells.

- 42 - BTA / Ing / S. Intensif 2023


22. What is the relationship between the ideas in 25. From this passage we can assume that ...
paragraphs 1 and 2 ?
(A) a pilot study on the capability of digital
(A) The brief definition of digital scent scent technology has been conducted
technology in paragraph 1 is further
(B) high predictability of air flow helps the
elaborated through a number of useful
smooth development of digital scent
application programs in paragraph 2
(B) Paragraph 1 mentions several devices to (C) digital scent technology still needs some
support digital scent technology followed by improvements
the explanation of developing such devices
in paragraph 2 (D) aromatheraphy is the only benefit of digital
scent technology in healthcare sector
(C) The authors agreement of a digital scent
technology development in paragraph 1 is (E) scent synthesizer and E-nose are less
contrasted with the idea of its termination in developed compared to other digital scent
paragraph 2 devices
(D) The definitions of digital scent technology in
paragraph 1 is followed by a number of its
applications in various sectors in paragraph
2 26. Which of the following can be concluded from
(E) Paragraph 1 contains the overview of digital the passage?
scent technology followed by the obstacles
in its development in paragraph 2 (A) Digital scent technology provides a
particular market to promote its benefits
23. Which of the following best restates the (B) Digital scent technology is just a dream of
underlined sentence in paragraph 2? people working in a technology sector

(A) The success of smell tastes in the global (C) Digital scent technology won't be able to
digital scent technology market depends on satisfy human's need
the price of electronic nose (D) Digital scent technology should improve the
(B) The digital scent technology market won’t effectiveness of human’s online interaction
be successful if it is still expensive and has
not been perfectly developed (E) Digital scent technology can be used in
(C) People face the problem of delay in mobile phones and video games
developing technology related to digital
scent in the global market
(D) The production of electronic nose has been
delayed because it has been limited by the
global digital scent technology
(E) To succeed in the global digital scent
technology market, delays between
successive smell tastes shouldn’t result in
overpriced e-nose

24. All of following statements are true according to


the passage, EXCEPT ...

(A) Digital scent technology benefits people


who want to purchase certain product online
(B) Unpredictable air flow is one of the
obstacles in digital scent technology
(C) iSmell is the latest development of digital
scent technology
(D) digital scent technology is an expensive
technology
(E) electronic nose is helpful in developing
digital scent technology

- 43 - BTA / Ing / S. Intensif 2023


BAHASA INGGRIS
PERSIAPAN UJIAN MASUK PTN (8)
PASSAGE 1

As a general rule, fat cells have the function to store energy. In brown fat cells, however, energy is used up
as heat -- brown fat thus serves as a biological heater. Most mammals therefore have this mechanism. In humans it
keeps newborns warm, in human adults, brown fat activation positively correlates with cardio-metabolic health.
"Nowadays, however, we're toasty warm even in winter," explains Prof. Dr. Alexander Pfeifer from the
Institute of Pharmacology and Toxicology at the University of Bonn. "So our body's own furnaces are hardly needed
anymore. At the same time, we are moving far less than our ancestors and eating an increasingly energy-dense diet.
………………… They cause the cells to gradually stop functioning and eventually even die.”
"Research groups around the world are therefore looking for substances that stimulate brown fat and thus
increase fat burning," says Pfeifer. Together with a group of colleagues, the team at the University of Bonn has now
identified a key molecule named inosine that is capable of burning fat. "It is known that dying cells release a mix of
messenger molecules that influence the function of their neighbors," explains Dr. Birte Niemann from Pfeifer's
research group. Together with her colleague Dr. Saskia Haufs-Brusberg, she planned and conducted the central
experiments of the study. "We wanted to know if this mechanism also exists in brown fat."
The researchers therefore studied brown fat cells subjected to severe stress, so that the cells were virtually
dying. "We found that they secrete the purine inosine in large quantities," Niemann says. More interesting, however,
was how intact brown fat cells responded to the molecular call for help: They were activated by inosine (or simply
by dying cells in their vicinity). Inosine thus fanned the furnace inside them. White fat cells also converted to their
brown siblings. Mice fed a high-energy diet and treated with inosine at the same time remained leaner compared to
control animals and were protected from diabetes.
The inosine transporter seems to play an important role in this context: This protein in the cell membrane
transports inosine into the cell, thus lowering the extracellular concentration. Therefore, inosine can no longer exert
its combustion-promoting effect.

01. The primary purpose of the passage is to 03. The word “used up” as used in the first
paragraph is synonymous with
(A) discuss the assumptions and reasoning
behind a theory (A) coagulated
(B) agglomerated
(B) describe the aim and method of an
(C) engineered
experiment
(D) generated
(C) present and analyze conflicting data about a (E) dissipated
phenomenon
04. Which of the following sentences can complete
(D) show the innovative nature of a procedure the missing sentence in paragraph 2 of the
used in a study passage?
(E) analyze the genetic components of a
substance (A) Furthermore, the number of severely
overweight people worldwide continues to
increase.
02. What could be the best title of the passage?
(B) All kinds of calorie-rich food mass-
(A) Benevolent Brown Fat produced for commercial resale have
detrimental effects on cells.
(B) How to Burn Fat Effectively
(C) These three factors are poison for brown fat
(C) Molecule To Boost Fat Burning cells.
(D) Metabolic Imbalance due to Energy-Dense (D) It is, therefore, paramount to lead a healthy
Diet life in early ages.
(E) Animal Experiment on Fat (E) This fact is puzzling both the researchers
and scientists.

- 44 - BTA / Ing / S. Intensif 2023


05. The second paragraph of the passage establishes 07. The word “siblings” as is used in the fourth
Dr. Pfeifer’s mood of paragraph refers to

(A) cautious optimism (A) large quantities of purine inosine


(B) nervous apprehension (B) mice
(C) jaded dismissal (C) fat cells
(D) initial concern (D) inosine molecules
(E) dramatic anticipation (E) control animals

06. According to the fourth paragraph, the relation 08. What topic does the paragraph succeeding the
between distressed brown fat cells and secretion passage most likely discuss?
of purine inosine is similar to the phenomenon
between (A) consequences of having insufficient amount
of inosine
(A) fire and water (B) effects of accumulated inosine
(B) dentistry and tooth (C) substitutes for inosine
(C) environment and ecologist (D) production of artificial inosine
(D) fossil fuel and polution (E) detail about roles of inosine transporter
(E) population growth and self-sufficiency

PASSAGE 2

Quite recently a group of researchers at the Department of Energy's Oak Ridge National Laboratory
(ORNL) has definitively linked the function of a specific domain of proteins important in plant-microbe biology to a
cancer trigger in humans, knowledge that had eluded scientists for decades. The team's findings, published in Nature
Communications Biology, open up a new avenue for the development of selective drug therapies to fight a variety of
cancers such as those that begin in the breast and stomach.
ORNL scientists set out to prove experimentally what they first deduced with computational studies: that
the plasminogen-apple-nematode (PAN) domain is linked to the cell proliferation that drives tumor growth in
humans and defense signaling during plant-microbe interactions in bioenergy crops. The association was first made
as researchers explored the genomes of crops like poplar and willow.
In the latest study, the ORNL team pinpointed four core amino acids called cysteine residues in the HGF
protein critical to the PAN domain's function and studied their behavior in human cancer cell lines. They found that
mutating any one of those amino acids turned off the signaling pathway known as HGF-c-MET that is abnormally
heightened in cancer cells, causing them to rapidly multiply and spread.
Since cysteine residues are known to have many functions, the scientists also randomly tested other
cysteines throughout the protein and found that none of them had the same impact on shutting down HGF-c-MET
signaling. Mutating the four key cysteines had no effect on the overall structure of the protein, and merely inhibited
the cancer signaling pathway, the team noted in the study.
Disrupting the right signal is one of the biggest challenges in developing new cancer therapies, said ORNL
geneticist Wellington Muchero. "It's very difficult to engineer molecules to interfere with an entire protein," he said.
"Knowing the specific amino acids to target within that protein is a big advancement. You don't have to search the
entire protein; just look for these four specific residues." The identification of those core residues is a testament to
the predictive power the team has built at ORNL, leveraging the lab's expertise in plant biology and biochemistry,
genetics, and computational biology, as well as its supercomputing resources and the CRISPR/CAS-9 gene editing
tool.

09. From paragraph 1 of the passage, the phrase


“knowledge that had eluded scientists for (C) it’s been decades that the knowlege interests
decades” means that scientists
(D) it’s been decades for scientists to
(A) it is a knowledge confusing scientists for comprehend the knowledge
decades (E) scientists have dealt with this knowledge for
(B) it is a knowledge scientists have failed to decades
grasp for decades

- 45 - BTA / Ing / S. Intensif 2023


10. The second paragraph indicates that, what 14. The passage most strongly suggests that the
enticed ORNL scientists about PAN domain was researchers at the Department of Energy's Oak
its Ridge National Laboratory share which
assumption?
(A) computational studies being used in the
initial research (A) Cancer is definitively curable, no doubt
(B) potential to drive tumor growth in humans about it.
(C) association with cell proliferation
(B) Cancer is the world’s biggest killer.
(D) lack of defense signaling during plant-
microbe interactions in bioenergy crops (C) More research on cancer cure should result
(E) quantity found in the genomes of crops like in good news.
poplar and willow
(D) Research on cancer cure has been conducted
in vain.
11. According to the passage, it is to conclude that (E) Funds and complicated procedures are
there might be a way to control the cancer needed to conduct more thorough research
growth in humans i.e. by taking the following on cancer cure.
steps

(i) mutating four cysteine residues altogether


(ii) preventing the cells from proliferating 15. The stance the ORNL geneticist Wellington
(iii) terminating the signaling pathway known as Muchero takes in the last paragraph is best
HGF-c-MET described as that of

The correct answer is (A) an idealist setting forth principles


(B) a scholar researching a medical controversy
(A) Only (i)
(B) Only (ii) (C) a speculant determined to business goal
(C) Only (iii) (D) an advocate seeking a compromise position
(D) (i) and (ii)
(E) (ii) and (iii) (E) an observer striving for neutrality

12. As used in the last paragraph, the word 16. What topic does the paragraph following the
“testament” most nearly means passage most likely discuss?

(A) evidence (A) Weaknesses of the research


(B) principle (B) Persons involed in the research
(C) advancement (C) The cancer rate after the research
(D) rule (D) More research findings
(E) pathway (E) Research controversy

13. What is the relationship between the ideas in


paragraphs 2 and 3?

(A) The latter contradicts the former


(B) The latter concludes the former
(C) The latter corrects the former
(D) The latter confirms the former
(E) The latter elaborates the former

- 46 - BTA / Ing / S. Intensif 2023


STATISTICS 1 STATISTICS 2

19. The pie chart above depicts the total amount of


U.S spending on animal-based food production.
On this chart it's important to note that only
$20.7 billion is being spent towards stopping
animal cruelty, while health care is given $314
billion yearly. In this case, which of the
following cannot be identified as animal cruelty
in the food industry?

(A) Many factory-farmed animals have been


genetically mutated to produce higher
quality goods such as beef and eggs.
(B) In the production of foie gras, antibiotics
used on ducks and geese keep them alive in
what should be deathly conditions, also
causing antibiotic-resistant bacteria which is
harmful to humans.
(C) 8.5 billion chickens are killed every year,
17. According to the graph above, we can conclude and 300 million are kept in cages for egg
that the production volume of biodiesel in production.
Indonesia … during COVID-19 pandemic. (D) To mass-produce bacons, female pigs spend
2 - 3 weeks nursing their young as opposed
(A) remained the same to the usual 13 - 17 weeks they would in the
(B) doubled wild.
(C) increased exponentially (E) On a typical farm, crocodiles and alligators
(D) grew slightly are usually skinned alive in the production
(E) decreased severely of leather goods.

18. Which claim about the production volume of 20. From the chart above, it’s plain to see that the
biodiesel in Indonesia is supported by the graph? allocated budget spent for fish production is …..
compared to that for ….. purpose, which are $4.5
(A) A small decline occurred over the course of billion and $37.2 billion respectively.
2017
(B) There is a sharp increase during 2020 (A) minimum – environment
(C) The production was stopped in 2015 (B) minute – environmental
(D) The production was stagnant between 2016 (C) minimal – environment
and 2017 (D) extensive – environmental
(E) A subtle difference was gained between (E) mediocre – environment
2017 and 2018

- 47 - BTA / Ing / S. Intensif 2023


PASSAGE 3

Most people know that eating too much dessert and processed food can contribute to physical health
problems like obesity and type 2 diabetes. Far less attention has been given to the impact of a high-sugar diet on
mental health though numerous studies have shown the deleterious effects a sweet tooth can have on mood, learning,
and quality of life. In addition to inflating waistlines, sugar and other sweeteners may contribute to a number of
mental health problems such as depression and anxiety.
The roller coaster of high blood sugar followed by a crash may accentuate the symptoms of mood
disorders. Research has tied heavy-sugar consumption to an increased risk of depression and worse outcomes in
individuals with schizophrenia. There are a couple of theories explaining the link. Sugar suppresses activities of a
hormone called BDNF that is low in individuals with depression and schizophrenia. Sugar is also at the root of
chronic inflammation, which impacts the immune system, the brain, and other systems in the body and also has been
implicated in depression. Interestingly, countries with high sugar intake also have a high rate of depression.
The Standard American Diet, which is full of sugar and fat, does not necessarily cause anxiety, but it does
appear to worsen anxiety symptoms and impair the body’s ability to cope with stress. Individuals who suffer from
panic attacks, for example, are hyper-alert to signs of impending danger. Sugar can cause blurry vision, difficulty in
thinking, and fatigue, all of which may be interpreted as signs of a panic attack, thereby increasing worry and fear. A
sugar high and subsequent crash can cause shaking and tension, which can make anxiety worse.
Research has established a correlation between sugar intake and anxiety. In a 2008 study, rats that binged
on sugar and then fasted displayed anxiety, and in a 2009 study rats fed sucrose compared to high-antioxidant honey
were more likely to suffer anxiety. While dietary changes alone cannot cure anxiety, they can minimize symptoms,
boost energy, and improve the body’s ability to cope with stress.

21. What is the author’s attitude toward the topic of 24. Based on the passage, anxiety symptoms will get
the passage? worse if ....

(A) Assertive (A) fat dominates the diet


(B) Thoughtful
(B) a diet is full of sweet food
(C) Suggestive
(D) Considerate (C) food consumed contains lots of fat and sugar
(E) Positive
(D) dietary changes fail to replace foods
22. The paragraph following the passage will likely containing sugar
talk about .... (E) processed food is the main daily meal

(A) time to change diets


(B) impacts of dietary changes
(C) how to stop sugar consumption
(D) further studies on dietary changes
(E) examples of diets to cope with stress

23. The passage can be best summarized as follows


....

(A) sugar consumption may decrease the quality


of life and increase risks of physical
illnesses
(B) dietary changes are the alternative to stay
away from high-sugar food
(C) there is a correlation between sugar intake
and mental health
(D) the negative impact of sugar on human
health has been shown empirically
(E) impact of sugar on health received less
attention than dessert and processed food

- 48 - BTA / Ing / S. Intensif 2023


BAHASA INGGRIS
PERSIAPAN UJIAN MASUK PTN (9)
PASSAGE 1

Only recently have researchers at the University of Massachusetts Amherst announced that they have
figured out how to engineer a biofilm that harvests the energy in evaporation and converts it to electricity. This
biofilm, which was announced in Nature Communications, has the potential to revolutionize the world of wearable
electronics, powering everything from personal medical sensors to personal electronics. "This is a very exciting
technology," says Xiaomeng Liu, graduate student in electrical and computer engineering in UMass Amherst's
College of Engineering and the paper's lead author. "It is real green energy, and unlike other so-called 'green-energy'
sources, its production is totally green."
That's because this biofilm -- a thin sheet of bacterial cells about the thickness of a sheet of paper -- is
produced naturally by an engineered version of the bacteria Geobacter sulfurreducens, known to produce electricity
and used previously in "microbial batteries" to power electrical devices. But such batteries require that so-called G.
sulfurreducens is properly cared for and fed a constant diet. By contrast, this new biofilm, which can supply as
much, if not more, energy than a comparably sized battery, works, and works continuously, because it is dead. And
because it's dead, it doesn't need to be fed.
"It's much more efficient," says Derek Lovley, Distinguished Professor of Microbiology at UMass Amherst
and one of the paper's senior authors. "We've simplified the process of generating electricity by radically cutting
back on the amount of processing needed. We sustainably grow the cells in a biofilm, and then use that
agglomeration of cells. This cuts the energy inputs, makes everything simpler and widens the potential applications."
The secret behind this new biofilm is that it makes energy from the moisture on your skin. Though we daily read
stories about solar power, at least 50% of the solar energy reaching the earth goes toward evaporating water. "This is
a huge, untapped source of energy," says Jun Yao, professor of electrical and computer engineering at UMass, and
the paper's other senior author. Since the surface of our skin is constantly moist with sweat, the biofilm can "plug-in"
and convert the energy locked in evaporation into enough energy to power small devices.
"The limiting factor of wearable electronics," says Yao, "has always been the power supply. Batteries run
down and have to be changed or charged. They are also bulky, heavy, and uncomfortable." But a clear, small, thin
flexible biofilm that produces a continuous and steady supply of electricity and which can be worn, like a Band-Aid,
as a patch applied directly to the skin, solves all these problems. What makes this all work is that G. sulfurreducens
grows in colonies that look like thin mats, and each of them connects to its neighbors through a series of natural
nanowires. The team then harvests these mats and uses a laser to engrave small circuits into the films. Once the
films are engraved, they're sandwiched between electrodes and finally sealed in a soft, sticky, breathable polymer
that you can apply directly to your skin. ………………………………………………………………
"Our next step is to increase the size of our films to power more sophisticated skin-wearable electronics,"
says Yao, and Liu points out that one of the goals is to power entire electronic systems in the near future, rather than
single devices. This research was nurtured by the Institute for Applied Life Sciences (IALS) at UMass Amherst,
which combines deep and interdisciplinary expertise from 29 departments to translate fundamental research into
innovations that benefit human health and well-being.

01. A crucial attribute of ‘real’ green-energy in 02. Which statement best expresses a relationship
Xiaomeng Liu’s point of view is its between the ‘battery’ and the modern biofilm
presented in the second paragraph?
(A) size with a thickness of a sheet of paper
(A) Both are equally sustainable, but the former
(B) potential to satisfy the energy needs of the
requires periodical maintenance, the latter
world’s population
doesn’t
(C) capability to gradually limit carbon footprint
(B) Both use no artificial chemicals, but the
the world over
former is lees ‘green’ than the latter
(D) origin from an engineered version of certain
(C) Both are ‘green’, but they differ
microbes
significantly in the amount of energy they
(E) ability to harvest the energy in evaporation could generate
and convert it to electricity

- 49 - BTA / Ing / S. Intensif 2023


(D) Both rely on G. sulfurreducens, the 07. Which of the following sentences can complete
proportions of which are not the same for the missing sentence in paragraph 4 of the
each passage?
(E) Both are inanimate, but the former needs
(A) Although this tiny battery is "plugged in" by
extra care, the latter doesn’t
applying it to your body, it can power small
devices.
(B) Until this tiny battery is "plugged in" by
03. “This is a huge, untapped source of energy”
applying it to your body, it can power small
(paragraph 3).
devices.
What does the word “this” refer to?
(C) Once this tiny battery is "plugged in" by
(A) solar power applying it to your body, it can power small
(B) solar energy devices.
(C) skin surface
(D) Unless this tiny battery is "plugged in" by
(D) new biofilm
applying it to your body, it can power small
(E) moisture on skin
devices.
(E) Before this tiny battery is "plugged in" by
04. The mean purpose of the third paragraph is to applying it to your body, it can power small
devices.
(A) relate the study of conventional green
energy to the study of ‘real’ green energy
(B) speculate on the reason for the researchers’
08. Which paragraph describes the limitations of the
unexpected results
biofilm that end-users might encounter in the
(C) describe specific procedures of the research
future?
(D) identify an important finding of the study of
biofilm
(E) share several issues in collecting and (A) second
analyzing data (B) third
(C) fourth
05. According to the last paragraph, the current (D) fifth
biofilm
(E) none of the above
(A) has produced sufficient energy to power
more sophisticated skin-wearable electronics
(B) will never be deemed as detrimental to the 09. According to the last paragraph, Yao and Liu’s
environment attitude toward the future innovation of the
(C) is energy efficient and reduces the need to experiment could be best described as that of
consume fossil fuel
(D) is good for humans only in the short run (A) avid
(E) cannot power large-scale electronic systems
(B) ambivalent
yet
(C) concerned
(D) derogatory
06. Which choice is close to the meaning of the word
“engrave” used in the fourth paragraph? (E) cynical

(A) seize
(B) etch
(C) curb
(D) kindle
(E) trim

- 50 - BTA / Ing / S. Intensif 2023


PASSAGE 2

Humankind has maintained an enduring fascination with the Moon. It was not until Galileo's time,
however, that scientists really began to study it. Over the course of nearly five centuries, researchers put forward
numerous, much debated theories as to how the Moon was formed. Now, geochemists, cosmochemists, and
petrologists at ETH Zurich shed new light on the Moon's origin story. In a study just published in the journal,
Science Advances, the research team reports findings that show that the Moon inherited the indigenous noble gases
of helium and neon from Earth's mantle. The discovery adds to the already strong constraints on the currently
favoured "Giant Impact" theory that hypothesizes the Moon was formed by a massive collision between Earth and
another celestial body.
During her doctoral research at ETH Zurich, Patrizia Will analysed six samples of lunar meteorites from an
Antarctic collection, obtained from NASA. The meteorites consist of basalt rock that formed when magma welled
up from the interior of the Moon and cooled quickly. They remained covered by additional basalt layers after their
formation, which protected the rock from cosmic rays and, particularly, the solar wind. The cooling process resulted
in the formation of lunar glass particles amongst the other minerals found in magma. Will and the team discovered
that the glass particles retain the chemical fingerprints (isotopic signatures) of the solar gases: helium and neon from
the Moon's interior. Their findings strongly support that the Moon inherited noble gases indigenous to the Earth.
"Finding solar gases, for the first time, in basaltic materials from the Moon that are unrelated to any exposure on the
lunar surface was such an exciting result," says Will.
Without the protection of an atmosphere, asteroids continually pelt the Moon's surface. It likely took a
high-energy impact to eject the meteorites from the middle layers of the lava flow similar to the vast plains known
as the Lunar Mare. Eventually the rock fragments made their way to Earth in the form of meteorites. Many of these
meteorite samples are picked up in the deserts of North Africa or in, in this case, the "cold desert" of Antarctica
where they are easier to spot in the landscape.
In the Noble Gas Laboratory at ETH Zurich resides a state-of-the-art noble gas mass spectrometer named,
"Tom Dooley" -- sung about in the Grateful Dead tune by the same name. The instrument got its name, when earlier
researchers, at one point, suspended the highly sensitive equipment from the ceiling of the lab to avoid interference
from the vibrations of everyday life. Using the Tom Dooley instrument, the research team was able to measure sub-
millimetre glass particles from the meteorites and rule out solar wind as the source of the detected gases. The helium
and neon that they detected were in a much higher abundance than expected. The Tom Dooley is so sensitive that it
is, in fact, the only instrument in the world capable of detecting such minimal concentrations of helium and neon. It
was used to detect these noble gases in the 7 billion year-old grains in the Murchison meteorite -- the oldest known
solid matter to-date.
Knowing where to look inside NASA's vast collection of some 70,000 approved meteorites represents a
major step forward. "I truly believe that there will be a race to study heavy noble gases and isotopes in meteoritic
materials," says ETH Zurich Professor Henner Busemann, one of the world's leading scientists in the field of extra-
terrestrial noble gas geochemistry. He anticipates that soon researchers will be looking for noble gases such as
xenon and krypton which are more challenging to identify. They will also be searching for other volatile elements
such as hydrogen or halogens in the lunar meteorites. Busemann comments, "While such gases are not necessary for
life, it would be interesting to know how some of these noble gases survived the brutal and violent formation of the
moon. Such knowledge might help scientists to create new models that show more generally how such most volatile
elements can survive planet formation, in our solar system and beyond."

10. It can be inferred from paragraph 1 that lunar 11. It is implied from paragraph 2 that the cosmic
science, … numerous formation theories … , has rays and solar wind are inclined to … lunar
always fascinated humans for a long time. meteorites during their formation process.

(A) which – still much debatable (A) add basalt layers to


(B) whose – still much debating (B) level off
(C) which – are still much debated (C) fortify
(D) whose – are still much debated (D) erode
(E) which – are still much debatable (E) reduce

- 51 - BTA / Ing / S. Intensif 2023


12. The following can be inferred about the solar 16. According to the last paragraph, ETH Zurich
gases according to paragraph 2, except that they Professor Henner Busemann is … that the study
are of heavy noble gases and isotopes in meteoritic
materials will soon gain much popularity.
(A) helium and neon
(B) involved in the formation process of the (A) very much amazed
lunar glass particles
(B) strongly convinced
(C) brought by meteorites falling on the Moon’s
surface (C) rather perplexed
(D) commonly known as noble gases among
(D) pleasantly surprised
scientists
(E) present inside the lunar magma (E) really astonished

13. The word “pelt” as used in paragraph 3 should


17. According to the last paragraph, xenon and
mean
krypton are mentioned most likely to
(A) damage
(A) inform that both survived the brutal and
(B) strike
violent formation of the moon
(C) puncture
(D) cover (B) indicate that both should appear before
(E) unthaw hydrogen and halogens do
(C) explain that both are volatile in certain
14. The relationship between Tom Dooley and the
conditions
noble gases is similar to the phenomenon
between (D) highlight that both are not necessary for life

(A) a speedometer and velocity (E) suggest that both will be in high demand on
(B) a measuring cup and a whisk the upcoming analyses
(C) pancreatic cancer and oncologist
(D) a poacher and his game 18. The word “volatile” in the last paragraph most
(E) diabetes and blood sugar nearly means: easily

15. The “scientists” mentioned throughout the (A) versatile


passage deal with the following fields, except
(B) adjustable
(A) geochemistry (C) unstable
(B) cosmochemistry
(C) petrology (D) vaporable
(D) geophysics (E) convertible
(E) astrology

- 52 - BTA / Ing / S. Intensif 2023


INFOGRAPHIC ANALYSIS 19. Data presented in the figure most directly
support which idea?

(A) 57% of adults who get news on digital


device use Facebook or Twitter
(B) The percentage of people using social media
news is double that of newspapers
(C) In terms of top news sources, people still
prefer reading newspapers to listening to
radio
(D) The patterns observed in the experiment
suggest that people tend to watch motion
picture than reading passages
(E) The positive bias towards social media users
may lead to unpopularity of other news
sources

20. In terms of social media as an online news


source, which statement is best supported by the
information presented in the infographic?

(A) Most people still obtain news from


Facebook
(B) YouTube has long been a pioneer in
supporting the future of journalism
(C) Google+ is more popular than Twitter in
news broadcasting
(D) People have been using Twitter as often as
Facebook to obtain news
(E) No other social media other than Facebook,
Twitter, YouTube and Google+ are reported
to serve the news source for people in the
digital age

PASSAGE 3

A motive is something that causes a person to act. A successful athlete, for example, is said to be “highly
motivated”. A student who avoids doing homework is said to be “unmotivated”. Generally, motivation is defined as
the internal process that energizes, directs, and sustains behavior. It is the personal “force” that causes you or me to
act in a particular way. Thus, job rotation may increase your enthusiasm for your work so that you devote more
energy to it. Job rotation, however, might not have the same effect on me.
Morale is sometimes confused with motivation, but it is different. Morale refers to an employee’s attitude
or feelings about the job, his or her superiors, and the firm itself. High morale results mainly from the satisfaction of
needs met by the job or its rewards. One need that might be satisfied, for instance, is the desire to be recognized for
a job well done. Another need that can be satisfied by work is the longing for financial security. Whereas high
morale can fuel motivation, low morale can do the opposite. It can lead to shoddy work, absenteeism, and high
turnover rates as employees leave to seek more satisfying jobs with other firms.

- 53 - BTA / Ing / S. Intensif 2023


Beginning in the late nineteenth century, both motivation and morale became the subjects of research as
employers sought to increase productivity. Probably the most famous study of motivation and morale were
performed by Frederic Taylor, considered the father of scientific management. After studying worker behavior at
two different steel companies, Taylor claimed that most people work only to earn money. He therefore reasoned that
pay should be tied directly to output. Taylor’s theory made an impression on employers and gave rise to the price-
rate system, under which employees were paid a certain amount for each unit produced. Today, however, Taylor’s
theory is considered overly simplistic. It is generally acknowledged that people work for a variety of reasons than
pay.

21. What is the main idea of the text? 24. What can be inferred from the text?

(A) Frederic Taylor conducted a famous (A) Company productivity became the subjects
research on motivation and morale. of research as employers sought to improve
(B) Motivation is defined as the internal process motivation.
energizing employees’ spirit of satisfaction.
(B) High turnover must be the result of low
(C) Motivation and morale have played a
morale and motivation.
different role in worker productivity.
(D) The price-rate system was made to motivate (C) People have different needs to be satisfied in
employees’ productivity. relation to their job.
(E) Motivation an morale are different in
meaning, but they are important to job (D) Under the price-rate system, employees
satisfaction. earned a certain amount for the units sold.
(E) Regular change increases employees’
22. The writer’s primary purpose is to .... enthusiasm and devotion to their job.

(A) persuade
(B) inform
(C) argue 25. This text would probably be assigned reading in
(D) analyze which of the following courses?
(E) criticize
(A) Statistics
23. The word “shoddy” in paragraph 2 is closest in (B) Literature
meaning to ....
(C) Management
(A) meticulous (D) Accounting
(B) inferior
(C) careful (E) Psychology
(D) helpless
(E) superior

- 54 - BTA / Ing / S. Intensif 2023


BAHASA INGGRIS
PERSIAPAN UJIAN MASUK PTN (10)

PASSAGE 1

Researchers at the University of Toronto, Johns Hopkins University and Vanderbilt University have
discovered that certain cells move surprisingly faster in thicker fluid -- think honey as opposed to water, or mucus as
opposed to blood -- because their ruffled edges sense the viscosity of their environment and adapt to increase their
speed. Their combined results in cancer and fibroblast cells -- the type that often creates scars in tissues -- suggest
that the viscosity of a cell's surrounding environment is an important contributor to disease, and may help explain
tumour progression, scarring in mucus-filled lungs affected by cystic fibrosis, and the wound-healing process.
The study, "Membrane ruffling is a mechanosensor of extracellular fluid viscosity," published today in
Nature Physics, sheds new light on cell environments, an under-explored area of research. "This link between cell
viscosity and attachment has never been demonstrated before," says Sergey Plotnikov, assistant professor in the
Department of Cell and Systems Biology in the Faculty of Arts & Science at the University of Toronto and a co-
corresponding author of the study. "We found that the thicker the surrounding environment, the stronger the cells
adhere to the substrate and the faster they move -- much like walking on an icy surface with shoes that have spikes,
versus shoes with no grip at all."
Understanding why cells behave in this surprising way is important because cancer tumours create a
viscous environment, which means spreading cells can move into tumours faster than non-cancerous tissues. Since
the researchers observed that cancer cells speed up in a thickened environment, they concluded that the development
of ruffled edges in cancer cells may contribute to cancer spreading to other areas of the body. Targeting the
spreading response in fibroblasts, on the other hand, may reduce tissue damage in the mucus-filled lungs affected by
cystic fibrosis. Because ruffled fibroblasts move quickly, they are the first type of cells to move through the mucus
to the wound, contributing to scarring rather than healing. These results also may imply that by changing the
viscosity of that mucus, one can control the cell movement.
"By showing how cells respond to what's around them, and by describing the physical properties of this
area, we can learn what affects their behaviour and eventually how to influence it," says Ernest Iu, PhD student in
the Department of Cell and Systems Biology in the Faculty of Arts & Science at the University of Toronto and study
co-author. Plotnikov adds, "For example, perhaps if you put a liquid as thick as honey into a wound, the cells will
move deeper and faster into it, thereby healing it more effectively."
Plotnikov and Iu used technical microscopic advancement to measure the traction that cells exert to move,
and changes in structural molecules inside the cells. They compared cancer and fibroblast cells, which have ruffled
edges, to cells with smooth edges. They determined that ruffled cell edges sense the thickened environment,
triggering a response that allows the cell to pull through the resistance -- the ruffles flatten down, spread out and
latch on to the surrounding surface.
The experiment originated at Johns Hopkins, where Yun Chen, assistant professor in the Department of
Mechanical Engineering and lead author of the study, and Matthew Pittman, PhD student and first author, were first
examining the movement of cancer cells. Pittman created a viscous, mucus-like polymer solution, deposited it on
different cell types, and saw that cancer cells moved faster than non-cancerous cells when migrating through the
thick liquid. To further probe this behaviour, Chen collaborated with U of T's Plotnikov, who specializes in the push
and pull of cell movement.
Plotnikov was amazed at the change in speed going into thick, mucus-like liquid. "Normally, we're looking
at slow, subtle changes under the microscope, but we could see the cells moving twice as fast in real time, and
spreading to double their original size," he says. Typically, cell movement depends on myosin proteins, which help
muscles contract. Plotnikov and Iu reasoned that stopping myosin would prevent cells from spreading, however
were surprised when evidence showed the cells still sped up despite this action. They instead found that columns of
the actin protein inside the cell, which contributes to muscle contraction, became more stable in response to the thick
liquid, further pushing out the edge of the cell.
………………., which may open the door to new treatments for people affected by cancer and cystic
fibrosis. Funding for the research was provided by the Natural Sciences and Engineering Research Council of
Canada, Canadian Institutes of Health Research, Canadian Network for Research and Innovation in Machining
Technology, Ontario Graduate Scholarship, U.S. Department of Health & Human Services and United States
Department of Defense.

- 55 - BTA / Ing / S. Intensif 2023


01. The following statements best support the idea in 06. According to paragraph 7, stopping myosin will
paragraph 1, except… prevent cells from spreading … it takes place in
a viscous environment.
(A) Fibroblast cells often create scars in human
tissues (A) although
(B) The viscosity of a cell's surrounding (B) unless
environment can explain wound-healing (C) since
process (D) when
(C) Lungs can also experience scarring due to (E) despite
mucus content in them
(D) All cells tend to move faster in a viscous
surrounding environment 07. Which of the following would best complete the
(E) Ruffled edges of cells help increase their missing part in the last paragraph?
speed
(A) However, the team are still clueless of how
02. In the context of cell’s surrounding viscosity, the to control the movement of ruffled cells
author most likely includes Sergey Plotnikov’s through thickened environments
comments (paragraph 2) in order to … the notion (B) Hence, modern technology should be
mentioned in the first paragraph. applied in search for a better method to
control the movement of ruffled cells
(A) rebut through thickened environments
(B) refuse (C) Additionally, teams are now investigating
(C) refute how to slow the movement of ruffled cells
(D) confirm through thickened environments
(E) consider (D) Furthermore, the researchers deserve an
accolade for their amazing finding
03. The words “cell movement” in the last part of (E) In addition, large amount of funds are
paragraph 3 would apply to … needed for this research

(A) cells responsible to create spongy nature of 08. In general, the passage characterizes the finding
the lungs about the behaviour of cell’s ruffled edges in a
(B) fibroblasts viscous surrounding environment as promising

(C) merely cells moving faster into cancerous
tissues (A) although further experiments seem
(D) all kinds of human cells wherever they move imperative in order to achieve conclusive
(E) cancer cells finding
(B) because its efficacy is scientifically proven
04. The phrase “technical microscopic to control tumours
advancement” (paragraph 5) should be … (C) since people are now well educated and
aware on how to reduce viscosity in their
(A) No change body
(B) microscopic advancement techniques (D) even though general public still deems it as
(C) microscope technical advance controversial
(D) advanced microscopy techniques (E) because the experiments were conducted by
(E) advanced technical microscope experts of top universities

05. As used in the sixth paragraph, “solution” most 09. Which branch of biology best deals with the
nearly means … passage?

(A) waste material (A) genetics


(B) substitute (B) taxonomy
(C) industrial solvent (C) cytology
(D) fine extraction (D) ecology
(E) liquid mixture (E) morphology

- 56 - BTA / Ing / S. Intensif 2023


PASSAGE 2

The following passage is excerpted from a memoir by an American singer, songwriter, record producer, and
actress: Mariah Carey. It was released on September 29, 2020 and published by Andy Cohen Books, an imprint of
Henry Holt.

“You’ve always been the light of my life.” My mother told me this over and over when I was a child. I
wanted to be her light. I wanted to make her proud. I respected her as a singer and a working mother. I loved her
deeply, and, like most kids, I wanted her to be a safe place for me. Above all, I desperately wanted to believe her.
But ours is a story of betrayal and beauty. Of love and abandonment. Of sacrifice and survival. I’ve
emancipated myself from bondage several times, but there is a cloud of sadness that I suspect will always hang over
me, not simply because of my mother but because of our complicated journey together. It has caused me so much
pain and confusion. Time has shown me there is no benefit in trying to protect people who never tried to protect me.
Time and motherhood have finally given me the courage to honestly face who my mother has been to me.
For me, this is the steepest cliff edge. If I can make it to the other side of this truth, I know there is relief of
epic proportions awaiting me. Those people who have hurt me, over and over, whom I have escaped or walled off,
are deeply significant in my story, but they are not central to my existence.
Removing myself from toxic people I love has been excruciatingly painful, but once I found the courage
(with prayer and professional help, of course), I simply let go and let God. (I’ll add, though, that there’s a huge
difference between simple and easy. It ain’t easy, baby.) Yet, there is no “artful” way of letting go of my mother,
and our relationship is anything but simple. Like many aspects of my life, my journey with my mother has been full
of contradictions and competing realities. It’s never been only black and white—it’s been a whole rainbow of
emotions.
Our relationship is a prickly rope of pride, pain, shame, gratitude, jealousy, admiration, and
disappointment. A complicated love tethers my heart to my mother’s. When I became a mother to Roc and Roe, my
heart grew two times over; as my capacity for pure love expanded, the ability to tow heavy pain from my past
diminished. Healthy, powerful love did that for me: it illuminated the dark spots and unearthed buried hurt. The
new, clear light that emanates from my children’s love now rushes through every artery, every cell, every dark nook
and cranny of my being.
Even after all this time, a part of me fantasizes that one of these days my mother will transform into one of
the caring mothers I saw on TV as a child, like Carol Brady or Clair Huxtable; that she will suddenly ask me,
“Honey, how, was your day?” before she gives me a report on her dog or her bird, or asks me to pay for something
or do something—that she will have genuine, sustained interest in me and what I’m doing or feeling. That one day
she will know me. That one day my mother will understand me.
To a certain extent, I know how my mother became who she is. Her mother certainly didn’t understand her.
And her father never had a chance to know her; he died while her mother was pregnant with her. She was one of
three children raised by a widowed Irish Catholic woman. My mother was known as the “dark one” because her hair
wasn’t blond and her eyes were a mix of brown and green, not pure blue like her brother’s and sister’s. Blue eyes
were a symbol of the purity of whiteness, and …………………………………...
My mother grew up in the 1940s and 1950s in Springfield, Illinois. It was the capital city at the center of a
state at the center of the country. But Springfield was also a center of insidious institutional racism. In 1908, a white
woman was allegedly raped by a Black man (the same accusation leveled against my father and countless other
innocent Black men), which ignited a three-day riot by white citizens in which two Black men were lynched and
four white men were shot to death by Black businessmen protecting their property. In the 1920s, when my mother’s
mother was coming of age, the Ku Klux Klan had a strong presence in the city and the city government, holding
several key positions and setting the moral compass for the community. Springfield was a city that openly cloaked in
hate.

- 57 - BTA / Ing / S. Intensif 2023


10. The general organization of the passage is best 14. Which of the following would best complete the
described as which of the following? missing part in the seventh paragraph?

(A) A narration that reveals how the author’s (A) in being of 100 percent “pure” Irish descent
mother has been a control freak since long was central to her mother’s entire identity.
ago
(B) that being of 100 percent “pure” Irish
(B) An accumulation of old-time actions written descent was central to her mother’s entire
in a memoir dedicated especially to the identity.
author’s mother
(C) due mainly to being of 100 percent “pure”
(C) A progression from irritability and bitterness Irish descent was central to her mother’s
into happiness and self confidence felt by entire identity.
the author towards her mother
(D) despite being of 100 percent “pure” Irish
(D) A personal testimony where the author descent was central to her mother’s entire
processes her troubled relationship with her identity.
mother
(E) being of 100 percent “pure” Irish descent
(E) A series of past events where the author’s was central to her mother’s entire identity.
feelings are oftenly invalidated by her
mother
15. The author’s presentation of her mother in
11. The last sentence of the first paragraph implies paragraphs 7 and 8 is most like that of a …
that the author felt …
(A) writer commenting on another fellow writer
(A) oblivious to the true meaning of the phrase
(B) mechanical engineer explaining how to fix a
“the light of my life” her mother had
machinary breakdown
expressed
(C) social critic delivering an exhortation
(B) irresolute of what her mother had asserted
(D) physicist explaining the reason planets never
(C) suspicious that her relationship with her
collide with each other
mother would turn into an ignominious fight
(E) columnist recounting a vexatious personal
(D) insecure every time her mother was around
experience
her
(E) overjoyed as her mother would love her
unconditionally
16. The last part of the passage: Springfield was a
city that openly cloaked in hate.
12. Which choice can replace the word The underlined part should be …
“emancipated” used in the second paragraph?
(A) No change
(A) disclosed
(B) which openly cloaked
(B) dismayed
(C) discharged (C) that is openly cloaked
(D) disdained
(D) openly cloaked
(E) dissected
(E) cloaking openly
13. In context, the phrase “nook and cranny”
(paragraph 5) most nearly means …

(A) memories
(B) thoughts
(C) metabolism
(D) feelings
(E) details

- 58 - BTA / Ing / S. Intensif 2023


INFOGRAPHIC ANALYSIS 17. Which choice most closely captures the meaning
of the word Joes?

(A) Gullibel people


(B) Hackers
(C) Ordinary citizens
(D) Spammers
(E) Cybercriminals

18. Which choice most closely captures the meaning


of the word spams?

(A) Emails
(B) Unwanted emails
(C) Bank emails
(D) Virtual account
(E) Newsletters

19. Phishing and spam are recently used as a primary


way to steal credentials from unwary users as
well as their money due to the following, except

(A) Users data are kept in emails, which are


prone to hacking
(B) Users share their address book on social
networks
(C) Users don’t examine their bank emails
(D) Users use public WIFI hotspots
(E) Users consider social sites as a treasure
trove for them to harvest

20. According to the infographic above, which


statement is false?

(A) 36% Joes basically open a spam email


because of a catchy subject line
(B) Downloading a file undoubtedly makes it
easier to install malicious code, helping
spammers spam even more
(C) 37% Joes have occasionally lost important
email in the Spam inbox
(D) 19% Joes hardly download a file from spam
email
(E) 14% Joes apparently consider buying from a
Spammer

- 59 - BTA / Ing / S. Intensif 2023


PASSAGE 3

Many modern educational experts claim that teaching facts and academic skills is less important than
achieving other social objectives. For some liberals, the schools must first change attitudes or provide nurturing in
place of failed families or help establish equality and social justice. For some conservatives, the schools must first
prepare kids for the workplace by molding them into supple corporate citizens, while others want the focus to be on
family values, a competitive spirit, or other social or behavioral objectives. But the idea of simply educating kids
seems to have taken a backseat to most educational experts and administrators. They miss the point that kids with
real academic skills, especially skills in reading, writing, and mathematics, are more likely to overcome social
barriers, more likely to have genuine self-esteem, and most likely to be genuinely prepared for the challenges of life
and the workplace. By emphasizing so many things besides a genuine, classical education, the educational
establishment tends to sell our kids short and bring about many of the problems they claim to be solving.
Consider the case of Wesley Elementary School in Houston. According to Richard Nadler in the article,
“Failing Grade,” Wesley has all the demographic markers of a school bound for failure. Over 80% of the students
qualify for subsidized lunches, and nearly all are minorities (92% black, 7% Hispanic). Yet it ranks among the best
schools of Houston, with first-graders placing at the 82nd percentile level in reading tests which is 50 points higher
than the expected level for similar at-risk schools.
What has made Wesley so successful? The answer is classical education in the form of Direct Instruction
curriculum designed by Siegfried Engelmann, an example of the much-ridiculed “sage-on-the-stage” approach. This
Direct Instruction system boosts reading, writing, and math scores by 30 to 40 percentile points in at risk schools.
Sadly, Engelmann, like others who successfully challenge popular fads in educational reform, has been rejecte d by
much of the educational establishment. His success is an embarrassed to them.

21. Which of the following best reflects the 23. From the second paragraph, it can be
author’s opinion about schools? inferred that ....

(A) Teaching social skills is more important than (A) the students at Wesley are mostly colored.
academic skills.
(B) Wesley is a successful prestigious school
(B) Teaching academic skills is more important
than social skills. (C) the colored students usually perform better
(C) Schools must be able to change the attitude
(D) Wesley curriculum is adopted by other
of the students.
schools
(D) Teaching social skills should use
conventional methods. (E) the students at Wesley are from the haves
(E) Teaching academic skills is somehow
contemporary.

22. The second paragraph is related to the first, 24. In writing the passage, the writer’s tone
in which the second presents could be best described as ....
(A) an elaboration of purposes of an effective
school. (A) persuasive
(B) a discussion on the requirements for the (B) descriptive
good school.
(C) a real example rather than an opinion of (C) informative
good schooling. (D) evaluative
(D) an illustration to support the opinions on
school subjects. (E) conservative
(E) evidence in favor of the value of social
objectives.

- 60 - BTA / Ing / S. Intensif 2023


BAHASA INGGRIS
PERSIAPAN UJIAN MASUK PTN (11)
Text 1

Banana peels can be used to purify drinking water contaminated with toxic heavy metals such as copper
and lead, according to a study. Researchers from the Bioscience Institute at Botucatu, Brazil, said that the skins can
outperform even conventional purifiers such as aluminium oxide, cellulose and silica. These have potentially toxic
side effects and are expensive. The team’s method follows previous work that showed that plant parts, such as apple
and sugar cane wastes, coconut fibres and peanut shells, can remove toxins from water. These natural materials
contain chemicals that have an affinity for metals.
In the process, the researchers dried the peels in the sun for a week, ground them and added them to river
water containing known concentrations of copper and lead. They found that the peels absorbed 97 per cent of the
metals after just one hour. The peels were tested in the lab and worked perfectly. Eventually their efficiency reduces,
at which point the metals should be removed from the skins so that they can be disposed of safely. In addition, they
said that although the peels were tested only on copper and lead, the material could also work on cadmium, nickel
and zinc. But they warned that this sort of filter is better suited to industrial purposes and cannot be used for water
purification at home as the extraction capacity of banana skins depends on the particle size of the heavy metals – and
this is difficult to measure. Dimitris Kalderis, from the Technical University of Crete, Greece, said: “The results are
very promising, and the banana peel process has proven to be a cost-effective and quick alternative to conventional
methods”. The knowledge is there, what we need right now is innovation and construction.”

01. What is the author’s main purpose writing in the 03. What does the word “these” in ‘These have
passage? potentially toxic ...’ (paragraph 1 line 3) refer to?

(A) To demonstrate the existence of toxic (A) Toxic effects


elements in the water. (B) Banana peels
(C) Heavy metals
(B) To explain that banana skins can remove
(D) Natural materials
toxins from water.
(E) Conventional purifiers
(C) To suggest that banana peel function can
further be examined.
04. All of the following have also been verified for
(D) To classify the types of plant parts found
their functions to purify water except ....
useful as water purifier.
(E) To describe the reason for the need to use (A) peanut sheels
banana skins in daily life. (B) apple wastes
(C) coconut fibres
(D) cocoa wastes
(E) sugar cane wastes
02. Paragraph 1 and 2 are related in that both ....

(A) demonstrate chemical elements of metals in


purifying water 05. The author believes that banana peels as a water
(B) discuss general and detailed information on purifier ....
banana content
(A) should be disposed of after ne hour use
(C) illustrate the need to use banana filters for (B) still have toxic side effects and expensive
industrial purposes (C) are suggested to use for industrial purposes
(D) highlight the advantages of banana skins for (D) can only be used for water purification at
metal absorption home
(E) should not be tested for other chemical
(E) present a cause effect relationship between elements
banana and metals

- 61 - BTA / Ing / S. Intensif 2023


Text 2

When Karl Kim immigrated to the United States from Korea as a teenager ten years ago, he had a hard time
learning English. Now he speaks it fluently, and recently he had a unique opportunity to see how our brains adapt to
a second language. Kim is a graduate student in the lab of Joy Hirsch, a neuroscientist at the Memorial Sloan-
Kettering Cancer Center in New York. He and Hirsch have recently found evidence that children and adults don’t
use the same parts of the brain when learning a second language.
The researchers used an instrument called a functional Magnetic Resonance Imager to study the brains of
two groups of bilingual people. One group consisted of those who had learned a second language as children. The
other consisted of people who, like Kim, learn their second language later in life. When placed inside the MRI
scanner, which allowed Kim and Hirsch to see which parts of the brain were getting more blood and were thus more
active, people from both groups were asked to think about what they had done the day before, first in one language
and then the other. They couldn’t speak out loud, because any movement would disrupt the scanning. Kim and
Hirsch look specifically at two language centers in the brain: Broca’s area, in the left frontal part, which is believed
to manage speech production, and Wernicke’s area, in the rear of the brain, thought to process the meaning of
language. Both groups of people, Kim and Hirsch found, used the same part of Wernicke’s area no matter what
language they were speaking. But their use of Broca’s area differed.
People who learned a second language as children used the same region in Broca’s area for both languages.
But those who learned a second language later in life made use of a distinct region in Broca’s area for their second
language - the one activated for their native tongue. How does Hirsch explain this difference? “When language is
being hard-wired during development,” says Hirsch, “the brain may intertwine sound and structures from all
languages into the same area. “ But once that wiring is complete, the management of a new language, with new
sounds and structures, must be taken over by a different part of the brain.
A second possibility is simply that we may acquire language differently as children than we do as adults. “If
you watch mothers or family members teaching an infant to speak,” says Hirsch, “it’s very tactile, it’s very auditory,
and it’s very visual. There are a lot of different inputs. And that’s very different from sitting in a classroom.”

06. The most appropriate title of the text is .... 09. It can be inferred from the text that ....

(A) Second Language Learning (A) Everybody uses the same parts of the brain
(B) Language Experiment when learning a new language.
(C) The Bilingual Brain
(B) Older learners are less likely to be
(D) Language Learning Process
successful in learning a new language than
(E) The Brain’s Function
children.
07. According to the text, Broca’s area .... (C) In their experiment, Kim and Hirsch found
having more blood in the brain will help
(A) deals with the processing of language mastering new language
meaning
(B) activates ones’ native tongue in language (D) Speech production is produced when the
learning brain intertwines sound and structures into
(C) handles speech production in the same area
communication (E) Infant can speak instantly when taught by
(D) processes meaning in language learning its parents who are language experts.
(E) manages brain to get blood to be more
active
10. This text would probably be assigned reading in
08. The word “tactile” in the last paragraph is closest which of the following courses?
in meaning to ....
(A) Biology
(A) concrete (B) Literature
(B) elusive (C) Medicine
(C) vague (D) Neurolinguistics
(D) subtle (E) Sociology
(E) delicate

- 62 - BTA / Ing / S. Intensif 2023


Text 3

Passage A

Hot yoga refers to yoga practiced in a heated environmet, with the room temperature generally reaching 90 to 105
degrees. The theory behind it is that hot yoga helps the body to sweat out toxins while allowing the practitioner to
safely achieve deeper proses. While the practice can offer health benefits and a sense of well-being, people
practicing hot yoga, especially beginners, should take certain precautions, according to Diana Zotos, a certified yoga
instructor and physical therapist in the Rehabilitation Department at Hospital for Special Surgery in Manhatta.
“Yoga of any type is physically challenging, and the heated environment of hot yoga makes the practice especially
demanding,” Zotos says. “The heat makes people feel as if they can stretch deeper into poses and can give them a
false sense of flexibility. This can lead to muscle strains or damage to the joint, including ligaments and cartilage.”

Passage B

Rose Parkes, a British Wheel of Yoga Teacher, is assessing the role of yoga in prisons as part of her PhD at the
University of Leicester Department of Criminology. In her research, she discusses the way in which spiritual
activities can empower and motivate prisoners to survive their imprisonment. Rose is investigating whether yoga
enables individuals to adjust to the prison environment and post-prison life. She believes that prisoners can benefit
from yoga because it is a practice which helps to foster understanding, self-acceptance, peace and wellbeing.
Working as a part-time Probation Officer, Rose witnessed the effectiveness of the technique at forming positive
relationships with other offenders, prompting the study to ascertain whether yoga can help people cope with
incarceration. She added: “Prisons are highly stressful environments and yoga may offer prisoners a much needed
physical and mental release of the tension of prison life, paradoxically turning prison cells into places of retreat,
where prisoners can develop self-discipline and concentration skills.”

11. Which of the following themes is mainly 14. Which of the following statements is the best
discussed in both passages? summary of both passages?

(A) Hot yoga. (A) Yoga has numerous physical and


(B) Yoga in prisons. psychological benefits.
(C) Yoga certificates. (B) Precautions are necessary for yoga
(D) The effect of Yoga. beginners and prisoners.
(E) The dangers of Yoga. (C) Hot yoga is highly recommended for
prisoners’ post prison-life.
12. Which of the following statements are true (D) Yoga instructors need to socialize the power
according to both passages? of yoga to society.
(E) People from various backgrounds can
(A) Yoga helps people in various ways. practice yoga of all types.
(B) Practicing yoga might be quite tiring.
(C) Yoga poses can lead to muscle injury. 15. It can be predicted that if yoga is ....
(D) Prisoners enjoy practicing yoga regularly.
(E) Yoga instructors mu7st warn people to be (A) practically simple, it will be attractive to
careful. everyone
(B) allowed, levels of violence in prisons can
13. Unlike Passage B, Passage A .... fully decrease
(C) practiced properly, people will live healthily
(A) contradicts the benefits of hot yoga and peacefully
(B) reveals the impacts of practicing yoga (D) well socialized, it will become popular
(C) shows the danger of yoga for beginners among senior citizens
(D) illustrates yoga as a gentle form of exercise (E) provided with a certified instructor, it will be
(E) explains the need for a certified yoga safe for prisoners.
instructor

- 63 - BTA / Ing / S. Intensif 2023


Text 4

Although photography was first made public in 1839, the theory behind the principles of the medium begins
with Aristotle’s description of how light waves behave when projected through a small aperture. This is
fundamentally the description of how a lens or camera’s aperture operates when it projects an image into the film at
the back of a camera. In the middle ages, Alhazen and Francis Bacon extended the principle to included a large,
darkened room with a small opening in one wall. In the 15 th to 18th centuries this camera obscura, as it came to be
called, was reduced in size and made convenient for artist to use in tracing scenic design and architectural
perspective.
The chemical principles basic to photography were also described well before photography was “invented”.
Johann Schulze, in 1727, demonstrated that silver salts turned dark when exposed to light. Carl W. Scheele, in 1777,
showed that ammonia retarted the effects of light, and he indicated a possible way of stabilizing the photochemical
process. By the end of the 18th century, the necessary equipment (the camera obscura) were available at least to
produce semipermanent photographic images.
The artistic style and aesthetics of Renaissance Europe placed a high value on a naturalistic rendering of nature
and thus legitimatized the use of machines like the camera obscura by artist. By the mid-18th century, a public
demand had made itself for realistic portraits, which was partially satisfied by other machines for recording human
likenesses. Beginning in the 1790’s, Jacques Charles conducted experiment in the automatic, if permanent,
recording of portrait silhouettes on photosensitive paper. Two imperatives –the need or perspectively accurate
landscape and architectural scenes and for objectively truthful portraits– created a climate for certain types and
styles of picture that, after 1839, would be achieved easily by photography.

16. Which is most likely the topic of the paragraph (C) Before the mid 18th century the
following this text? photochemical processes were unstable.
(D) The upper-class were the common portrait
(A) Portraits of inanimate object. object in the Renaissance era.
(B) Process of publishing portraits. (E) It look some decades before society was
(C) Photographer’s status in society. aware of the art of photography.
(D) More recent photography application.
(E) Photography as an exclusive form of art.
19. How is the information in the text presented?
17. What is the main idea of paragraph 1?
(A) Developments of photography are
(A) A lens projected images accurately onto the explained.
camera film. (B) Characteristics of photographers are
(B) Aristotle discovered the wonder of light outlined.
waves. (C) Types of photography objects are
(C) Alhazen and Francis Bacon were the dark contrasted.
room inventors. (D) The advantages of photography are
(D) Camera obscura used to be enormous described.
before it was reduced in size. (E) The styles of photographers are compared.
(E) The initial idea of photography came up
long before it was publicized.
20. The word ‘he’ in “…and he indicated a possible
18. Which of the following can be inferred from the way of stabillizing ...” (line 10) refers to ....
text?
(A) Aristotle
(A) Camera sizes became small due to the (B) Francis Bacon
fashion at that time. (C) Johann Schulze
(B) A camera was the only tool to record (D) Carl W. Scheele
human likeness in the mid 18th century. (E) Jacques Charles

- 64 - BTA / Ing / S. Intensif 2023


Text 5

Text A

The tropical rain forest is a forest of tall trees in a region of year-round warmth. An average of 50 to 260
inches (125 to 660 cm) of rain falls yearly.

Rainforests now cover less than 6% of Earth’s land surface. Scientists estimate that more than half of all the
world’s plant and animal species live in tropical rain forests. Tropical rainforests produce 40% of Earth’s oxygen.

A tropical rain forest has more kinds of trees than any other area in the world. Scientists have counted about
100 to 300 species in one 2 12 -acre (1-hectare) area in South America. Seventy percent of the plants in the rainforest
are trees.

About 14 of all the medicines we use come from rain forest plants. Curare comes from a tropical vine, and is
used as an anesthetic and to relax muscles during surgery. Quinine, from the cinchona tree, is used to treat malaria.
A person with lymphocytic leukemia has a 99% chance that the disease will go into decrease because of the rosy
periwinkle. More than 1,400 varieties of tropical plants are thought to be potential cures for cancer.

Each of the three largest rainforests––the American, the African, and the Asian––has a different group of
animal and plant species. Each rain forest has many species of monkeys, all of which differ from the species of the
other two rain forests. In addition, different areas of the same rain forest may have different species. Many kinds of
trees that grow in the mountains of the Amazon rain forest do not grow in the lowlands of that same forest.

Text B

Though dry forests occur in climates that are warm year-round, and may receive several hundred centimeters
of rain per year, they deal with long dry seasons which last several months and vary with geographic location. These
seasonal droughts have great impact on all living things in the forest.

Deciduous trees predominate these forests, and during the drought a leafless period occurs, which varies with
species type. The newly bare trees open up the canopy layer, enabling sunlight to reach ground level and facilitate
the growth of thick underbrush. Though less biologically diverse than rain forests, tropical dry forests are still home
to a wide variety of wildlife including monkeys, large cats, parrots, various rodents, and ground dwelling birds.
Many of these species display extraordinary adaptations to the difficult climate.

The most diverse dry forests in the world occur in southern Mexico and in the Bolivian lowlands. The dry
forests of the Pacific Coast of northwestern South America support a wealth of unique species due to their isolation.
The dry forests of central India and Indochina are notable for their diverse large vertebrate faunas. Dry forests of
Madagascar and New Caledonia are also highly distinctive (pronounced endemism and a large number of relictual
taxa) for a wide range of taxa and at higher taxonomic levels.

Species tend to have wider ranges than moist forest species, although in some regions many species do display
highly restricted ranges; most dry forest species are restricted to tropical dry forests, particularly in plants; beta
diversity and alpha diversity high but typically lower than adjacent moist forests. However, due to overexploitation
for forest resources, many important medicinal plants are disappearing from the tropical dry forest regions.

- 65 - BTA / Ing / S. Intensif 2023


21. The theme that best represents the two texts 24. Based on both texts it can be best speculated that
above would be .... ....

(A) flora and fauna in different forests in wet (A) a rainfall rate of a forest can better predict
and dry lands population density of the forest
(B) medical prospects of the flora from rain and
(B) less rainfalls in an area cause a less chance
dry forests
for the area to be a forest
(C) tropical rain forests as a source of herbal
medications (C) for forests to act more as a source of
(D) biodiversity richness of rain and dry forests medicines, their flora should be varied
(E) differences of fauna types in rain and dry
(D) the higher the rainfall of a dry forest, the
forests
more unique species the forest has
(E) the denser the flora population of a forest
22. The following statements reflect opinions in both is, the richer in herbs the forest is
texts, EXCEPT ....

(A) about half of plant and animal species live


in rain forests 25. Both texts differ in their focus, in that Text A
(B) most trees in dry forests are bare during the deals with ....
rainy season
(C) rain forests have less animal species than (A) both flora and fauna; Text B fauna only
dry forests (B) medical potentials; Text B species types
(D) plants from both forests can be used as
medicines (C) rain forest species; Text B dry forest
(E) rain and dry forests grow in warm regions species
(D) areas of rain forests; Text B forest
population
23. The statement that best extracts the gist of the
two texts would be .... (E) herb-based medications; Text B forest types

(A) forests can function well as a source of


herbal medications
(B) planting more trees in forests is an
assurance for biodiversity
(C) flora and fauna contribute to the shaping of
forest types
(D) biodiversity can be most likely found in
tropical forests
(E) dry forests are only found regions other
than wet areas

- 66 - BTA / Ing / S. Intensif 2023


BAHASA INGGRIS
PERSIAPAN UJIAN MASUK PTN (12)
PASSAGE 1
When is a thumb not a thumb? When it's an elongated wrist bone of the giant panda used to grasp bamboo.
Through its long evolutionary history, the panda's hand has never developed a truly opposable thumb and instead
evolved a thumb-like digit from a wrist bone, the radial sesamoid. This unique adaptation helps these bears subsist
entirely on bamboo despite being bears (members of the order Carnivora, or meat-eaters). In a new paper published
in Scientific Reports, the Natural History Museum of Los Angeles County's Curator of Vertebrate Paleontology
Xiaoming Wang and colleagues report on the discovery of the earliest bamboo-eating ancestral panda to have this
"thumb." Surprisingly, it's longer than its modern descendants.
While the celebrated false thumb in living giant pandas (Ailuropoda melanoleuca) has been known for
more than 100 years, ………………………………………………………………. Uncovered at the Shuitangba site
in the City of Zhaotong, Yunnan Province in south China and dating back 6-7 million years ago, a fossil false thumb
from an ancestral giant panda, Ailurarctos, gives scientists a first look at the early use of this extra (sixth) digit –and
the earliest evidence of a bamboo diet in ancestral pandas– helping us better understand the evolution of this unique
structure. "Deep in the bamboo forest, giant pandas traded an omnivorous diet of meat and berries to quietly
consuming bamboos, a plant plentiful in the subtropical forest but of low nutrient value," says NHM Vertebrate
Paleontology Curator Dr. Xiaoming Wang. "Tightly holding bamboo stems in order to crush them into bite sizes is
perhaps the most crucial adaptation to consuming a prodigious quantity of bamboo."
This discovery could also help solve an enduring panda mystery: why are their false thumbs so seemingly
underdeveloped? As an ancestor to modern pandas, Ailurarctos might be expected to have even less well-developed
false “thumbs”, but the fossil that Wang and his colleagues discovered revealed a longer false thumb with a
straighter end than its modern descendants' shorter, hooked digit. So why did pandas' false thumbs stop growing to
achieve a longer digit? "Panda's false thumb must walk and 'chew'," says Wang.
"Such a dual function serves as the limit on how big this 'thumb' can become." Wang and his colleagues
think that modern panda’s shorter false thumbs are an evolutionary compromise between the need to manipulate
bamboo and the need to walk. The hooked tip of a modern panda’s second thumb lets them get a grip on bamboo
while letting them carry their impressive weight to the next bamboo meal. After all, the “thumb” is doing double
duty as the radial sesamoid–a bone in the animal’s wrist.
“Five to six million years should be enough time for the panda to develop longer false thumbs, but it seems
that the evolutionary pressure of needing to travel and bear its weight kept the ‘thumb’ short –strong enough to be
useful without being big enough to get in the way,” says Denise Su, associate professor at the School of Human
Evolution and Social Change and research scientist at the Institute of Human Origins at Arizona State University,
and co-leader of the project that recovered the panda specimens. “Evolving from an omnivorous ancestor and
becoming a pure bamboo-feeder, pandas must overcome many obstacles,” Wang says. “An opposable ‘thumb’ from
a wrist bone may be the most amazing development against these hurdles.”

01. The main purpose of the first paragraph is to 03. Which of the following can complete the missing
part in paragraph 2 of the passage?
(A) question a suggestion. (A) how this wrist bone evolved was not
(B) analyze a report. understood due to a near-total absence of
(C) outline a hypothesis. fossil records.
(D) describe a phenomenon. (B) how this wrist bone evolved was fully
(E) criticize a research finding. understood due to a near-total absence of
fossil records.
02. Which choice most closely captures the meaning (C) how this wrist bone evolved was well
of the word “subsist” used in the first paragraph? understood despite a near-total absence of
fossil records.
(A) consume (D) how this wrist bone evolved was not
(B) munch understood in spite of a near-total absence of
(C) devour fossil records.
(D) survive (E) how this wrist bone evolved was understood
(E) depend regarding a near-total absence of fossil
records.

- 67 - BTA / Ing / S. Intensif 2023


04. In the context of the fourth paragraph, the word 08. Which paragraph of the passage clarifies the idea
“manipulate” most nearly means that there used to be a swift in panda’s menu?

(A) cheat on (A) 1


(B) intrigue
(B) 2
(C) curtail
(D) garner (C) 3
(E) handle
(D) 4
05. The notion that pandas must walk and 'chew' (E) 5
(third paragraph) is mentioned in order to clarify
the reason why their false thumbs
09. The author makes his main point in the last
paragraph with the aid of
(A) gradually disappeared
(B) ceased to develop
(A) frequent use of metaphor
(C) were longer
(D) changed their function (B) personal statement with examples
(E) didn’t meet the standard to be herbivorous
(C) deductions with several quotations
06. According to the passage, the author implies that (D) repetition and parallel construction
ancient pandas used
(E) rethorical questions
(A) to have shorter false thumbs with hooked
end 10. This passage is related to the specific field of
(B) their so-called “radial sesamoid” to identify
whether certain plants are edible or not (A) botany
(C) to have keen instinct that told them when
(B) conservation
and where to find prodigious quantity of
bamboo (C) ecology
(D) to base their diet on both plant and animal
(D) evolution
origin
(E) their short false thumbs to crush bamboos (E) genetics
into small pieces

07. Which choice best describes the developmental


pattern of the passage?

(A) A detailed depiction of research procedures


(B) An accurate response to a series of
hypotheses
(C) A cheerful recounting of an amusing story
(D) A firm rebuttal against research analysis
(E) An exploratory analysis of a research

PASSAGE 2

Were you to sail through school with high grades and perfect test scores, you probably did it with traits
beyond sheer smarts. A new study of more than 6000 pairs of twins finds that academic achievement is influenced
by genes affecting motivation, personality, confidence, and dozens of other traits, in addition to those that shape
intelligence. The results may lead to new ways to improve childhood education. "I think this is going to end up
being a really classic paper in the literature," says psychologist Lee Thompson of Case Western Reserve University
in Cleveland, Ohio, who has studied the genetics of cognitive skills and who was not involved in the work. "It's a
really firm foundation from which we can build on."

- 68 - BTA / Ing / S. Intensif 2023


Researchers have previously shown that a person's IQ is highly influenced by genetic factors, and have
even identified certain genes that play a role. They've also shown that performance in school has genetic factors. But
it's been unclear whether the same genes that influence IQ also influence grades and test scores. In the new study,
researchers at King's College London turned to a cohort of more than 11,000 pairs of both identical and nonidentical
twins born in the United Kingdom between 1994 and 1996. Rather than focus solely on IQ, as many previous studies
had, the scientists analyzed 83 different traits, which had been reported on questionnaires that the twins, at age 16,
and their parents filled out. The traits ranged from measures of health and overall happiness to ratings of how much
each teen liked school and how hard they worked. Then, the researchers collected data on how well each individual
scored on the General Certificate of Secondary Education (GCSE) exam, an exam that all students in the United
Kingdom must take and which is used for admission to advanced classes or colleges.
The team found nine general groups of traits that were all highly hereditary—the identical twins were more
likely to share the traits than nonidentical twins—and also correlated with performance on the GCSE. Not only were
traits other than intelligence correlated with GCSE scores, but these other traits also explained more than half of the
total genetic basis for the test scores. In all, about 62% of the individual differences in academic achievement—at
least when it came to GCSE scores—could be attributed to genetic factors, a number similar to previous studies'
findings, the team reports online today in the Proceedings of the National Academy of Sciences.
“It’s really important to understand why children vary in academic achievement,” says developmental
psychologist Kaili Rimfeld of King’s College London, an author of the new paper. “These twin studies show that
there’s a genetic basis for the differences in how easy or enjoyable children find learning.” Understanding that
there’s a genetic basis for why people differ in not only intelligence, but also their drive to learn, she says,
underscores the need for personalized classrooms where students can learn in different ways—from computer
programs to hands-on projects—that are most fitted to their own personalities.
The results, Thompson points out, would likely differ in less-developed countries where children don’t
have equal access to education; academic achievement in these places is shaped more by opportunities than genetics.
And the new study gives little information on what the genes might be that influence test scores. “Each one of these
traits is very complex,” she says, “so we’re talking about hundreds of genes that are acting together.” Future studies,
she says, may be able to shed light on specific genes that affect academic achievement, which could help diagnose
or treat learning disabilities.

11. Referring to paragraph 1 of the passage, it is said 13. What is the implied connection between
this is going to end up being a really classic paragraphs 2 and 3?
paper in the literature, which should mean
(A) The former serves as an introductory
(A) The research will end all kind of classic statement followed by more detailed
paper. descriptions provided in the latter.
(B) All classic paper will be replaced by this (B) The research procedure explained in the
one. former is followed by a finding in the latter.
(C) The research will result in a great paper. (C) The latter confronts all information given in
the former.
(D) The paper will come up after the research is
(D) There is a cause-and-effect relationship
accomplished.
between both paragraphs.
(E) Researchers should refer to classic paper if (E) The former serves as a general statement;
they want to gain a satisfactory result. the latter depicts examples.

12. Which word is synonym to “cohort” used in the 14. All of the following are said to have been the
second paragraph? focus of the latest study, except students’

(A) adversary (A) health


(B) set (B) IQ
(C) clan (C) happiness
(D) group (D) preference for school
(E) accomplice (E) effort in school

- 69 - BTA / Ing / S. Intensif 2023


15. It can be inferred from the whole passage that INFOGRAPHIC ANALYSIS
identical twins ... nonidentical ones.

(A) may perform better in GCSE exam than

(B) may perform worse in GCSE exam than

(C) may have similar academic achievement


unlike

(D) tend to have higher IQ than

(E) have more genes to enhance school


performance than

16. According to Kaili Rimfeld (paragraph 4) it’s


really important to understand why children …
in academic achievement.

(A) different

(B) difference

(C) differently

(D) differing
18. From the infographic, we can deduce that the
(E) differ average work week is close to … of the entire
hours within 7 days.

(A) 30%
17. One essential point the author conveys from the (B) 40%
fifth paragraph is that (C) 45%
(D) 50%
(A) the study of genetics in accordance with (E) 60%
academic achievement doesn’t apply to less-
developed countries 19. Meanwhile, professional and business leaders
who carry mobile devices, like smartphones,
(B) less-developed countries tend to have lower tend to work more than … of the entire hours
rank in academic achievement that their within a week.
developed counterparts. (A) 32%
(B) 42%
(C) if the study of genetics had been applied in (C) 52%
less-developed countries, they would have (D) 62%
ranked impressively in academic (E) 72%
achievement.
20. The average commute time people spend is
(D) less-developed countries encounter a serious around … for a one-way trip.
issue other than opportunities and genetics.
(A) 47 minutes
(E) information about specific genes that affect (B) half an hour
academic achievement will be released in (C) an hour an a half
the near future. (D) 23 minutes
(E) a quarter hour

- 70 - BTA / Ing / S. Intensif 2023


PASSAGE 3

Embezzlement is the act of dishonestly withholding assets for the purpose of conversion (theft) of such assets
by one or more individuals, to whom such assets have been entrusted, to be held and/or used for other purposes.
Embezzlement is a kind of financial fraud. __21__, a financial advisor could embezzle funds from investors, or a
spouse could embezzle funds from his or her partner. Embezzlement may range from the very minor in nature,
involving only small amounts, to the immense, involving __22__ sums and sophisticated schemes. More often,
embezzlement is performed in a manner that is premeditated, systematic and/or methodical, with the explicit intent
to conceal the activities from other individuals, __23__ it is usually done without the other individuals’ knowledge
or consent. It often involves the trusted individual embezzling only a small fraction of the total of the funds he/she
receives or controls; in an attempt to minimize the risk of the detection of the __24__ of the funds or resources.
When successful, embezzlements continue for years without detection. It is often only when a relatively large
proportion of the funds are needed at one time; or, when a major __25__ reorganization (the closing or moving of a
plant, or a merger/acquisition of a firm) requires the complete and independent accounting of all real and liquid
assets. It is often prior to, or concurrent with, the reorganization, that the victims realize the funds, savings, assets or
other resources, are missing and that they have been duped.

21. (A) However 24. (A) misplacing

(B) First (B) misarrangement

(C) In fact (C) misorganization

(D) In addition (D) misproportion

(E) For instance (E) misallocation

22. (A) generous

(B) large 25. (A) institute

(C) humble (B) institution

(D) petite (C) institutionaly

(E) trivial (D) institutional

(E) institutionalize

23. (A) in other words

(B) so that

(C) while

(D) in addition

(E) although

- 71 - BTA / Ing / S. Intensif 2023


BAHASA INGGRIS
PERSIAPAN UJIAN MASUK PTN (13)
Text 1

An ecosystem consists of the biological community that occurs in some locale, and the physical and chemical
factors that make up its non-living or abiotic environment. There are many examples of ecosystems  a pond, a forest,
an estuary, a grassland. The boundaries are not fixed in any objective way, although sometimes they seem obvious, as
with the shoreline of a small pond. Usually the boundaries of an ecosystem are chosen for practical reasons having to do
with the goals of the particular study.
The study of ecosystems mainly consists of the study of certain processes that link the living, or biotic, components
to the non-living, or abiotic, components. Energy transformations and biogeochemical cycling are the main processes
that comprise the field of ecosystem ecology. As we learned earlier, ecology generally is defined as the interactions of
organisms with one another and with the environment in which they occur. We can study ecology at the level of the
individual, the population, the community, and the ecosystem.
Studies of individuals are concerned mostly about physiology, reproduction, development or behavior, and studies
of populations usually focus on the habitat and resource needs of individual species, their group behaviors, population
growth, and what limits their abundance or causes extinction. Studies of communities examine how populations of
many species interact with one another, such as predators and their prey, or competitors that share common needs or
resources.

01. What is the writer’s purpose of writing the passage? 04. Which of the following is irrelevant to the idea of
ecosystems described in the passage?
(A) To present the concept of ecosystems.
(B) To limit the boundaries of an ecosystem. (A) Zoo
(C) To present some examples of ecosystems. (B) Farmland
(D) To clarify the study of ecology at different (C) Desert
levels. (D) Garden
(E) To differentiate between biotic and abiotic (E) Sanctuary
environments.

05. The writer’s tone in writing the passage is ....


02. The ideas in paragraphs 2 and 3 are related in that
paragraph 3 .... (A) skeptical
(B) scientific
(A) provides examples discussed in paragraph 2 (C) critical
(B) summarize some key ideas in paragraph 2 (D) serious
(C) further explains information in paragraph 2 (E) open
(D) argues for ideas presented in paragraph 2
(E) outlines the ideas in paragraph 2

03. The word ‘they’ in “... with the environment in


which they occur.” (paragraph 2) refers to ....

(A) the studies of certain processes


(B) the transformations of energy
(C) the components of non-living
(D) the interactions of organisms
(E) the studies of ecosystems

- 72 - BTA / Ing / S. Intensif 2023


Text 2

Everyone likes to group things. Language students group words as verbs, nouns and so on; collections of words are
classified as phrases, or clauses, or sentences, and these again are reclassified according to their function. In the same
way, botanists classify plants as algae, or fungi, or gymnosperms, etc. Zoologists classify animals as vertebrates and
invertebrates. The vertebrates can be further classified as mammals, reptiles, birds, fish, etc. Classification enables us to
keep hold of more information and, if it is based on the right data, enables us to understand better the ideas we are
studying.
Chemists are no exception. The chemical classification of materials, if it is based on a good system, should enable
us to understand better the many substances which exist in our world. What is to be the basis of our classification?
Perhaps the most obvious one is appearance. Materials could be classified as solid, liquid or gas with some mixed types
as, for example, mud being solid/liquid material and steam a liquid/gas material. Appearance could enable us to
subdivide our main classification groups a little further; the solid may be green, or black, powdery or crystalline; the
liquid may be colored, oily, thick, or free flowing; the gas may be colored. However, we soon realize that many
probably quite different materials have the same appearance. Both air and the deadly carbon-monoxide gas are
colorless, odorless gases, but we would not like to group them as the same thing. Many different liquids are colorless,
water-like materials.

06. The examples provided in paragraph 2 clarify that (D) when appearance is the basis, chemists are not
.... involved
(E) in material classification, chemicals should not
(A) many kinds of liquid should be grouped as one be included
(B) different kinds of gas can be colorless and
odorless
(C) materials in chemistry should be classified 09. The paragraph following the passage most likely
differently deals with the classification of ...
(D) chemical materials have complicated
classifications (A) flora and fauna
(E) taxonomy can be made and applied further to (B) human sounds
other areas (C) liquids and gases
(D) human behaviors
07. Paragraph 2 exemplifies the idea about classification (E) words and phrases
that ....

(A) chemicals may be solid, liquid, and gaseous 10. How does the author organize the ideas?
(B) appearance is not an ever-useful basis in
chemistry (A) Putting the main idea with examples.
(C) the use of colors is better than that of (B) Presenting causes followed by effects.
appearance (C) Interpreting different ways of classifying.
(D) both colors and appearance should be (D) Presenting the strengths of the main idea.
considered (E) Exposing supporting details chronologically.
(E) colors should be included for identifying
appearance

08. The sensence “Chemists are no exception”


(Paragraph 2 line 1) could poosibly be restated as ....

(A) chemical materials can also be put into


classification
(B) classification of chemical materials is without
exception
(C) chemists may also classify materials using
certain criteria

- 73 - BTA / Ing / S. Intensif 2023


Text 3

Passage A

School is necessary because it, first, makes communication with diverse people essential, as parents do not choose
where their children go, and secondly, homeschooling and extra-curricular activities connected to it cannot bring that
diversity, for the attending group is self-selecting rather than “unfiltered mixture”. I believe that none of the two
assumptions is warranted nor true.
In the first place, parents still select schools for their children on the basis of common values, cultures and
achievements. As such, public schools then offer obvious misrepresentations of the society.
Not only that, I think it nonsense that homeschooling should somehow get rid of diversity from socialization in
sports teams or other clubs. What members of a basketball team or a debate club share is not race, religion, nor income
bracket: it is the desire to participate.

Passage B

Children can perhaps be best described as beings of potential during their school years. Each child is in the process
of discovering their talents and having them be cared for in an attempt to best reach self-actualization.
Under the homeschooling system parents become much more exclusive mentors to a child, and this can be
problematic. Although parents are generally aware of what their child is capable of, their evaluations are not always
precise.
This could be for a number of reasons. First, it may be due to the fact that their children are predominately seen in
the home environment, limiting their chances to show off their potential in other situations. It could also be because
parents sometimes assume that their children will share the same talents that they had. Simply stated, it would be
unreasonable to assume parents could see the whole picture.

11. The common theme underlying both passages above (D) parents’ personal preference to a certain type of
is .... school for their children
(E) wrong assumptions on both home school
(A) arguments against home school education education and home schooling
practices
(B) evidence of several problems with 14. The best summary of both passages is ....
homeschooling
(C) reasons why homeschooling should be (A) ideas of school and home school learning are
eliminated wrong; yet, school is better
(D) failure which is associated with homeschooling (B) schools are the right medium for socializing;
(E) formal school and home school education yet parents are still selective
(C) there is no rational support to believe education
12. Which of the following statements reflects a fact at home and at school
mentioned in either passage above? (D) children are growing their ways to develop
their talents and personality
(A) Joining a social club is a matter of wish to (E) parents and teachers make inaccurate
participate. evaluations on children’ talents
(B) Interacting with other individuals is very
important. 15. If a child’s parents are not able to identify the child’s
(C) Children are individuals with inborn capacity. talent comprehensively, a homeschool child’s basket
(D) Sports teams are a form of social unit. ball talent will ....
(E) Parents play a role as a private guru.
(A) never be revealed until after the right expert
13. The idea in Passage B which is different from that in can reveal the child’s talent
Passage A is .... (B) grow normally because somebody’ talents is
not affected by anyone else
(A) thoughts that schools are a medium for (C) grow little and later it may stop growing at all
individual to socialize freely as there is little stimulation
(B) the need to join social events not diversity that (D) remain undevelope because none is able to
home schoolers seek reveal the child’s potential
(C) parents’ limited understanding of their children (E) still grow in so far he/she has the need to
potentials and talents partake in a baske ball team

- 74 - BTA / Ing / S. Intensif 2023


Text 4

Loss of habitat may happen as development takes place in a species range. Many animals have a range of hundreds
of square kilometers. The mountain lion of North America, for instance, has a range of up to 1,000 square kilometers
(386 square miles). To successfully live and reproduce, a single mountain lion patrols this much territory. Urban areas,
such as Los Angeles, California, and Vancouver, British Columbia, Canada, grew rapidly during the 20th century. As
these areas expanded into the wilderness, the mountain lion’s habitat became smaller. That means the habitat can
support fewer mountain lions. Because enormous parts of the Sierra Nevada, Rocky, and Cascade mountain ranges
remain undeveloped, however, mountain lions are not endangered.
Loss of habitat can also lead to increased encounters between wild species and people. As development brings
people deeper into a species range, they may have more exposure to wild species. Poisonous plants and fungi may grow
closer to homes and schools. Wild animals are also spotted more frequently. These animals are simply patrolling their
range, but interaction with people can be deadly. Polar bears, mountain lions, and alligators are all predators brought
into close contact with people as they lose their habitat to homes, farms, and businesses. As people kill these wild
animals, through pesticides, accidents such as collisions with cars, or hunting, native species may become endangered.

16. What does the author want to say to the readers? 19. It can be said that to avoid loss of habitats, what the
government of a country should do is to ....
(A) People will see wild animals more often when
they lose their habitat.
(A) create new and better habitats for wild animals
(B) Development can increase the animals’
encounter with human beings. (B) buid good fauna conservations as many as
(C) Animals will migrate from their habitat as possible
human beings build their homes.
(C) prevent wild animals from coming into human
(D) Development of human settlements may threat
wild animals and human beings. settlements
(E) Development will improve both human (D) move animals from their previous habitat into
settlement and wild animals’ living place. new better places
(E) stop the development of human settlement near
17. Paragraphs 1 and 2 are related in that the latter .... wild animals’ living area

(A) describes human development mentioned in


paragraph 1 20. It seems that the author believes that loss of habitats
(B) continues the explanation about development ....
in paragraph 1
(C) develops the ideas about loss of habitat stated (A) is a part of natural selection in modern
in paragraph 1 civilization
(D) exemplifies loss of habitat due to human
housing set in paragraph 1. (B) reduces the opportunities to encounter wild
(E) explains the effect of the loss due to the animals
development mentioned in paragraph 1 (C) makes wild animals more dangerous to human
beings.

18. The word “territory” in ‘... mountain lion patrols this (D) leads flora and fauna to go into extinction more
much territory’ (paragraph 1) refers to .... quickly
(E) brings native species into a phase of slow
(A) mountain lion’s range regeneration
(B) species range
(C) urban areas
(D) mountain
(E) habitat

- 75 - BTA / Ing / S. Intensif 2023


Text 5
Passage A

Secondhand smoke is the smoke that is exhaled or that comes from the burning end of a cigarette, pipe, or cigar.
Secondhand smoke can come in through cracks in the walls. It can hang around in hallways and doorways where
people have been smoking.
Breathing someone else’s smoke can be deadly, especially if you live or work in a place where people smoke.
That is why it is so important for smokers to go all the way outside if they want a cigarette. When one person
smokes inside, it can cause problems for everyone else.
Children who are around tobacco smoke in their homes have more health problems like asthma and ear
infections. They are sicker and stay in bed more. They miss more school days than children who homes are smoke-
free. Babies who live in homes with secondhand smoke are more likely to die as infants than other babies.

Passage B

The first conclusive evidence on the danger of passive smoking came from Takeshi Hirayama’s study in 1981
on lung cancer in non-smoking Japanese women married to men who smoked. Although the tobacco industry
immediately launched a multimillion dollar campaign to discredit the evidence, dozens of further studies have
confirmed the link. Research then broadened into other areas and new scientific evidence continues to accumulate.
The risk of lung cancer in nonsmokers exposed to passive smoking is increased by between 20 and 30 percent,
and the excess risk of heart disease is 23 percent. Children are at particular risk from adults smoking. Adverse health
effects include pneumonia and bronchitis, caughing and wheezing, worsening of asthma, middle ear diseases, and
possibly neuro-behavioural impairment and cardiovascular disease in adulthood. A pregnant woman’s exposure to
other people’s smoking can harm her fetus. The effects are compounded when the child is exposes to passive
smoking after birth.

21. Which of the following themes is mainly 23. The idea of Passage A is similar to that of
discussed in both passages? Passage B, in that it is ....

(A) The disadvantages of smoking. (A) a good decision for smokers to live without
smoking habits
(B) The danger of becoming smokers.
(B) a great benefit for pregnant women to live
(C) The benefit of being a non smoker. without smoking
(C) an unfortunate for nonsmokers to live
(D) The danger of being a passive smoker.
without smoking habits
(E) The illnessses linked to passive smoking. (D) a list of several health risks for nonsmokers
who live with smokers
(E) a chance to escape from several deadly
illnesses due to smoking
22. Which of the following opinions is mentioned in
both passages? 24. Both passages can be best summarized as which
of the following?
(A) Active smokers’ smoke endangers passive
smokers. (A) Children and pregnant women tend to get
(B) Smoking endangers smokers living with various diseases.
nonsmokers. (B) Children of smoking pregnant women will
be likely to smoke.
(C) Evidence on the danger of passive smoking (C) Many smokers will not have serious health
is conclusive. problems in their life.
(D) Smoking effects the health of a baby of (D) Non-smokers living with smokers are likely
pregnant woman. to get health problems.
(E) Smokers and non-smokers are likely to get
(E) Serious diseases in smokers may result smoking-relates diseases.
from chain smoking.

- 76 - BTA / Ing / S. Intensif 2023


25. Based on both passages, it can be hypothesized
that ....

(A) more people will surely die in the


environment of smokers
(B) home settings determine whether somebody
will smoke or not
(C) a baby born from a mother who smokes
will tend to be a smoker
(D) evidence of lung cancer will be greater in
the baby of a smoking mother
(E) by avoiding passive smoking, chances of
getting deadly illnesses are lesser.

- 77 - BTA / Ing / S. Intensif 2023


BAHASA INGGRIS
LATIHAN UJIAN MASUK PTN (1)
Text 1

A team of French and Swedish researchers has presented new fossil evidence for the origin of one of the most
important and emotionally significant parts of our anatomy: the face. Using micron resolution X-ray imaging, they
show how a series of fossils, with a 410 million year old armored fish called Romundina at its center, documents the
step-by-step assembly of the face during the evolutionary transition from jawless to jawed vertebrates.
Vertebrates, or backboned animals, come in two basic models: jawless and jawed. Today, the only jawless
vertebrates are lampreys and hagfishes, whereas jawed vertebrates number more than fifty thousand species,
including human. It is known that jawed vertebrates evolved from jawless ones, a dramatic anatomical
transformation that effectively turned the face inside out.
In embryos of jawless vertebrates, blocks of tissue grow forward on either side of the brain, meeting in the
midline at the front to create a big upper lip surrounding a single midline "nostril" that lies just in front of the eyes.
In jawed vertebrates, this same tissue grows forward in the midline under the brain, pushing between the left and
right nasal sacs, which open separately to the outside. This is why human face has two nostrils rather than a single
big hole in the middle. The front part of the brain is also much longer in jawed vertebrates, with the result that our
nose is positioned at the front of the face rather than far back between our eyes.
Until now, very little has been known about the intermediate steps of this strange transformation. The scientists
studied the skull of Romundina, an early armored fish with jaws, or placoderm, from arctic Canada. Romundina has
separate left and right nostrils, but they sit far back, behind an upper lip like that of a jawless vertebrate. This skull is
a mix of primitive and modern features, making it an invaluable intermediate fossil between jawless and jawed
vertebrates.
By imaging the internal structure of the skull using high-energy X-rays at the European Synchrotron (ESRF) in
Grenoble, France, the scientists show that the skull housed a brain with a short front end, very similar to that of a
jawless vertebrate. “In effect, Romundina has the construction of a jawed vertebrate but the proportions of a jawless
one," says Per Ahlberg, one of the lead scientists. "This shows us that the organization of the major tissue blocks
was the first thing to change, and that the shape of the head caught up afterwards," he adds.

01. The author’s purpose in writing this article is to 03. The word "invaluable" in paragraph 4 is closest
.... in meaning to ....
(A) persuade that jawless vertebrates evolved
(A) priceless
from jawed ones.
(B) worthless
(B) illustrate the classification of vertebrates.
(C) negligible
(C) explain the origin of jaws in human faces.
(D) luxurious
(D) reveal how a vertebrate came to have a
(E) inconsequential
face.
(E) inform the brain transformation of jawless
04. According to the text, which of the following is
vertebrates.
TRUE about Romundina?
02. According to the text, the following statements
(A) It is an ancient unarmored fish with jaws.
about jawless vertebrates are true EXCEPT ....
(B) It used to exist over a half million years
(A) Jawless vertebrates can transform to jawed
ago.
vertebrates.
(B) Some of their features are apparent on the (C) The nostrils are separate under the lower
skull of Romundina. lip.
(C) They only have one nostril due to the
(D) It has the skull proportions of a jawless
growth of tissue blocks in embryos.
vertebrate.
(D) Currently lampreys and hagfishes are the
only jawless vertebrates. (E) The skull strongly resembles modern
(E) The front part of the brain is much longer in human features.
jawed vertebrates.

- 78 - BTA / Ing / S. Intensif 2023


05. The tone of the passage is ....

(A) amused
(B) cynical
(C) informal
(D) positive
(E) ignorant

Text 2

Passage A
Soaring carbon emissions from a meat-hungry developing world could be cut back substantially by
improving animal breeds and feed, according to a study. It is estimated that livestock farming contributes 18-51 per
cent of the world’s greenhouse gas emissions. Demand for livestock products is predicted to double by 2050 as a
result of growing populations, urbanizations, and better income in the developing world, which will cause emissions
to rise.
The study, published in the Proceedings of the National Academy of Science, suggests that 12 per cent of
the total livestock-related emissions in 2030 could easily be curtailed with simple improvements in production.
These include: switching to more nutritious pasture grasses; supplementing livestock diets based on grass with small
amounts of crop residues or grains; restoring degraded grazing lands; growing trees that trap carbon while producing
leaves that livestock could eat; and adopting more productive breeds.
“Organizations from the West; especially the World Watch Institute, have continued to blame livestock-
keeping for being one of the major polluters of the world, yet livestock keeping’s positives by far outweigh the
negatives,” said Mario Herrera, co-author of the paper and a senior scientist at International Livestock Research
Institute.

Passage B
Livestock farmers in developing countries have relatively small environmental footprint and their animals
provide them with food, income and transport for their crops, said John Byron. “What these farmers need are
technological options and economic incentives that help them intensify their production in sustainable ways,” he
added.
Steinfeld, coordinator of the Livestock, Environment and Development Initiative at the Food and
Agriculture Organization, said: “If one were able to connect this to smallholder development by making poor
farmers benefit through the possibility of carbon offsets and carbon markets, that would indeed create a win-win
situation where one would have socioeconomic benefits, targeting poor people, while reducing the carbon ‘hoof
print’ i.e. the carbon footprint of livestock”.
Improving livestock production should be done to improve livelihoods and not just for climate reasons, said
Kirtana Chandrasekaran, food campaigner for Friends of the Earth. She added that intensive agriculture also
contributes to biodiversity loss so “it’s very dangerous” just to look at lowering emissions when there’s a whole host
of other factors affecting improvement in livestock farming”.

06. Which of following statements can be (C) Desirable livestock farming is parallel with
hypothetically figured out based on the reduction of greenhouse gas emissions,
information in these two texts? improvement of biodiversity, and better
socioeconomic quality of the poor.
(A) The increase in population, urbanization,
(D) Whatever the technological attempts are
and earnings leads to the higher demand for
made to improve livestock production to
livestock consumption, and thus the higher
meet the people’s demand, higher carbon
carbon emissions.
emissions will constantly increase.
(B) The significant increase in greenhouse gas
(E) Although it affects positively the poor’s
emissions is due to changes in people’s
socioeconomic status, livestock farming
lifestyles and activities, including the higher
activities negatively influence biodiversity
demand for livestock products.
as well as global climate condition.

- 79 - BTA / Ing / S. Intensif 2023


07. The following ideas reflect opinions in the two 09. The argument for livestock farming in the first
texts, EXCEPT .... text differs significantly from that in the second
text, in that the first text deals with ....
(A) Better management in livestock farming
links to less gas emissions. (A) seven minor topics; the second three minor
(B) Better life quality should also result from ones.
improved livestock farming. (B) three major topics; the second five major
(C) Sustainable livestock farming potentially ones.
leads to low carbon emissions. (C) one general topic; the second two minor
(D) In spite of its carbon emissions, livestock ones.
farming gives more advantages. (D) two major topics; the second three major
(E) Livestock industries have affected ones.
greenhouse gas emissions significantly. (E) eight minor topics; the second six minor
ones.
08. It can be concluded from the two texts that cattle
productions are considered ‘successful’ if they 10. The theme of these two texts would most likely
.... be ....

(A) contribute more to economic benefits for the (A) research for better livestock farming
humans’ livelihood.
(B) livestock farming and carbon emissions
(B) address reduction of carbon emissions,
biodiversity and better quality of life. (C) livestock as a top source of air pollution.
(C) result from provisions of appropriate
(D) arguments for sustainable livestock farming
technology and economic motivations.
(D) are controlled from the side of economic and (E) cutting carbon emissions in livestock
technological management. farming
(E) consider both technology and natural factors
affecting livestock farming.

Text 3

Scientists are as obsessed with the question of why the super old survive and thrive, as Ponce de Leon was
to find the Fountain of Youth. They want to understand why the Japanese islands of Okinawa are home to the
world’s largest population of centenarians, with almost 600 of its 1.3 million inhabitants living into their second
century --many of them active and looking decades younger than their actual years. Like weekend visitors on the
summer ferry to Martha’s Vineyard, scientists and sociologists block the boats to Sardinia and Nova Scotia, Canada,
to see why those craggy locales hide vast clusters of the super old.
As well as studying these populations intensively to unlock their secrets, scientists have also taken a hard
look at the very old in the U.S., most notably in the New England Centenarian Study, led by Dr. Thomas Perls, a
geriatrician at Boston University. While the very old are happy to offer homespun explanations for their longevity -
”I never took a drink”, “I drank a shot of whiskey every day” - experts are trying to unravel and understand the
biological factors that allow some people to reach 100 while others drop off in their 70s or 80s. Researchers are
particularly interested in determining which factors allow up to 30% of those who reach 100 to do so in sufficient
mental and physical health: a whopping 90% of centenarians, according to Perls, remain functionally independent up
to age 92.
It is pretty obvious even to nonscientists that how you get there depends partly on the genes you are born
with, the partly on lifestyle-what and how much you eat, where you live and what types of stress and trauma you
experience. How much depends on each factor, though, was unknown until Swedish scientists tackled the problem
in 1998. They did it by looking at the only set of people who share genes but not lifestyle: identical twins who were
separated at birth and reared apart. If genes were most important, you would expect the twins to die at about the
same age. In fact, they do no, and the average difference convinced the scientists that only about 20% to 30% of
how long we live is genetically determined. The dominant factor is lifestyle.

- 80 - BTA / Ing / S. Intensif 2023


11. What is the topic of the text above? 14. How is the information of the last paragraph in
the text organized?
(A) Survival.
(B) Youth. (A) Each question is provided with an
(C) Long-life span. illustration.
(D) Old age.
(B) Scientific questions are followed by studies.
(E) Health secrets.
(C) Each study is followed by research findings.
12. Which of the following best expresses the main
idea of the text? (D) Scientific questions are presented from
general to specific.
(A) The population of the elderly people is (E) Three related questions are followed by one
increasing. finding.
(B) Elderly people cluster in particular parts of
the world.
(C) Biological factors influence mental and 15. According to the information in the passage
physical health. people may ....
(D) Genes and life styles are essential for a long-
life span. (A) reach an old age if their parents do so
(E) Several biological factors are at work (B) reach old age if they keep a healthy life style
affecting life span.
(C) reach old age if they are brought up
13. Which of the following is true about the separately from their siblings.
information in the text? (D) not reach old age unless they live in areas
where it is prevalent.
(A) Super old people normally can exceed 100.
(B) Okinawa people look younger at their actual (E) fail to reach an old age unless they are
age. mentally healthy
(C) All alcoholic drinks decrease life
expectation.
(D) All of Dr. Perls’ subjects are self-reliant.
(E) Gene quality contributes much more to life
span.

- 81 - BTA / Ing / S. Intensif 2023


BAHASA INGGRIS
LATIHAN UJIAN MASUK PTN (2)

Text 1

1 There have been a number of differences in the way Geography is now to be approached in the National
Curriculum. It was decided that there would be a renewed emphasis on spatial knowledge, as well as the human and
physical processes. This should cover some technical procedures such as using grid references. There should also be
a renewed commitment towards the concept of fieldwork and the use of maps, as well as written communication.
6 Dealing with geography, the National Curriculum includes certain topics, but not necessarily how they
should be taught. For example, the focus at key stage 1 is developing knowledge about the United Kingdom and the
world. Students should study certain facts such as the world’s seven continents and their locations. They should be
able to name and identify the four countries and capital cities of the UK. Trips to London may include extra-
curricular education that can aid student’s understanding of the United Kingdom. They also should be able to
identify seasonal weather patterns, identify hot and cold areas of the world, and use world maps and globes to
identify the UK and other countries and oceans.
14 As they progress to key stage 2, students are expected to extend their knowledge to include Europe, North
and South America as well as significant human and physical features. They should be able to identify the position
of latitude, longitude, the Equator and other large features of the world such as the Tropics of Cancer and Capricorn.
18 Students at key stage 2 should study more physical geography including the climate zones, biomes, and
features such as volcanoes and earthquakes. Trips to destinations such as Iceland could encourage further learning
about some of the world’s physical geography. There is a huge emphasis on geographical skills at this stage.
Students should be able to use the eight points of a compass, four and six-figure grid references and keys on
Ordinance Survey maps in order to develop their knowledge.

01. What is the topic of the passage? 03. According to the passage, key stage 2 focuses
more on ....
(A) The art of teaching geography.
(B) The world’s physical geography. (A) the Tropics of Cancer and Capricorn
(C) Topical approaches in teaching Geography.
(B) broader geographical abilities
(D) Different strategies in teaching Geography.
(E) The geographical topics in the National (C) patterns of climate change
Curriculum.
(D) geographical physics
02. The word “seasonal” in line 11 in the passage (E) the trip to Iceland
means ....

(A) serial
(B) cyclical
(C) current
(D) regular
(E) situational

Text 2

1 While reading to your baby, it may be the back-and-forth talking, not just the sound of words being read
from a passage, that is the key to language development, according to new research at the University of Iowa (UI).
4 For the study, researchers observed how mothers responded to their 12-month-old babies during book
reading, puppet play, and toy play. They found that the babies made more speech-like sounds during reading than
when playing with puppets or toys. They also discovered that mothers were more responsive to these types of
sounds while reading to their child than during the other activities.

- 82 - BTA / Ing / S. Intensif 2023


9 These findings might help explain why reading has been so strongly associated with language development
in young children. “A lot of research shows that book reading even to infants as young as six months of age is
important to language outcomes, but I am trying to explain why by looking at the specifics, which could be
responding to speech-like sounds,” said Dr. Julie Gros-Louis, assistant professor of psychology at the UI and
corresponding author.
14 The researchers also found that no matter the context, mother’s responses to speech-like sounds were often
imitations or an expansion of the sound. For instance, if the baby said, “Ba,” the mother might respond with “Ba-ba”
or “Ball,” even if it had nothing to do with the story. Mothers also frequently pointed at objects in the pictures and
identified them, such as “horse.”
18 Gros-Louis says she used mothers and their babies for this study because their interactions have been
studied more than those between fathers and their children. That would make it easier to compare the current results
to past findings.

04. What does the passage discuss? 06. What is the most essential information according
to the passage?
(A) Reading to infants is essentially important.
(B) First language is acquired through reading to (A) Babies learn language through word
babies. repetition.
(C) Mother and her baby develop crucial innate (B) A mother teaches language unconsciously to
relationship. her baby.
(D) Infants start learning language at the age of (C) Mother is the most influential figure to
6 months. baby’s language learning.
(E) Infants learn more from their mother than (D) Reading to babies means introducing
father. language to him/her.
(E) Reading activity is more stimulating than
05. The word “explain” in line 9 is best replaced by playing.
....

(A) clarify
(B) inform
(C) narrate
(D) explore
(E) describe

Text 3

In addition to inactive lifestyle and unsupportive environment, genes and family history also contribute to
overweight and obesity cases. Studies of identical twins who have been raised apart show that genes have a strong
influence on a person’s weight. Overweight and obesity tend to run in families. Your chances of being overweight
are greater if one or both of your parents are overweight or obese. Your genes also may affect the amount of fat you
store in your body and where on your body you carry the extra fat. Because families also share food and physical
activity habits, a link exists between genes and the environment. Children adopt the habits of their parents. A child
who has overweight parents who eat high-calorie foods and are inactive will likely become overweight too.
However, if the family adopts healthy food and physical activity habits, the child’s chance of being overweight or
obese is reduced.
Some hormone problems may also cause overweight and obesity, such as underactive thyroid
(hypothyroidism), Cushing’s syndrome, and polycystic ovarian syndrome (PCOS). Underactive thyroid is a
condition in which the thyroid gland does not make enough thyroid hormone. Lack of thyroid hormone will slow
down your metabolism and cause weight gain. Cushing’s syndrome is a condition in which the body’s adrenal
glands make too much of the hormone cortisol. People who have Cushing’s syndrome gain weight, have upper-body
obesity, a rounded face, fat around the neck, and thin arms and legs. PCOS is a condition that affects about 5-10
percent of women of childbearing age. Women who have PCOS often are obese, have excess hair growth, and have
reproductive problems and other health issues. These problems are caused by high levels of hormones called
androgens.

- 83 - BTA / Ing / S. Intensif 2023


Certain medicines may cause you to gain weight. These medicines include some corticosteroids,
antidepressants, and seizure medicines, which can slow the rate at which your body burns calories, increase your
appetite, or cause your body to hold on to extra water. All of these factors can lead to weight gain.

10. What is the best summary of the passage?


07. What topic does the paragraph preceding the
passage most likely discuss? (A) Hormones and medicines are other factors
causing overweight and obesity cases. When
(A) Factors causing overweight and obesity there is not enough thyroid hormone
condition. produced, the process of metabolism
becomes slow, which then increases one’s
(B) Effects of lifestyles on people’s overweight
weight. Problematic body weight also occurs
and obesity.
when there is too much cortisol and high
(C) Unsupportive environment as the cause of level of androgens.
overweight and obesity condition.
(B) Genes and family history are also the risk
(D) Inactive lifestyle and unsupportive factors of overweight and obesity. Genes
environment as the cause of overweight and determine the mechanism of fat storing in
obesity condition. one’s body. Children will become
overweight or obese if their parents do not
(E) Inactive lifestyle, unsupportive environment,
have the habit of consuming healthy food
genes, and family history as the causes of
and regular exercises. There is a link
overweight and obesity condition. between family habit and overweight and
obesity cases in children.
(C) Other causes of overweight and obesity
08. The author’s attitude regarding the causes of
conditions are hormonal condition and
overweight and obesity cases is ....
certain kinds of medicines. Lack of thyroid
hormone, too much hormone cortisol, and
(A) neutral high level of androgens lead to problems in
(B) cynical one’s body weight. Medicines that can cause
overweight and obesity cases are
(C) positive
corticosteroids, antidepressants, and seizure
(D) concerned medicines.
(E) approving (D) Four other causes of overweight and obesity
cases are genes, family history, hormones,
and medicines. Genes influence the
mechanism of fat storing in one’s body as
09. Based on the passage, it can be hypothesized that
well as the part of the body in which fat is
the more a child’s family practices a healthy
stored. It is more likely that children adopt
lifestyle, the ....
things practiced in their family. So when
parents apply a healthy lifestyle, the children
(A) better the child’s mental health will be
may apply the same lifestyle.
(B) higher the possibility for the child to be in a
(E) A part from lack of physical activities and
better shape
healthy life environment, other causes of
(C) less the frequency of regular exercises overweight and obesity cases are genes,
needed by the child family history, hormones, and medicines.
Genes influence how much fat is stored in
(D) less the possibility for the child to become one’s body. Parents whose lifestyle leads to
overweight or obese overweight and obesity conditions will
(E) the more high-calorie food is in need for the probably have children with the same habit.
child consumption Certain hormonal condition and certain
kinds of medicines also create problems
leading to overweight and obesity cases.

- 84 - BTA / Ing / S. Intensif 2023


Text 4

1 Perhaps even more problematic to human welfare in the coming century than a further decrease in the
availability of fresh water is the impact of climatic disruption on the supply of food. As with water, global access to
food is already questionable. In 2009, the number of people suffering from protein-calorie malnutrition exceeded 1
billion after reaching a low of around 830 million in the 1990s. In poor countries around the world, malnutrition
underlies roughly one-third of the entire burden of disease. And roughly 2 to 3 billion people already suffer from
micronutrient deficiencies.
7 Looking forward, farmers around the world will need to double agricultural production by 2050 in order to
keep up with demand from a growing and more prosperous human population that prefer a more meat-based diet.
This doubling of output will need to occur despite problems already found in agricultural productivity. Water
scarcity, as discussed above, is a major constraint to increasing agricultural production. In addition, roughly one-
third of the Earth’s land surface suffers from land degradation from the combined effects of soil erosion,
salinization, nutrient depletion, and desertification. Finally, the rise of the biofuels industry is generating enormous
demand for grain. By increasing grain demand and, as a consequence, demand for arable land and irrigation,
growing grains as biofuel feedstock will be deemed more important than human food needs. Thus, growing grains
for biofuel needs an immediate banning action.
17 Amid rapidly rising demand for food, increasing environmental pressures on food production, and growing
human malnutrition, climate change additionally compromises both agricultural yields and the nutritional quality of
the crops produced. Agricultural productivity is well known to be sensitive to changes in growing season
temperatures. Observational, longitudinal, and modeling studies all confirm that a 1°C rise in temperature
corresponds to roughly a 10% reduction in yield of the major grains. As temperatures rise 2 oC - 6°C over the next
century, the reduction in agricultural yield will depend, in part, on our capacity to adapt and, in part, on how
temperature variability changes: but, in general, yields are expected to drop.

11. The passage implies that .... (C) Well known agricultural yields alter due to
the sensitivity of the growing season
(A) in the end, the rise of the biofuels industry is temperatures.
leading to enormous supply and demand for
(D) It is common knowledge that agricultural
crops
yields are highly affected by season and
(B) with the rise of the temperature each year,
temperatures.
human beings will eat food of lower quality
than today (E) The growing season temperatures are well
(C) farmers around the world will need to known to be sensitive to changes of the
multiply agricultural yields by 2050 due to agricultural yields.
their high demand
(D) a bigger problem about human welfare in the
13. Which of the following obviously shows the
next century is decrease in the supply of
fresh water author’s bias?
(E) increasing environmental pressures and food
(A) And roughly 2 to 3 billion people already
production compromises both agricultural
experience malnutrition.
yields and the nutritional quality of the crops
produced (B) Thus, growing grains for biofuel needs an
immediate banning action.

12. Which of the following is the restatement of the (C) Finally, the rise of the biofuels industry is
sentence ‘Agricultural productivity is well resulting high demand for crops.
known to be sensitive to changes in growing (D) Farmers around the world will need to
season temperatures’, in lines 19-20? increase agricultural production by 2050.

(A) Agricultural yields increases with the (E) The number of people suffering from
differences of the growing season protein-calorie malnutrition exceeded 1
temperatures. billion.

(B) Sensitivity of well known agricultural yields


alters with the growing season temperatures.

- 85 - BTA / Ing / S. Intensif 2023


14. The statement about questionable global access
to food in the text is followed by ....

(A) examples of its effects


(B) analysis of needs
(C) information about its rate of change
(D) analogy of demand
(E) arguments of temperature

Text 5

1 Flocks of chattering African Grey parrots, more than a thousand flashes of red and white on gray at a time,
were a common sight in the deep forests of Ghana in the 1990s. But a 2016 study published in the journal Ibis
reveals that these birds, in high demand around the world as pets, and once abundant in forests all over West and
central Africa, have almost disappeared from Ghana.
5 According to the study, the pet trade and forest loss -particularly the falling of large trees where the parrots
breed -are major factors contributing to the decline. Uncannily good at mimicking human speech, the African Grey
is a prized companion in homes around the world. Research has shown that grays are as smart as a two-year-old
human child - capable of developing a limited vocabulary and even forming simple sentences.
10 The gray parrot has a wide historic range across West and central Africa - 1.1 million square miles (nearly
three million square kilometers - from Cote d’Ivoire and Ghana in West Africa, through Nigeria and Cameroon and
the Congo forests, to Uganda and western Kenya. Ghana accounts for more than 30,000 square miles (75,000 square
kilometers) of that range, and losses of grays there have not been some of the most devastating. These African Grey
parrots were rescued from smugglers and released on Ngamba Island in Lake Victoria. The African Grey parrot is
the single most heavily traded wild bird.
17 “The Grey parrot population in Ghana has declined catastrophically, and the species is now very rare across
the country,” said Nigel Collar, of Bird Life International, a global partnership dedicated to conserving birds and
their habitat. Collar was one of the authors of the paper, which notes that since 1992 Ghana has lost 90 - 99 percent
of its African grays.
21 “Dedicated searching, including visits to roosts, which had as many as 1,200 individuals 20 years ago,
yielded just a handful of gray parrot sightings,” said Nathaniel Annorbah, a Ghanaian graduate student at
Manchester Metropolitan University, in England, who was the study’s lead author.

15. It can be inferred from the passage that .... 16. The sentence ‘Uncannily good at mimicking
human speech, the African Grey is a prized
(A) Ghana accounts for five percents of the total companion in homes around the world,’ in lines
population of gray parrots 6-7 can best be restated as ....
(B) compared to a two-year-old child, a gray
(A) the African Grey is better at responding
parrot is better at producing short sentences
human speech so that it becomes a valuable
(C) there is a tremendous decrease of the gray prize around the globe
parrot population in Ghana in the last two
(B) due to its extraordinary capability of
decades
imitating people’s expressions, the African
(D) compared to other countries in Africa. Grey becomes a highly-valued pet all over
Ghana has the most population of gray the world
parrots
(C) the African Grey becomes a valuable present
(E) Ngamba Island in Lake Victoria is a good around the world because it has an uncanny
place for smuggling gray parrots ability to deliver a speech

- 86 - BTA / Ing / S. Intensif 2023


(D) because of its extremely ability at 18. The author first quotes Collar’s statement and
accompanying people, the African Grey then the statement is followed by ....
becomes the valuable present around the
world (A) an explanation about Collar’s study
(E) the African Grey is good at echoing human (B) a claim from Bird Life International
speech due to its ability in accompanying
(C) a supporting statement from Annorbah
people around the world
(D) a contrasting finding about gray parrots
17. Which of the following statements obviously (E) a description of bird conservation and
shows the author’s bias? habitat

(A) The Grey parrot population in Ghana has


declined catastrophically.
(B) The African Grey parrot is the single most
heavily traded wild bird.
(C) The gray parrot has a wide historic range
across West and central Africa.
(D) The losses of grays in Ghana forest are not
the most devastating matters.
(E) The pet trade and forest loss are two main
factors causing the decline of Grey bird
population.

- 87 - BTA / Ing / S. Intensif 2023


BAHASA INGGRIS
LATIHAN UJIAN MASUK PTN (3)

Text 1

Over this decade, employment in jobs requiring education beyond a high school diploma will grow more rapidly
than employment in jobs that do not; of the 30 fastest growing occupations, more than half require post-secondary
education. With the average earnings of college graduates at a level that is twice as high as that of workers with only
a high school diploma, higher education is now the clearest ___01___ into the middle class.
In higher education, the U.S.. has been outpaced internationally. While the United States ranks ninth in the world in
the proportion of young adults enrolled in college, we’ve fallen to 16th in the world in our share of certificates and
degrees awarded to adults ages 25-34 — lagging behind Korea, Canada, Japan and other nations. We also suffer
from a college attainment gap, as high school graduates from the wealthiest families in our nation are almost certain
to continue on to higher education, while just over half of our high school graduates in the poorest quarter of
families attend college. And while more than half of college students graduate within six years, the ___02___ for
low-income students is around 25 percent.
Acknowledging these factors early in his Administration, the former President Obama challenged every American to
commit to at least one year of higher education or post-secondary training. ___03___ that America would once
again have the highest proportion of college graduates in the world by 2020,.
(www.whitehouse.gov)

01. The option that best completes is …. 03. The option that best completes is ….

(A) effort (A) Americans will deserve higher education for


(B) position their future
(C) beginning (B) the President has set up a new educational
(D) advantage goal for the country
(E) pathway (C) American students are suggested to take
entrepreneurial skills
(D) the government recommends Americans for
02. The option that best completes is …. college education
(E) middle class Americans are invited to
(A) learning achievement provide financial aids
(B) academic capacity
(C) logical understanding
(D) completion rate
(E) potential understanding

Text 2
Every January, many people start working out, hoping to lose weight. But as studies attest, exercise often
produces little or no weight loss – and even weight gain – and resolutions are soon abandoned. But new science
suggests that if you stick with the right kind of exercise, you may change how your body interacts with food. It is
more than a matter of burning calories; exercise also affects hormones.
A study in 2012 from the University of Wyoming looked at a group of women who either ran or walked
and, on alterenate days, sat quietly for an hour. After the running, walking or sitting, researchers drew blood to test
for the levels of certain hormones and then directed the women to a room with a buffet. Human appetite is
complicated, driven by signals from the brain, gut, fat, cells, glands, genes and psyche. But certain appetite-related
hormones, in particular ghrelin, which sstimulates hunger, are known to be instrumental in determining how much
we consume.
The study has shown that exercise typically increases the production of ghrelin. Workouts make you
hungry. In the Wyoming study, when the women ran, their ghrelin levels spiked, which should have mant they
would attack the buffet with gusto. But they did not. In fact, after running, these women consumed several hundred
fewer calories than they burned.
(www.indianexpress.com)

- 88 - BTA / Ing / S. Intensif 2023


04. The author presents an effect of having specific 06. Which of the following best restates the sentence
exercise followed by …. “…, these women consumed several hundred
fewer calories than they burned.” (paragraph 3
(A) exemplifying reasons for the increase of lines 3-4)?
ghrelin
(B) discussing the relation between appetite and (A) Burning calories for the women was more
exercise important than having calories.
(C) arguing for the importance of having (B) The women’s calory intake was less than
sufficient exercise that of their calory burning.
(D) explaining why ghrelin increase does not (C) Instead of burning fewer calories, the
affect appetite women consumed more calories.
(E) demonstrating why hunger comes after (D) Rather than having more calories, the
exercise women got rid of fewer calories.
(E) When consuming fewer calories, women
05. Based on the information in paragraph 2, the needed to burn more.
author implies that ….
07. The paragraph following the passage will likely
(A) women’s level of appetite is related to their ….
daily physical activity
(B) women are sensitive in response to (A) show the worth of types of exercise
treatments during an experiment (B) deal with merits of weight loss
(C) a particular hormone is responsible for the (C) explain the work of ghrelin
sum of food consumption (D) discuss hormones pertinent to appetite
(D) human’s appetite is a host of either physical (E) describe the implication of the research
or psychological factors findings
(E) innovative research methods are employed
in a study of eating habits

Text 3

Mental illness involves specific types of mood disorders that cause cognitive distortions. When people
suffering from one of these illnesses perceive reality incorrectly, they will likely find themselves suffering from
sadness, anxiety, hallucinations, or psychosis. With clinical depression itself, people may have both good moods and
bad moods from time to time. The bad moods, however, will likely predominate.
When people are excessively pessimistic and feel like they are experiencing a mid-life crisis, they might very
well be suffering from a major depression disorder. Misery and lack of hope may come to the forefront, and people
might very well feel that they can no longer go on. It is at this point that they will need to schedule a consultation
with a licensed therapist. In some cases, they might be started on anti-depressant medications. The goal is to correct
cognitive distortions so that the patients can regain their passion for life. The worst cases may take many months to
overcome. Strong support from loved ones is essential.
Beyond unipolar depression, which might stem from thyroid mood swings or a hormonal imbalance, the other
major type of depression is bipolar disorder. Signs of bipolar disorder might differ from person to person and can
thus be hard for professionals to tease out. Swinging from despair to euphoria will indicate presence of bipolar
manic depression. In some cases, patients might believe that they have superhuman powers or traits. People who are
bipolar might also spend money without regard to the future. They could also engage in risky behaviors.
(www.beingsad.com)

08. Based on the passage above, it can be inferred


that ….

(A) clinically people tend to have changing (D) the first aid for those with mental illnesses is
moods over time having a psychiatrist
(B) depressions are closely associated with one’s (E) essentially there is no best theraphy for
cognition state mentally disorder people
(C) people who get angry easily may suffer from
mental disorders

- 89 - BTA / Ing / S. Intensif 2023


09. Another way of saying “The bad moods, (D) effective curative measures for all kinds of
however, will likely predominate.” (paragraph 1 mental disorders
lines 4-5) is that …. (E) descriptions of unipolar and bipolar types of
depression
(A) even so more and more people suffer from
bad feelings
(B) nonetheless depressions happen to anyone 11. In paragraph 2 the author presents a serious case
more often of mental disorderliness followed by a/an ….
(C) yet one’s happiness in life tends to be hard
to achieve (A) argument for the possible causes
(D) but there is a general tendency of feeling (B) description of clinical diagnosis
sorrowful (C) suggestion for curative measusres
(E) nevetheless people tend to feel gloomier at a (D) explanation on an expert’s therapy
time (E) recommendation for effective cures

10. The paragraph following the passage will likely


talk about ….

(A) follow-up actions for anyone experiencing


bipolar psychosis
(B) an axample of how a serious mental disorder
begins to occur
(C) necessary first clinical treatments in the
event of depression

Text 4

The coal, oil, and natural gas that drive the industrial world's economy all contain carbon inhaled by plants
hundreds of millions of years ago—carbon that now is returning to the atmosphere through smokestacks and exhaust
pipes, joining emissions from forest burned to clear land in poorer countries. Carbon dioxide is foremost in an array
of gases from human activity that increase the atmosphere's ability to trap heat. (Methane from cattle, rice fields, and
landfills, and the chlorofluorocarbons in some refrigerators and air conditioners are others.) Few scientists doubt that
this greenhouse warming of the atmosphere is already taking hold. Melting glaciers, earlier springs, and a steady rise
in global average temperature are just some of its harbingers.
By rights it should be worse. Each year humanity dumps roughly 8.8 billion tons (8 metric tons) of carbon into
the atmosphere, 6.5 billion tons (5.9 metric tons) from fossil fuels and 1.5 billion (1.4 metric) from deforestation.
But less than half that total, 3.2 billion tons (2.9 metric tons), remains in the atmosphere to warm the planet. Where
is the missing carbon? "It's a really major mystery, if you think about it," says Wofsy, an atmospheric scientist at
Harvard University. His research site in the Harvard Forest is apparently not the only place where nature is breathing
deep and helping save us from ourselves. Forests, grasslands, and the waters of the oceans must be acting as carbon
sinks. They steal back roughly half of the carbon dioxide we emit, slowing its buildup in the atmosphere and
delaying the effects on climate.
(environment.nationalgeographic.com)

12. How are ideas in paragraphs 1 and 2 related? 13. Which of the following is relevant with the idea
of supporting the function of forests as carbon
(A) Carbon dioxide and how the ocean serves as
sink described in the passage?
a huge sink.
(B) Industrialized countries and how they solve
(A) Tree planting.
pollution issues.
(C) Activities in poor countries and the number (B) Reclamation of soil.
of pollutant they produce. (C) Making use of biogas.
(D) Greenhouse phenomena and how research is
(D) Diversifying forest fauna.
conducted to deal with them.
(E) Polluting substances in the atmosphere and (E) Composting household waste.
how the nature absorbs them.

- 90 - BTA / Ing / S. Intensif 2023


14. The author holds the assumption that …. 15. The points discussed in paragraph 2 of the
passage clarify the ideas that ….
(A) household apliances may pose
environmental danger (A) pollutants severely damage forests and
glaciers
(B) carbon dioxide is the only pollutant
threatening global temperature (B) universities play important roles in saving
our planet
(C) poor countries produce more carbon than
that of industrialized countries (C) nature has its own mechanisme to getting rid
of pollution
(D) the carbon emitted to the air at present is the
same carbon absorbed by plants in the past (D) why the atmosphere is still warm is an
unsolved mystery
(E) the rate of melting glaciers is proportional
with the increasing number of carbon in the (E) greenhouse gases are essential indicators of
air climate change

- 91 - BTA / Ing / S. Intensif 2023


BAHASA INGGRIS
LATIHAN UJIAN MASUK PTN (4)

Text 1

If you are like most Americans, you probably do not get eight hours sleep each night. But, if you also
constantly feel exhausted, experience headaches for no obvious reason or have high blood pressure, you could have a
more serious problem. That is because these can all be the result of snoring, which is, in turn, the most common
symptom of potentially serious health problem-obstructive sleep apnea (OSA).
While most people think of snoring as a minor annoyance, research shows it can be hazardous to your health.
That is because for over 18 million Americans it is related to OSA. People who suffer from OSA repeatedly and
unknowingly stop breathing during the night due to a complete or partial obstruction of their airway. It occurs when the
jaw, throat, and tongue muscles relax, blocking the airway used to breathe. The resulting lack of oxygen can last for a
minute or longer, and occur hundreds of times each night.
Thankfully, most people wake when a complete or partial obstruction occurs, but it can leave you feeling
completely exhausted. OSA has also been linked to a host of health problems including: acid reflux, frequent nighttime
urination, memory loss, stroke, depression, diabetes, and heart attack. People over 35 are at higher risk.
OSA can be expensive to diagnose and treat, and is not always covered by insurance. A sleep clinic will
require an overnight visit. Doctors then analyze the data and prescribe one of several treatments. These may require you
to wear uncomfortable device that force air through your nose and mouth while you sleep to keep your airways open,
and may even include painful surgery.
Fortunately, there is now a comfortable, far less costly and invasive treatment option available. A recent case
study published by Eastern Virginia Medical School’s Divisions of Sleep Medicine in the Journal of Clinical Sleep
Medicine concludes that wearing a simple chinstrap while you sleep can be effective treatment for OSA. The chinstrap,
which is now available from a company called ‘My Snoring Solution’, works by supporting the lower jaw and tongue,
preventing obstruction of the airway. It is made from a high-tech, lightweight, and super-comfortable material.
Thousands of people have used the ‘My Snoring Solutions’ chinstrap to help relieve their snoring symptoms, and they
report better sleeping, and better health overall because of it.

01. What is the topic of the text? 04. The word “complete” (par. 2) is most similar to
which of the following?
(A) Causes, impacts and treatments of snoring
(B) Snoring as a symptom of health problems (A) Perfect
(C) OSA as occurring among Americans (B) Finished
(D) Diagnosis and treatments of patients with OSA
(C) Blocking
(E) Factors causing Americans to suffer from OSA
(D) Whole
02. The purpose of the text is to .... (E) Covered

(A) show several aspects related with OSA


05. Which of the following statements is NOT stated in
(B) persuade patients with OSA to buy a new the text?
invention
(C) expose reasons for immediate treatments of (A) People suffering from OSA repeatedly stop
OSA breathing during the night.
(D) describe physiological processes of OSA (B) Snoring is a slight annoyance, but it can be
(E) explain recent inventions to cure patients with hazardous to health.
OSA
(C) The chinstrap is now available in supermarkets
in most countries.
03. The pronoun “it” (par. 2) refers to ....
(D) Most people wake when a complete or partial
(A) your health obstruction occurs.
(B) snoring (E) The chinstrap is made from a high-tech and
(C) health problem comfortable material.
(D) research
(E) minor annoyance

- 92 - BTA / Ing / S. Intensif 2023


06. It is implied in paragraph 5 that ....

(A) recent tools to treat OSA are more sensitive


(B) treatments of OSA are research-based
(C) snoring can be greatly reduced by wearing a
chinstrap
(D) causes of snoring have been reported in a journal
(E) patients with OSA have a new hope in the
treatment with their illness

Text 2

In another new study, researchers found high levels of mercury in the feathers of saltmarsh sparrows living
in the wetlands of North Cinder in New York’s Long Island. Whereas loons accumulate mercury by eating large
fish, songbirds are poisoned by eating spiders and other bugs.
“You might ask what difference it makes if a little bird is suffering from mercury poisoning,”
conservationist Timothy Tear of the Nature Conservancy in New York says. But matter it does, since the effect of
mercury on birds provides information about how the heavy metal affects the brain. Researchers from the College of
William and Mary studied song performance in four species of birds along the South River in Virginia, an area that
was contaminated by industrial mercury from 1929 until 1950. They found that wrens affected by mercury sang
shorter songs with fewer notes. Affected sparrows produced mating calls that had a lower tonal frequency and were
less complex.
“Mercury is far more prevalent in our environment than anybody thought. All these decades, people were
out looking for mercury and missing this key group, the little songbirds out in the yard, “Evers says, “These birds
have a message for us: Mercury is everywhere on Earth now, at levels that Mother Nature never intended.”
(discovermagazine.com)

07. In presenting the ideas, the writer starts by 09. In other words, the sentence “Mercury is
describing …. everywhere on Earth now, at levels that Mother
Nature never intended.” (paragraph 3 line 3) may
(A) the eating pattern of birds affected by be restated as …
mercury poisoning
(B) the locations contaminated by mercury (A) mercury is reaching dangerous levels
pollution (B) levels of mercury are controlable by nature
(C) the effects of mercury on animals’ behavior (C) accumulation of mercury tends to be
(D) the chain of mercury pollution among alarming
animals (D) mercury in fish can be dangerous for
(E) the latest research findings on mercury pregnant women
contamination (E) issues on mercury are only addressed by
American countries

08. Based on the passage, it can be inferred that ….


10. The paragraph coming before the passage most
(A) mercury affects birds’ frequency of mating likely deals with information on ….
(B) birds are examples of pollution indicator
organisms (A) environmental science experts
(C) birds can spot the location of fish (B) mercury-heavily polluted location
contaminated by mercury (C) research on effects of mercury on humans
(D) birds can give the most reliable information (D) other animals affected by mercury pollution
regarding mercury (E) effects of mercury contamination on ground
(E) contaminated fish are only found in the water
industrialized countries

- 93 - BTA / Ing / S. Intensif 2023


Text 3

Infections with Ebola virus are acute. There is no carrier state. Because the natural reservoir of the virus is
unknown, the manner in which the virus first appears in a human at the start of an outbreak has not been determined.
However, researchers have hypothesized that the first patient becomes infected through contact with an infected
animal.
After the first case-patient in an outbreak setting is infected, the virus can be transmitted in several ways.
People can be exposed to Ebola virus from direct contact with the blood and or secretions of an infected person.
Thus, the virus is often spread through families and friends because they come in close contact with such secretions
when caring for infected persons. People can also be exposed to Ebola virus through contact with objects, such as
needles, that have been contaminated with infected secretions.
Nosocomial transmission refers to the spread of a disease within a health-care setting, such as a clinic or
hospital. It occurs frequently during Ebola HF outbreaks. It includes both types of transmission described above. In
African health-care facilities, patients are often cared for without the use of a mask, gown, or gloves. Exposure to
the virus has occurred when health care workers treated individuals with Ebola HF without wearing these types of
protective clothing. In addition, when needles or syringes are used, they may not be of the disposable type, or may
not have been sterilized, but only rinsed before reinsertion into multi-use vials of medicine.

11. In presenting the ideas, the writer starts by …. 13. In other words, the sentence “Nosocomial
transmission refers to the spread of a disease
(A) exposing how acutely the Ebola virus infects within a health-care setting, such as a clinic or
patients hospital.” (Paragraph 3) may be restated as ….
(B) revealing the first case of the Ebola virus
(A) Nosocomial transmission is a specific
infecting humans
disease that only infects medical settings
(C) stating how the first Ebola virus infection on (B) Nosocomial transmission takes place in
human took place inappropriate settings of medical equipment
(C) Nosocomial infections refer to a health
(D) affirming the confusion of researchers about
center not well-equipped with medical
the unknown virus
standards
(E) describing no clear date about the first (D) Nosocomial transmission is the disease
outbreak of the virus infection transmission from infected health centers to
humans
(E) Nosocomial infections are the transmission
of diseases from patients to their family
12. The passage above implies that …. members
(A) health clinics have no clinical standards in 14. The part following the passage will likely
handling Ebola virus-infected patients discuss about ….
(B) health workers wear inappropriate protective
clothing in African health facilities (A) directions to carefully exploit the virus for
medical purpose
(C) any object owned by patients can be sources (B) details to prevent the occurrence of
of exposure to the Ebola virus nosocomial infections
(D) at the beginning of its outbreak, animals (C) procedures to sterilize medical equipment
became the host of the Ebola virus thoroughly
(D) ways to locate source of nosocomial
(E) close contact with patients is the most transmission
frequent way of being infected by Ebola (E) hints to improve conditions of African
virus health-care facilities

- 94 - BTA / Ing / S. Intensif 2023


BAHASA INGGRIS
LATIHAN UJIAN MASUK PTN (5)

TEXT 1

Passage A
Chemists at the University of Burgos (Spain) have manufactured a sheet that changes colour in the presence of
water contaminated with mercury. The results can be seen with the naked eye but when photographing the membrane
with a mobile phone the concentration of this extremely toxic metal can be quantified. Mercury contamination is a
problem that is particularly affecting developing countries. It proses a risk to public health since it accumulates in the
brain and the kidneys causing long term neurological illnesses. It is emitted from industrial and mining waste, especially
small-scale gold mining.
A team at the University of Burgos have now developed a technique for detecting the presence of this
dangerous metal in water “in a cheap, quick and in situ way,” as explained to SINC by Jose Miguel Garcia, one of the
authors of the study. Details have been published in the Analytical Methods journal.
(www.eurekalert.org)

Passage B
There is growing evidence that people with large numbers of silver fillings are also at risk. It was once
believed that mercury bonded so completely with the other metals used in dental amalgam that it would not escape from
the filling. We now know this is not true, and in fact the amount of mercury vapor leaving fillings has been measured.
Multiple fillings can leak enough mercury into a person's system to cause mercury poisoning. While in the
average adult one or two fillings will not cause harm, having multiple fillings can leak enough mercury into a person's
system to cause mercury poisoning. For women, there are special health concerns. Mercury crosses the placenta, and
also passes into breast milk. And even trace amounts can cause birth defects. Mercury has been linked to regressive
autism, autism spectrum disorder, ADHD, developmental delays, and a variety of physical, cognitive, and behavioral
problems. It has also been linked to intensified PMS (post-menstrual syndrome). All of which means that for a woman,
especially one in her child bearing years, or for a minor, silver fillings can be a serious health hazard.
(www.chabad.org)

01. Based on both passages, it can be hypothesized 03. Passage A differs from passage B in that passage
that …. A…

(A) invaluable information of mercury hazards is (A) gives details on mercury findings
only in papers (B) discusses mercury specific effects
(B) brain and kidneys are mostly the target of (C) describes dangerous metal in water
mercury to poison (D) presents causes of human problems
(C) researchers seem to worry about mercury to (E) deals with general information on mercury
human existence
(D) people have less knowledge on the negative
effects of mercury
(E) a woman suffering from neurological 04. Which of the following statements is true
disorders may get back pains according to both passages?

(A) A small amount of mercury may cause one to


02. Both passages above likely deal with the common be deaf.
theme which is …. (B) Mercury is closely related to human health
problems.
(A) the effect of mercury leaks
(C) Special health concerns on female are due to
(B) the health hazards of mercury mercury.
(C) the danger of mercury poisoning (D) People should be cautious when using
mercury for teeth.
(D) the flow of mercury in blood vessels
(E) Research on mercury is being developed to a
(E) the problematic behavior of mercury large extent.

- 95 - BTA / Ing / S. Intensif 2023


Text 2

Obesity is a medical condition in which excess body fat has accumulated to the extent that it may have an
adverse effect on health, leading to reduced life expectancy and or increased health problems. Body mass index (BMI),
a measurement which compares weight and height, defines people as overweight (pre-obese) when their BMI is
between 25 kg/m and 30 kg/m, and obese when it is greater than 30 kg/m.
Obesity increases the likelihood of various diseases, particularly heart disease, type 2 diabetes, breathing
difficulties during sleep, certain types of cancer, and osteoarthritis. Obesity is most commonly caused by a combination
of excessive dietary calories, lack of physical activity, and genetic susceptability although a few cases are caused
primarily by genes, endocrine disorders, mediations or psychiatric ilness. Evidence to support the view that some obese
people eat little yet gain weight due to a slow metabolism is limitted; on average obese people have a greater energy
expenditure than their thin counterparts due to the energy required to maintain an increased body mass.
The primary treatment for obesity is dieting and physical exercise. To supplement this, or in case of failure,
anti-obesity drugs may be taken to reduce appetite or inhibit fat absorption. In severe cases, surgery is performed or an
intra-gastric balloon is placed to reduce stomach volume and or bowel length, leading to earlier satiation and reduced
ability to absorb nutrients from food.
Obesity is a leading preventable cause of death worldwide, with increasing prevalence in adult and children,
and authorities view it as one of the most serious public health problems of the 215 century. Obesity is stigmatized in
much of the modern world (particularly in the Western world), though it was widely perceived as a symbol of wealth
and fertility at other times in history, and still is in some parts of the world.

05. The author's attitude regarding obesity in the (D) Obesity is widely considered as a symbol of
passage is ... wealth. Therefore, many people who are obese
tend to be reluctant to do a diet.
(A) pessimistic
(E) Obese people eat little but easily gain weight.
(B) assertive To reduce their weight, they actually do a lot
of physical exercise.
(C) objective
(D) concerned
08. It can be hypothesized from the passage that ...
(E) reactive
(A) the more varied food we eat, the more health
risks we will suffer from
06. What topic does the paragraph following the
passage above most likely discuss? (B) the more energy we have, the more food we
consume
(A) How obese people are treated
(C) the wealthier we are, the more likely we will
(B) How obesity is perceived in general be obese
(C) How obesity is succesfully reduced (D) the more diseases due to obesity we suffer
from, the more treatments we need
(D) How people are made aware of obesity
(E) the less physical exercise we do, the more
(E) How the causes of obesity are minimized
likely we will be obese

07. What is the best summary of the passage?

(A) Obesity that is preventable can cause various


diseases. Dieting and physical exercise can
prevent obesity.
(B) Obese people tend to have health problems.
The problems are considered as the 21st
century problems.
(C) Obesity leads to a shorter life because of the
increased health problems. Therefore, people
are advised to have a surgery.

- 96 - BTA / Ing / S. Intensif 2023


Text 3

The Pulitzer Prize came about as part of an attempt by newspaperman Joseph Pulitzer to upgrade the
profession of journalism. Pulitzer, the owner of the New York World and the St. Louis Post-Dispatch, made a proposal
in 1903 to Colombia University to make a $2 million bequest to the university for the dual purposes of establishing a
school of journalism at the university and also establishing prizes for exceptional work in journalism and other fields.
However, the university did not initially respond as one might expect to such a seemingly generous offer.
Interestingly, Columbia University did not immediately agree to the proposal by Pulitzer inasmuch as
journalism was not held in high regard in general and the Pulitzer papers were more known for their sensationalization
of the news than for the high quality of journalism. The trustees of the university were not at all sure that they wanted a
school of journalism because newspaper reporting was considered more of a trade than a profession at the time and they
did not want to decrease the academic prestige of their institution. It took years of discussions and negotiations before
the terms for the establishment of the school of journalism and the prizes bearing Pulitzer’s name were agreed upon, and
it was not actually until the year after Pulitzer’s death in 1911 that construction began on the building to house
Columbia’s new school of journalism. The school of journalism opened in 1913, and the first prizes were awarded in
1917, for work done the previous year.
The method for selecting Pulitzer Prize winners and the categories for prize has changed slightly over the
years. Today, 21 different awards are given in three different areas, with the majority of awards going to journalists; 14
of the 21 awards are from various aspects of journalism (i.e. news reporting, feature writing, cartoons, and
photography), 6 awards are given in letters (in fiction, non fiction, history, drama, poetry, and biography), and 1
award in music. Columbia University appoints nominating juries consisting of experts in each field, and the nominating
juries submit these nominations for each category to the Pulitzer Prize Board, which makes the final decisions and
awards the prizes.

09. Historically, the Pulitzer Prize was particularly 12. Which of the following statements reflects a fact
meant to be awarded to …. mentioned in the passage about the Pulitzer Prize?

(A) the Columbia University (A) It was after Pulitzer’s death that prizes were
(B) the best work in journalism awarded.
(C) the Pulitzer’s newspapers
(B) The first prizes were awarded by Pulitzer
(D) the best school of journalism
himself.
(E) the outstanding writer in literature
(C) All kinds of writings might receive Pulitzer
10. The following were offered by John Pulitzer to the prizes
Columbia University except ….
(D) The prizes were awarded by experts of the
Columbia University.
(A) a large sum of money
(B) the establishment of a school of journalism (E) The majority of the prizes go to the best
(C) a grant for exceptional writers newspapers in the world.
(D) the addition of a new study program
(E) scholarships for the best students

11. We can infer from the passage that one of the main 13. Another way of saying “journalism was not held
reasons why the Columbia University’s first in high regard” in paragraph 2 sentence 1 is that
reaction to Pulitzer’s offer was negative is because ….

(A) journalism was considered unimportant
(A) the Pulitzer papers were known to focus on (B) journalism was entirely ignored
trivial things.
(B) the need for journalists was at that time not yet (C) journalism was not acceptable
recognized. (D) journalism was quite unpopular
(C) the study program offered was considered
teaching a skill not science. (E) journalism was generally not recommended
(D) the offer had decreased the academic prestige
of the institution.
(E) the school of journalism would take a long
time to establish.

- 97 - BTA / Ing / S. Intensif 2023


BAHASA INGGRIS
INTERNATIONAL EXAM (1)

Text 1
Paragraph one
When the explorer comes home victorious, everyone goes out to cheer him. We are all proud of his achievement,
proud on behalf of the nation and of humanity. We think it is a new feather in our cap, and one we have come by
cheaply. How many of those who join in the cheering were there when the expedition was fitting out, when it was
short of bare necessities, when support and assistance were most urgently wanted? Was there then any race to be
first? At such a time the leader has usually found himself almost alone; too often he has had to confess that his
greatest difficulties were those he had to overcome at home before he could set sail. So it was with Columbus, and
so it has been with many since his time.

Paragraph two
Amundsen has always reached the goal he has aimed at, this man who sailed his little yacht over the Arctic Ocean,
round the north of America, on the course that had been sought in vain for four hundred years. So, when in 1910 he
left the fjord on his expedition in the Fram, to drift right across the North Polar Sea, would it not have been natural if
we had been proud to support such a man? But was it so? For a long time he struggled to complete his equipment.
Money was still lacking, and little interest was shown in him and his work. He himself gave everything he possessed
in the world. But nevertheless had to put to sea loaded with anxieties and debts, as he sailed out quietly on a summer
night.
Adapted from the introduction by Fridtjof Nansen to The South Pole, R Amundsen (1912)

01. In paragraph one, the race to be first refers 03. Both paragraphs make their point with the aid of
ironically to the
(A) repetition and parallel construction
(A) lack of response to urgent appeals for help (B) specific details of time and place
(B) willingness to give credit (C) metaphor
(C) lack of support to the explorer before he (D) reference to historical documents
achieves his goals (E) rhetorical questions
(D) rush to laud the explorer
(E) eagerness of the explorer to be alone
04. From both paragraphs taken together, it appears
that Amundsen and Columbus shared all of the
02. The feather in our cap refers to following except the fact that they

(A) our willingness to take unearned credit for a (A) were explorers
triumph (B) were not always supported when they most
(B) the pride we have in being human needed it
(C) our sense of having got a reward for our (C) achieved feats that should have received
investment accolades
(D) way we respond to all success (D) had difficulties to face apart from those they
(E) the way we express our joy faced on their expeditions
(E) sailed the seas alone

Text 2
The ground is full of seeds that cannot rise into seedlings; the seedlings rob one another of air, light and water, the
strongest robber winning the day, and extinguishing his competitors. Year after year, the wild animals with which
man never interferes are, on the average, neither more nor less numerous than they were; and yet we know that the
annual produce of every pair is from one to perhaps a million young; so that it is mathematically certain that, on the
average, as many are killed by natural causes as are born every year, and those only escape which happen to be a
little better fitted to resist destruction than those which die. The individuals of a species are like the crew of a
foundered ship, and none but good swimmers have a chance of reaching the land.
Adapted from an essay by T H Huxley

- 98 - BTA / Ing / S. Intensif 2023


05. The robber in the first sentence is most like 06. The main point the author conveys is that
which of the following mentioned in the
paragraph? (A) natural populations of animals in the wild
increase in numbers exponentially
(A) wild animals (B) all members of a species are in violent
competition with one another
(B) produce of every pair
(C) in the struggle to survive, the fittest survive
(C) individuals of a species (D) members of one generation of a population
are all more or less alike
(D) crew of a foundered ship
(E) man’s interference destroys the natural
(E) good swimmers balance

Text 3

The literature on drug addiction has grown at a rate that defies anyone to keep abreast of the literature, and
apparently in inverse proportion to our understanding of the subject. Addiction, or dependence, as it is more
fashionable to call it, excites controversy and speculation yet true understanding of the phenomenon remains elusive.
In fact the area is fraught with speculation and acrimonious debate. Definition of terms such as drug, addiction, and
abuse is obviously less controversial than attempts to explain the nature of drug dependence, yet even the
terminology is imprecise and overlain with subjective connotations. At its most basic, a drug, as defined by the
World Health Organization, is simply any substance which when taken into the living organism may modify one or
more of its functions. This kind of definition is too wide to be of any use in a discussion of dependence: it covers
everything from insulin to aspirin, penicillin to alcohol.

07. The author implies that he thinks the term 08. We can infer from the first sentence that
dependence in the context of drugs
(A) not all that has been written on the subject of
(A) is more accurate the older term addiction addiction has added to our understanding
(B) no one can have read all the li terature on
(B) has not always been the preferred term
any drug
(C) is a currently under-used term (C) the more that is published the more we are
likely to understand
(D) is an avant-garde aberration (D) the rate of growth should be higher if we are
(E) is more controversial than the term addiction to understand the subject
(E) writing about addiction is fashionable

Text 4

Rome, founded on the banks of the Tiber river, boasts two ancient walls that, when they were built, surrounded
the city. Although both were built as walls intended to defend the city protectively and stood ten meters tall they
were erected under different historical circumstances.

It’s thought that the Servian Wall was constructed in the early fourth century BCE and named after Servius
Tullius, who was the sixth king of Rome. The eleven-kilometer wall encircled Rome’s seven hills and stood entirely
on the east side of the Tiber River.

The Aurelian Wall, built in the late third century CE by the Roman Emperor Aurelian, was more sturdier than
the older wall. It was nineteen kilometers long greatly expanded and surrounded the city of Rome as well as a small
section of the Tiber’s west bank. Erected almost 600 years after the Servian Wall, the Aurelian Wall protected Rome
while the army was away, defending the empire’s far-flung frontiers from enemy attacks. The massive wall deterred
many enemies who might have been tempted to attack Rome during those intervals the city was sparsely defended.

- 99 - BTA / Ing / S. Intensif 2023


09. The expression “walls intended to defend the city 12. The word “Wall” should be
protectively” should be
(A) NO CHANGE
(A) NO CHANGE (B) Wall had built
(B) defensive walls for defending the city (C) Wall had been built
(C) protectively defensive wall (D) Wall, which built
(D) walls to provide defensive protection (E) Wall was built
(E) defensive walls
13. The phrase “more sturdier than” should be
10. The expression “tall they” should be
(A) NO CHANGE
(A) NO CHANGE (B) much sturdier than
(B) tall. They (C) more sturdier then
(C) tall, they (D) much more sturdier than
(D) tall; they (E) much sturdier then
(E) tall—they
14. Which of the following placements for the
11. The words “It’s” should be underlined portion makes it most clear that it was
Rome that had expanded?
(A) NO CHANGE
(B) Among historians, its (A) Where it is now
(B) After the words surrounded the
(C) It (C) After the word Rome
(D) Its (D) After the words of the
(E) It has (E) After the words the Tiber’s

Text 5

“I have been taken prisoner for the liberties of my country, and never will prove a traitor to her interests.”
Before entering a British-run prison during the American Revolution prisoner of war, James Forten, said these
words as a patriotic rejection of his British captor’s offer to free him and educate him in England.

He knew his chance of surviving imprisonment were slim. Forten also knew that if released at the war’s end or
as part of an exchange, he, a free black man, might be captured and sold into slavery as he journeyed home to
Philadelphia. Forten not only survived but became one of the most successful businessmen and ardent abolitionists
in the United States.

Forten’s rise to prosperity began upon his return home when a sailmaker hired him to design, mend, and sew
wails. Forten’s knowledge of ships, gained from his experiences as a sailor during the war, paid off. He rose to the
position of foreman, and in 1798, Forten bought the sailmaker’s business.

16. Which of the following sentences, if added here,


15. The phrase “Revolution prisoner of war, James would provide the most logical transition from
Forten,” should be the first paragraph to the second paragraph?

(A) NO CHANGE (A) Forten was one of many to serve in the


American Revolution.
(B) Revolution, prisoner of war James Forten,
(B) Forten’s rejection was risky.
(C) Revolution, prisoner of war James Forten (C) However, the offer was no longer interesting
(D) Revolution prisoner of war, James Forten to him.
(D) Such an offer must have been unusual.
(E) Revolution of war prisoner, James Forten (E) Many would later admire Forten’s skills as
an innovator.

- 100 - BTA / Ing / S. Intensif 2023


17. The words “chance of surviving” should be (C) a comparison between Forten’s work as a
businessman and his role as an abolitionist.
(A) NO CHANGE
(D) an analysis of how Forten transitioned from
(B) survival chance
a prisoner to a businessman and abolitionis.
(C) chances to surviving
(D) chances of surviving (E) some reasons why Forten came into his
(E) chance to survive goals.

18. The words “exchange, he,” should be


20. The word “rose” should be
(A) NO CHANGE
(A) NO CHANGE
(B) exchange; he as
(C) exchange--he (B) had risen
(D) exchange. He
(C) arose
(E) exchange; he was
(D) had a rose
19. If the writer were to delete the sentence “Forten
(E) raised
not only survived but became one of the most
successful businessmen and ardent abolitionists
in the United States”, the third paragraph would
primarily lose:

(A) a description of the tactics Forten used to


survive imprisonment and become a
successful businessman and abolitionist.
(B) a transition from a discussion of the
ramifications of Forten’s decision to a
discussion of his success as a sailmaker and
abolitionist.

Text 6

The Bahamas, a series of semitropical islands off the southeast coast of the United States, which are home to
some of the most unusual geological formations in the world: underwater caves known as blue holes. These vertical
caves were formed over thousands of years, and their cold depths provide abundant clues to the islands’ past.

During the formation process, tiny grains of calcium carbonate separated from the seawater. These grains built
up, then compacted, forming the limestone that makes up the islands. Over time, rainwater permeated the porous
limestone but was trapped just above sea level. Buoyed by the denser seawater below. Jostled back and forth by
tides, the layer of slightly acidic, brackish water eroded limestone faster than either rainwater—or seawater—could
alone. As the limestone eroded caves formed.

21. The phrase “a series of semitropical island off 22. The phrase “States, which are” should be
the southeast coast” should be…
(A) NO CHANGE
(A) NO CHANGE
(B) a series of semitropical islands off the (B) States, are
southeastern coast
(C) a series of semitropical islands of the (C) States are
southeastern coast
(D) States,
(D) a series of semitropical islands of the
southeast coast (E) States, in which are
(E) a serie of semitropical islands of the
southeast coast

- 101 - BTA / Ing / S. Intensif 2023


23. Given that all the following statements are true, 25. The phrase “buoyed by” should be
which one, if added here, would most effectively
introduce the topic of paragraph 2? (A) NO CHANGE
(B) being buoyed because of
(A) The Bahamas were formed from calcium (C) it being buoyed by
carbonate, a component of seawater. (D) buoying it was
(E) to be buoyed by
(B) Calcium carbonate, a common rock
substance, is also found in seawater.
(C) Much of the land making up the Bahamas is 26. The phrase “rainwater—or seawater—” should be
still underwater.
(A) NO CHANGE
(D) Most types of limestone contain calcium (B) rainwater, or seawater,
carbonate. (C) rainwater, or seawater
(E) Calcium carbonate is the most essential (D) rainwater or seawater
substance to form the Bahama islands. (E) rainwater, nor seawater

24. The word “built” should be 27. The phrase “limestone eroded caves” should be

(A) NO CHANGE (A) NO CHANGE


(B) are built (B) limestone, eroded caves
(C) will build (C) limestone eroded, caves
(D) were built (D) limestone eroded caves,
(E) build (E) limestone eroding caves

Text 7

In regard to propaganda the early advocates of universal literacy and a free press envisaged only two
possibilities: the propaganda might be true, or it might be false. They did not foresee what in fact has happened,
above all in our Western capitalist democracies - the development of a vast mass communications industry,
concerned in the main neither with the true nor the false, but with the unreal, the more or less totally irrelevant. In a
word, they failed to take into account man's almost infinite appetite for distractions.
In the past most people never got a chance of fully satisfying this appetite. They might long for distractions,
but the distractions were not provided. Christmas came but once a year, feasts were "solemn and rare," there were
few readers and very little to read, and the nearest approach to a neighborhood movie theater was the parish church,
where the performances, though infrequent, were somewhat monotonous. For conditions even remotely comparable
to those now prevailing we must return to imperial Rome, where the populace was kept in good humor by frequent,
gratuitous doses of many kinds of entertainment - from poetical dramas to gladiatorial fights, from recitations of
Virgil to all-out boxing, from concerts to military reviews and public executions. But even in Rome there was
nothing like the non-stop distraction now provided by newspapers and magazines, by radio, television and the
cinema. In Brave New World non-stop distractions of the most fascinating nature (the feelies, orgy-porgy,
centrifugal bumblepuppy) are deliberately used as instruments of policy, for the purpose of preventing people from
paying too much attention to the realities of the social and political situation. The other world of religion is different
from the other world of entertainment; but they resemble one another in being most decidedly "not of this world."
Both are distractions and, if lived in too continuously, both can become, in Marx's phrase, "the opium of the people"
and so a threat to freedom. Only the vigilant can maintain their liberties, and only those who are constantly and
intelligently on the spot can hope to govern themselves effectively by democratic procedures. A society, most of
whose members spend a great part of their time, not on the spot, not here and now and in the calculable future, but
somewhere else, in the irrelevant other worlds of sport and soap opera, of mythology and metaphysical fantasy, will
find it hard to resist the encroachments of those who would manipulate and control it.

- 102 - BTA / Ing / S. Intensif 2023


In their propaganda today's dictators rely for the most part on repetition, suppression and rationalization –
the repetition of catchwords which they wish to be accepted as true, the suppression of facts which they wish to be
ignored, the arousal and rationalization of passions which may be used in the interests of the Party or the State. As
the art and science of manipulation come to be better understood, the dictators of the future will doubtless learn to
combine these techniques with the non-stop distractions which, in the West, are now threatening to drown in a sea of
irrelevance the rational propaganda essential to the maintenance of individual liberty and the survival of democratic
institutions.
Adapted from the introduction to Brave New World, A Huxley (1931)

28. The author would be most likely to agree that


propaganda 30. The author refers to Brave New World
(paragraph 2) as a fictional example of a society
(A) can serve a vital function in democracy in which
(B) is concerned mainly with the irrelevant
(A) non-stop distractions are the main
(C) is now combined with entertainment
instrument of government policy
(D) is universally recognized as a danger
(B) people are totally unaware of political
(E) needs constant vigilance to avoid
realities
(C) entertainment is used to keep people from
29. The early advocates of universal literacy (line 1)
full awareness of social realities
are mentioned as
(D) entertainment resembles religion in its
effects on the masses
(A) advocates of propaganda
(E) non-stop entertainment is provided as it was
(B) opponents of an idea that the author thinks is in Rome
correct
(C) proponents of an idea that the author wishes 31. By intelligently on the spot (paragraph 2) the
to counter author apparently means
(D) people who made wrong predictions about (A) alert to the dangers of propaganda
freedom of the press (B) in a particular society at a particular time
(E) social commentators unaware of man’s (C) in a specific time and place
appetite for distractions (D) conscious of political and social realities
(E) deeply aware of current trends
Text 8
The principle of selection solved the riddle as to how what was purposive could conceivably be brought
about without the intervention of a directing power, the riddle which animate nature presents to our intelligence at
every turn, and in face of which the mind of a Kant could find no way out, for he regarded a solution of it as not to
be hoped for. For, even if we were to assume an evolutionary force that is continually transforming the most
primitive and the simplest forms of life into ever higher forms, and the homogeneity of primitive times into the
infinite variety of the present, we should still be unable to infer from this alone how each of the numberless forms
adapted to particular conditions of life should have appeared precisely at the right moment in the history of the earth
to which their adaptations were appropriate, and precisely at the proper place in which all the conditions of life to
which they were adapted occurred: the humming-birds at the same time as the flowers; the trichina at the same
time as the pig; the bark-coloured moth at the same time as the oak, and the wasp-like moth at the same time as
the wasp which protects it. Without processes of selection we should be obliged to assume a "pre-established
harmony" after the famous Leibnitzian model, by means of which the clock of the evolution of organisms is so
regulated as to strike in exact synchronism with that of the history of the earth!
All forms of life are strictly adapted to the conditions of their life, and can persist under these conditions
alone. There must therefore be an intrinsic connection between the conditions and the structural adaptations of the
organism, and, since the conditions of life cannot be determined by the animal itself, the adaptations must be called
forth by the conditions. The selection theory teaches us how this is conceivable, since it enables us to understand
that there is a continual production of what is non-purposive as well as of what is purposive, but the purposive alone
survives, while the non-purposive perishes in the very act of arising. This is the old wisdom taught long ago by
Empedocles.
Adapted from an essay by A Weismann (1889)

- 103 - BTA / Ing / S. Intensif 2023


32. It can be inferred that the author believes that the 34. The examples in paragraph 2 [the humming-birds
Leibnitzian model (paragraph 1) is at the same time…which protects it.] are
intended to
(A) ingenious and worthy of serious
consideration (A) reinforce the author’s point that is difficult
(B) untenable by all rational people to explain adaptation
(C) an acceptable solution to Kant’s dilemma
(B) show that adaptations must take place only
(D) unworthy of further consideration
at specific times and in specific places
(E) an alternative that might still be valid
(C) give specific illustration of organisms that
33. The author’s primary purpose in this extract is to are particularly well-adapted to their
conditions
(A) suggest that a particular theory explains
(D) show organisms that have evolved
otherwise puzzling phenomena
synchronously in a predestined manner
(B) describe the details of the selection theory
for a lay audience (E) demonstrate that intelligent design is needed
(C) justify a particularly controversial model of for purposive evolution
the origins of life
(D) persuade the reader that Empedocles was
right
(E) prove that selection is the only possible way
of looking at evolutionary biology

Text 9

Should we really care for the greatest actors of the past? Could we have them before us? Should we find them too
different from our accent of thought, of feeling, of speech, in a thousand minute particulars which are of the essence
of all three? Dr. Doran's long and interesting records of the triumphs of Garrick, and other less familiar, but in their
day hardly less astonishing, players, do not relieve one of the doubt. Garrick himself, as sometimes happens with
people who have been the subject of much anecdote and other conversation, here as elsewhere, bears no very
distinct figure. One hardly sees the wood for the trees. On the other hand, the account of Betterton, "perhaps the
greatest of English actors," is delightfully fresh. That intimate friend of Dryden, Tillatson, Pope, who executed a
copy of the actor's portrait by Kneller which is still extant, was worthy of their friendship; his career brings out the
best elements in stage life. The stage in these volumes presents itself indeed not merely as a mirror of life, but as an
illustration of the utmost intensity of life, in the fortunes and characters of the players. Ups and downs, generosity,
dark fates, the most delicate goodness, have nowhere been more prominent than in the private existence of those
devoted to the public mimicry of men and women. Contact with the stage, almost through it’s history, presents
itself as a kind of touchstone, to bring out the bizarrerie, the theatrical tricks and contrasts, of the actual world.

35. In the expression One hardly sees the wood for 36. The doubt referred to in line 4 concerns whether
the trees, the author apparently intends the word
trees to be analogous to (A) the stage personalities of the past would
appeal on a personal level to people like the
(A) features of Doran’s language style author
(B) their contemporaries would have understood
(B) details learned from oral sources
famous actors
(C) personality of a famous actor (C) the acting of famous stage personalities
would appeal to us today
(D) detail’s of Garrick’s life (D) Garrick was as great as he is portrayed
(E) stage triumphs of an astonishing player (E) historical records can reveal personality

- 104 - BTA / Ing / S. Intensif 2023


37. In the last sentence: Contact with the stage, 38. Information supplied in the passage is sufficient
almost through it’s history should be to answer which of the following questions?

(A) No change I. Who did Doran think was probably the best
English actor?
(B) Contact with the stage, almost through its
II. What did Doran think of Garrick?
history
III. Would the author give a definite answer to
(C) Contact with the stage, almost throughout the first question posed in the passage?
it’s history
(A) I only
(D) Contact with the stage, almost throughout its (B) II only
history
(C) I and III only
(E) Contact with the stage, almost through it (D) II and III only
history (E) I, II and III

SAT Identification of Sentence Errors


Source: majortests.com

01. Illiteracy is an enormous problem, it affects millions of people worldwide, and is an impediment to social
progress. No error.

02. The company president has taken steps to ensure that she can handle the pressure and anxiety associated with
the job, including joining a yoga class and enlisting the support of a network of friends. No error.

03. If you are sure that you are in the right, you would not mind an independent examination of the case. No error.

04. The union insisted on an increase in their members’s starting pay, and threatened to call a strike if the company
refused to meet the demand. No error.

05. Television viewers claim that the number of scenes depicting alcohol consumption have increased dramatically
over the last decade. No error.

06. Employees with less personal problems are likely to be more productive. No error.

07. The three richest men in America have assets worth more than the combined assets of the sixty poorest
countries of the world. No error.

08. Shipwrecked on a desert island, coconuts and other fruits formed the basis of the sailor’s diet. No error.

09. Fifty percent of the people alive today have never made a phone call, but thirty percent still have no electricity
connections to their homes. No error.

10. The rhododendron, which ornaments so many English gardens, is not native to Europe. No error.

11. The farmer should not have been so careless as to leave the door of the house unbolted when he had gone to
bed. No error.

12. A census of the island revealed a population of only 10,000 people. No error.

13. The engineer, who is renowned for his ingenuity, has designed a very unique cooling system for our new plant
in Spain. No error.

14. Shoes of those kind are bad for the feet; low heels are better. No error.

15. My father saw how much Uncle Tom was enjoying his early retirement, and so he decided to do the same. No
error.

- 105 - BTA / Ing / S. Intensif 2023


BAHASA INGGRIS
INTERNATIONAL EXAM (2)
Text 1

The principle of selection solved the riddle as to how what was purposive could conceivably be brought
about without the intervention of a directing power, the riddle which animate nature presents to our intelligence at
every turn, and in face of which the mind of a Kant could find no way out, for he regarded a solution of it as not to
be hoped for. For, even if we were to assume an evolutionary force that is continually transforming the most
primitive and the simplest forms of life into ever higher forms, and the homogeneity of primitive times into the
infinite variety of the present, we should still be unable to infer from this alone how each of the numberless forms
adapted to particular conditions of life should have appeared precisely at the right moment in the history of the earth
to which their adaptations were appropriate, and precisely at the proper place in which all the conditions of life to
which they were adapted occurred: the humming-birds at the same time as the flowers; the trichina at the same
time as the pig; the bark-coloured moth at the same time as the oak, and the wasp-like moth at the same time as
the wasp which protects it. Without processes of selection we should be obliged to assume a "pre-established
harmony" after the famous Leibnitzian model, by means of which the clock of the evolution of organisms is so
regulated as to strike in exact synchronism with that of the history of the earth!
All forms of life are strictly adapted to the conditions of their life, and can persist under these conditions
alone. There must therefore be an intrinsic connection between the conditions and the structural adaptations of the
organism, and, since the conditions of life cannot be determined by the animal itself, the adaptations must be called
forth by the conditions. The selection theory teaches us how this is conceivable, since it enables us to understand
that there is a continual production of what is non-purposive as well as of what is purposive, but the purposive alone
survives, while the non-purposive perishes in the very act of arising. This is the old wisdom taught long ago by
Empedocles.
Adapted from an essay by A Weismann (1889)

01. It can be inferred that the author believes that the 03. The examples in paragraph 2 [the humming-birds
Leibnitzian model (paragraph 1) is at the same time…which protects it.] are intended
to
(A) ingenious and worthy of serious consideration
(B) untenable by all rational people (A) reinforce the author’s point that is difficult to
explain adaptation
(C) an acceptable solution to Kant’s dilemma
(B) show that adaptations must take place only at
(D) unworthy of further consideration specific times and in specific places
(E) an alternative that might still be valid (C) give specific illustration of organisms that are
particularly well-adapted to their conditions
02. The author’s primary purpose in this extract is to (D) show organisms that have evolved
synchronously in a predestined manner
(A) suggest that a particular theory explains (E) demonstrate that intelligent design is needed
otherwise puzzling phenomena for purposive evolution
(B) describe the details of the selection theory for
a lay audience
(C) justify a particularly controversial model of
the origins of life
(D) persuade the reader that Empedocles was right
(E) prove that selection is the only possible way of
looking at evolutionary biology

- 106 - BTA / Ing / S. Intensif 2023


Text 2

This passage is part of an introduction written by a well-known doctor and essayist for his 1996 book about rare
neurological disorders.

1 I am writing this with my left hand, although I am strongly right-handed. I had surgery to my right shoulder
a month ago and am not permitted, not capable of, use of the right arm at this time. I write slowly, awkwardly – but
more easily, more naturally, with each passing day. I am adapting, learning, all the while – not merely this left-
handed writing, but a dozen other left-handed skills as well. I have also become very adept with my toes, to
compensate for having one arm in a sling; I was quite off balance for a few days when the arm was first
immobilized, but now I walk differently, I have discovered a new balance. I am developing different patterns,
different habits … a different identity, one might say, at least in this particular sphere. There must be changes going
on with some of the programs and circuits in my brain – altering synaptic weights and connectivities and signals
(though our methods of brain imaging are still too crude to show these).
10 Though some of my adaptations are deliberate, planned, and some are learned through trial and error (in the
first week I injured every finger on my left hand), most have occurred by themselves, unconsciously, by
reprogrammings and adaptations of which I know nothing (any more than I know, or can know, how I normally
walk). Next month, if all goes well, I can start to readapt again, to regain a full (and “natural”) use of the right arm,
to reincorporate it back into my body image, myself, to become a dexterous human being once again.
15 But recovery, in such circumstances, is by no means automatic, a simple process like tissue healing – it will
involve a whole nexus of muscular and postural adjustments, a whole sequence of new procedures (and their
synthesis), learning, finding, a new path to recovery. My surgeon, an understanding man who has had the same
operation himself, said, “There are general guidelines, restrictions, recommendations. But all the particulars you will
have to find out for yourself.” Jay, my physiotherapist, expressed himself similarly: “Adaptation follows a different
path in each person. The nervous system creates its own paths. You’re the neurologist – you must see this all the
time.”
22 Nature’s imagination, as physicist Freeman Dyson likes to say, is richer than ours, and he speaks,
marvellingly, of this richness in the physical and biological worlds, the endless diversity of physical forms and
forms of life. For me, as a physician, nature’s richness is to be studied in the phenomena of health and disease, in the
endless forms of individual adaptation by which human organisms, people, adapt and reconstruct themselves when
faced with the challenges and vicissitudes of life.
27 Thus while one may be distressed by the trials of developmental disorders or disease, one may sometimes
see them as creative too – for if they destroy particular paths, particular ways of doing things, they may force the
nervous system into making other paths and ways, force on it an unexpected growth and evolution. This other side
of development or disease is something I see, potentially, in almost every patient. That such radical adaptations can
occur demand a view of the brain as dynamic and active rather than programmed and static, a supremely efficient
adaptive system geared for evolution and change, ceaselessly adapting to the needs of the organism -- its need,
above all, to construct a coherent self and world, whatever defects or disorders of brain function befall it. That the
brain is minutely differentiated is clear: there are hundreds of tiny areas crucial for every aspect of perception and
behaviour (from the perception of color and of motion to, perhaps, the intellectual orientation of the individual). The
miracle is how they all cooperate, are integrated together, in the creation of a self.
37 This sense of the brain’s remarkable plasticity its capacity for the most striking adaptations, not the least in
the special (and often desperate) circumstances of neural or sensory mishap, has come to dominate my own
perception of my patients and their lives. So much so, indeed, that I am sometimes moved to wonder whether it may
be necessary to redefine the very concepts of “health” and “disease,” to see these in terms of the ability of the
organism to create a new organization and order, one that fits its special, altered disposition and needs, rather than in
the terms of a rigidly defined “norm.”

04. The passage can primarily be described as


(D) A case study followed by a scientific
(A) Scientific evidence used to refute an hypothesis
established theory (E) A personal account leading to a general
(B) Amusing anecdotes countered by a profound observation
insight
(C) Skeptical commentary evolving into a
detached analysis

- 107 - BTA / Ing / S. Intensif 2023


05. The author describes himself as “strongly right- 10. The physiotherapist’s remarks (lines 19-21)
handed” (line 1) in order to reveal the assumption that
(A) Convey the ease with which he learned to be
(A) Patients have complete control over the
ambidextrous
progress of their recovery
(B) Contrast his particular abilities and those of
other individuals (B) Each neurologist follows a different path to
(C) Suggest the difficulties he had to overcome understanding
(D) Evoke a sympathetic response from the (C) All neurologists are aware of the nervous
reader system’s adaptability
(E) Characterize the sources of his physical
strength (D) The author is inadequately informed about
the intricacies of the muscular system
06. The author’s remark in lines 7-8 (“There must ... (E) Some neurologists consider both healthy and
signals”) can best be described as injured brain processes to be parallel

(A) Conjecture
(B) Irony
11. In line 23, “richness” most nearly means
(C) Inquiry
(D) Observation (A) Biological importance
(E) Evidence (B) Economic wealth
(C) Meaning
07. In line 9, “crude” most nearly means
(D) Variety
(A) Obvious (E) Resources
(B) Natural
(C) Offensive
(D) Undeveloped 12. Why does the author mention that he is a
(E) Vulgar physician in lines 24-26?

(A) To emphasize his particular point of view


08. The author’s parenthetical reference in lines 12-
13 serves to (B) To illustrate the limits of scientific
knowledge
(A) Depict his physical capabilities before his (C) To establish that he views brain function
accident strictly from his patients’ standpoint
(B) Highlight the process of learning through
(D) To suggest that his main concern is the
experience
surgery he performs on patients
(C) Explain his continuing inability to perform
simple tasks (E) To dispel any doubt about his credentials in
the field of neurology
(D) Rationalize the frustration he felt about
achieving his goals
(E) Illustrate a point about unconscious
adaptation 13. Lines 27-29 primarily encourage readers to view
disease as
09. In lines 18-19 the surgeon advises the author to
(A) A source of psychological harm
(A) Become more open-minded about muscular (B) An opportunity for productive change
adjustments than he had been
(B) Follow a detailed and specific regimen of (C) An inevitable fact of human existence
rehabilitation (D) A force that retards evolutionary change
(C) Find out how others have dealt with the
same problem (E) A condition to be dealt with on a spiritual
(D) Develop his own procedures for coping level
(E) Endure patiently until he physically recovers

- 108 - BTA / Ing / S. Intensif 2023


Text 3

Gordon reaches over across the car and I'm startled, not so much by his touching me as that I hadn't realized
the car was stopped and thought for a second that Gordon had simply abandoned the steering wheel and we would
crash into the next telephone pole. But now I see that we are in a driveway, the same driveway I have watched him
walk down many times in the past two and a half weeks. The car is off, the parking brake pulled up. Gordon sweeps
my hair to one side and kisses the back of my neck. He has strong lips; I wonder if his kisses are making red marks
on my skin. I wonder if he will kiss my mouth like that. I speculate: What should I do now? How should I touch
him? Does he think we are going to do this thing in the car? There are a thousand things I want to ask him all of a
sudden. I want to find out what high school he went to, what his friends' names are, his favorite books, how he
spent, say, the last ten years of his life. I want to know him. Somewhere there is a puppy wandering away from his
new home, never to be found. A steamboat is charging up the river with passengers on board, waving. A new pilot is
nervous at his first landing. At NASA they are on the verge of a discovery. I think that someday, after I die, I will go
somewhere and be sitting across a card table from another dead person, playing hearts and saying, "When I was
twenty-three I wanted to be a veterinarian but couldn't get into a vet school. ... When I was twenty-seven I fell in
love with a man named Victor and then met a man named Gordon..." Sometimes I don't think I can ever tell a story
of my life accurately because life is still continuing and there's no way to objectively portray it. I could try but it's
like trying to pick up an endless pane of glass -- I can't even get a grip on it. How should I know if I'm doing the
right thing as I let the porch door slam behind me and enter Gordon's house? Maybe when I'm dead my soul will sit
somewhere, map out everything I did and be able to tell exactly where I erred. Maybe in some other life I'll
determine the secret to correctness, to doing things right, to knowing that one can do things right.

14. The first-person narrator, in the very opening of the 17. She would like to
passage, is startled because
(A) she's been lost in thought and quite oblivious (A) go to Gordon's house and play cards with him.
to what has been going on around her. (B) love Gordon more than she had loved Victor.
(B) Gordon reaches across the car and touches her.
(C) be kissed with strong lips, at the back of her
(C) they would crash into the next telephone pole.
neck.
(D) she has watched him many times walk down
that same driveway. (D) discover a way to tell what is right and what is
(E) Gordon has abandoned the steering wheel. wrong.
(E) know more about Victor's previous life.
15. Gordon, in the opening of the story,
(A) used to be her school friend.
(B) is her lover. 18. This is most probably a passage from ________
(C) has been a neighbor for two and a half weeks. a(n) novel.
(D) is a pilot; works for the NASA.
(A) successful
(E) wanted to become a veterinarian, but couldn't
get into a vet school. (B) romantic
(C) best-selling
16. She talks of the puppy wandering away from its (D) historical
home to emphasize
(E) 19th-century
(A) how funny the situation is.
(B) the fact that life just goes on all the same.
(C) there is a whole wide world out there waiting
for her.
(D) how much she is feeling lost herself.
(E) that someday she is certainly going to die.

- 109 - BTA / Ing / S. Intensif 2023


Text 4

Sarah Jeanette Connor was driving across a vast and lonely landscape of cacti and brown sand toward a
frightening range of mountains, shadowed as they were by swollen rain clouds. Lightning fired behind them
threateningly. The promise of a storm was in the air. In a way, the Terminator had killed her, Sarah was thinking as
she bent to pick up her .357 Colt Python revolver just to check that it was loaded. She then slipped it back into the
holster hidden under the dashboard of the open Jeep. She was certainly no longer the naive nineteen-year-old
waitress she had been just a few months ago. That person had died when the two people she loved more than her
own life -- the father of the child now growing in her, and her mother -- had been brutally murdered. She certainly
remembered every detail of these terrible events, but lately she had been learning how to lock the feelings into a
place where they would not interfere with her survival. Because she must survive. For now, Sarah Connor was the
most important human being in the world. Sarah raised a Walkman to her ear and listened to something she had
recorded a few minutes ago when she'd stopped to get gas. "Should I tell you about your father?" she said, as she
glanced down at her swelling stomach. "Will it change your decision to send him here, to his death?' She imagined
the point in the future when that choice would have to be made, and she shuddered. "But if you don't send Kyle, you
can never be." That crazy thought never failed to keep coming back to her mind. Kyle Reese, a young soldier who
had volunteered to be sent through time to protect her from the Terminator, had made her pregnant with the son who
would one day order the father back through time to make her pregnant, so that one day the son would order the
father back in time to make her... Thus, a circle of events, bending infinitely back on itself... She remembered what
her high school science teacher used to say about paradoxes: The universe is one big python, feeding on its own
tail... Randall Frakes, Terminator 2: Judgment Day.

19. Cross out the incorrect statement: Sarah Jeanette 22. She has second thoughts when it comes to
Connor
(A) what she should do now.
(A) is alone.
(B) her 357 Colt Python revolver.
(B) is pregnant.
(C) is driving an open vehicle. (C) her problematic pregnancy at such a time.
(D) has a gun. (D) the recording capabilities of her Walkman.
(E) is a waitress. (E) whether she should tell her son the truth about
his father.
20. Cross out the incorrect statement: Kyle Reese
23. What her high school science teacher used to say
(A) would become the son of his own son. about the universe would support the idea of a
(B) came through time from the future. universe in terms of a(n)
(C) is the Terminator's alter ego.
(A) linear evolution.
(D) is the father of the child Sarah is carrying.
(E) has been killed presumably by the Terminator. (B) passage into the unknown.
(C) vicious circle.
21. Right now, Sarah Connor is the most important
human being in the world and must therefore (D) lineage of pythons.
survive, because (E) paradox inexplicable by scientific means.

(A) Kyle Reese's child will someday fight back the


Terminator.
(B) she is the only one who can kill the Terminator
with her revolver.
(C) she has an important appointment to keep with
the Terminator.
(D) there is a terrible storm where she is heading
in her car.
(E) what her high school science teacher used to
say about paradoxes cannot be true.

- 110 - BTA / Ing / S. Intensif 2023


Text 5

A certain type of superior person is fond of asserting that "everything is relative." This is, of course,
nonsense, because, if everything were relative, there would be nothing for it to be relative to. However, without
falling into metaphysical absurdities, it is possible to maintain that everything in the physical world is relative to an
observer. This view, true or not, is not that adopted by the "theory of relativity." Perhaps the name is unfortunate;
certainly it has led philosophers and uneducated people into confusions. They imagine that the new theory proves
everything in the physical world to be relative, whereas, on the contrary, it is wholly concerned to exclude what is
relative and arrive at a statement of physical laws that shall in no way depend upon the circumstances of the
observer... Both psychology and physics, from their different angles, are compelled to emphasize the respects in
which one man's perception of a given occurrence differs from another man's. Some of these differences are due to
differences in the brains or minds of the observers, some to differences in their sense-organs, some to differences of
physical situation: these three kinds may be called respectively psychological, physiological, and physical... The
kind that concerns us here is the purely physical kind. Physical differences between two observers will be preserved
when the observers are replaced by cameras or recording machines, and can be reproduced in a film or on the
gramophone... If there were no reality in the physical world, but only a number of dreams dreamed by different
people, we should not expect to find any laws connecting the dreams of one man with the dreams of another. It is the
close connection between the perceptions of one man and the roughly simultaneous perceptions of another that
makes us believe in a common external origin of the different related perceptions. Physics accounts both for the
likenesses and for the differences between different people's perceptions of what we call the "same"" occurrence.
But in order to do this it is first necessary for the physicist to find out just what the likenesses are. They are not quite
those traditionally assumed, because neither space nor time separately can be taken as strictly objective. What is
objective is a kind of mixture of the two called "space-time."

24. The proposition, "Everything is relative," is not


viable, because 27. We need some measure of objectivity, and this
may be found in the concept of
(A) it is absurd and nonsensical.
(B) only the superior type of people assert it. (A) similar dreams.
(C) there is nothing for it to be relative to. (B) space-time.
(D) of the undeniable fact that certain types of
superior persons assert metaphysical (C) perceptive physics.
absurdities. (D) observational physiology.
(E) if everything were relative, there would be (E) trustworthy observers.
nothing to judge it by.

25. The name chosen for the "theory of relativity" is 28. Bertrand Russell

(A) misleading. (A) is himself a follower of the Theory of


(B) appropriate. Relativity.
(C) imaginative. (B) is a typical example of certain superior
(D) nonsensical. people.
(E) metaphysical. (C) thinks that the supporters of the Theory of
Relativity are just talking nonsense.
26. The Theory of Relativity would rely on data
(D) would like to meet with a careful observer of
obtained through
physical differences.
(A) The Theory of Relativity would rely on data (E) has become really skeptical about the
obtained through viability of psychology as a science.
(B) careful observers.
(C) different perceptions.
(D) cameras and recording machines.
(E) none of the above.

- 111 - BTA / Ing / S. Intensif 2023


Text 6

The second plan we now have to examine is that of giving to each person what she deserves. Many people,
especially those who are comfortably off, think that this is what happens at present: that the industrious and sober
and thrifty are never in want, and that poverty is due to idleness, extravagance, drink, betting, dishonesty, and bad
character generally. They can point to the fact that a laborer whose character is bad finds it more difficult to get
employment than one whose character is good; that a farmer or country gentleman who gambles and bets heavily,
and mortgages his land to live wastefully and extravagantly, is soon reduced to poverty; and that a man of business
who is lazy and does not attend to it becomes bankrupt. But this proves nothing but that you cannot eat you cake and
have it too: it does not prove that your share of the cake was a fair one. It shows that certain vices and weaknesses
make us poor; but it forgets that certain other vices make us rich. People who are hard, grasping, selfish, cruel, and
always ready to take advantage of their neighbors, become very rich if they are clever enough not to overreach
themselves. On the other hand, people who are generous, public-spirited, friendly, and not always thinking of the
main chance, stay poor when they are born poor unless they have extraordinary talents. Also, as things are today,
some are born poor and others are born with silver spoons in their mouths: that is to say, they are divided into rich
and poor before they are old enough to have any character at all. The notion that our present system distributes
wealth according to merit, even roughly, may be dismissed at once as ridiculous. Everyone can see that it generally
has the contrary effect; it makes a few idle people very rich, and a great many hardworking people very poor.

29. Choose the correct statement: The passage says 32. "Born with a silver spoon in one's mouth." This
that expression means

(A) the hard-working, sober and thrifty people (A) congenitally ridiculous
never want something for themselves.
(B) having no character at all
(B) bad people always lose their money, and
stay poor ever after. (C) lucky to have been born of rich parents
(C) especially those who are comfortably off
think that what one gets is what one (D) having extraordinary talents
deserves. (E) wise enough not to spend all one's money
(D) the poor never want to be rich anyway; they
prefer to be idle rather than work.
(E) we must make a plan to give each person
what she reserves. 33. The author is of the opinion that the present
system is
30. Those who are comfortably off think that getting
rich is due to (A) just
(B) generous
(A) being a farmer or a country gentleman.
(B) having a good character. (C) public-spirited
(C) mortgaging one's lands. (D) thrifty
(D) all kinds of extravagances.
(E) leading an immoral life. (E) unfair

31. "You cannot eat your cake and have it too." This
proverb means

(A) Poverty is due to extravagance and


wastefulness.
(B) Your share of the cake is a fair one.
(C) Anyone can see that certain vices and
weaknesses make us poor.
(D) One may become rich if one does not
overreach oneself.
(E) You have to choose between spending and
saving, and these two are diametrically
opposing.

- 112 - BTA / Ing / S. Intensif 2023


Text 7

Liberalism has its own history and tradition. Socialism has its own formulas and aims. Socialism seeks to pull
down wealth; Liberalism seeks to raise up poverty. Socialism would destroy private interests; Liberalism would
preserve private interests in the only way in which they can be safely and justly preserved, namely, by reconciling
them with public right. Socialism would kill enterprise; Liberalism would rescue enterprise from the trammels
[obstacles] of privilege and preference. Socialism assails the pre-eminence of the individual; Liberalism seeks, and
shall seek more in the future, to build up a minimum standard for the mass. Socialism exalts the rule; Liberalism
exalts the man. Socialism attacks capital; Liberalism attacks monopoly. The inherent vice of Capitalism is the
unequal sharing of blessings. The inherent virtue of Socialism is the equal sharing of miseries. The great principle
which this House ought to guard and cherish is that, when the tax collector comes to the private citizen and takes
from him of his wealth for the service of the public, the whole of that money taken shall go for the purposes for
which it is intended, and that no private interests, however powerfully they may be organized and however
eloquently advocated, shall thrust their dirty fingers into the pie and take the profit for themselves.

34. According to the passage, Liberalism and 37. According to the passage, Capitalism is
Socialism are
(A) preferable to socialism.
(A) grossly similar (B) as bad as Socialism.
(B) privately destructive. (C) inherently virtuous.
(C) preferably enterprising. (D) better than Socialism.
(D) mutually exalting. (E) full of blessings for all.
(E) simply irreconcilable.
38. Choose the correct interpretation:
35. According to the passage, Liberalism aims at It was said that

(A) supporting the privileged. (A) the House of Commons has a responsibility
(B) building up monopolies. to fulfill in the interests of the rich.
(C) eliminating poverty. (B) the money taken from the rich should be
(D) destroying individualism. distributed among the poor.
(E) changing history. (C) every penny of the taxes collected by the
state must be spent in the service of the
36. According to the passage, Socialism aims at public.
(D) taxation should not only be better organized
(A) restricting individualism. but also be more eloquently advocated.
(B) preserving private preferences. (E) profit means private fingers in the public
(C) supporting individual interests. pie.
(D) distributing public property.
(E) conserving private interests.

SAT Identification of Sentence Errors


Source: majortests.com

01. We have no choice but to appoint Mary: she is the best of the two candidates, and there is no prospect of finding
more applicants. No error.

02. The reason I will not be going to Mexico this year is because I will use up all my travel money attending an
important meeting in Singapore. No error.

03. If you were to work at least four hours a day on the project, we would complete it in a shorter time, and with
less problems. No error.

04. The manager tried hard to effect a change in company policy, but the owner, who steadfastly refused to
compromise, overruled him on every point. No error.

- 113 - BTA / Ing / S. Intensif 2023


05. The new library is undoubtedly well stocked and functional, but no one can say that its atmosphere is anything
like the old one. No error.

06. My uncle, who was on vacation, along with my two cousins and I, went fishing down by the river. No error.

07. Hopefully, we will be able to complete the building before the rainy season sets in. No error.

08. You would have to choose her, if you are looking for the best athlete to represent the school. No error.

09. All the trapped miners began to lose hope, it had been twenty four hours since the tunnel collapsed. No error.

10. Because they played by the rules, the members of the team were given a standing ovation even though it did not
win the match. No error.

11. Her avaricious relatives assembled at the lawyer's office to hear the reading of Jemima's will. No error.

12. He was not merely expected to contribute funds to the project, but to work as hard as the other patrons. No
error.

13. None of us knows what the outcome of the battle between the coordinator and us will be. No error.

14. Neither of my brothers do anything to make life better for our parents who are both suffering from arthritis. No
error.

15. The teacher sat down besides the frightened child and tried to reassure him that the monster was merely
imaginary. No error.

- 114 - BTA / Ing / S. Intensif 2023


BAHASA INGGRIS
INTERNATIONAL EXAM (3)
Text 1

I chose to wander by Bethlehem Hospital; partly, because it lay on my road round to Westminster; partly, because I
had a fancy in my head which could be best pursued within sight of its walls. And the fancy was: Are not the sane
and the insane equal at night as the sane lie a dreaming? Are not all of us outside this hospital, who dream, more or
less in the condition of those inside it, every night of our lives? Are we not nightly persuaded, as they daily are, that
we associate preposterously with kings and queens, and notabilities of all sorts? Do we not nightly jumble events
and personages and times and places, as these do daily? Said an afflicted man to me, when I visited a hospital like
this, “Sir, I can frequently fly.” I was half ashamed to reflect that so could I - by night. I wonder that the great
master, when he called Sleep the death of each day’s life, did not call Dreams the insanity of each day’s sanity.
Adapted from: The Uncommercial Traveller, C Dickens (1860)

01. It can be correctly inferred that Bethlehem 02. The author makes his point with the aid of all of
hospital the following except

I. is very close to Westminster (A) rhetorical questions


II. has patients who are regarded as insane (B) personal anecdote
III. is a place the author has visited before (C) allusion
(D) frequent use of metaphor
(A) I only (E) repetition and parallel construction
(B) II only
(C) III only
(D) I and II
(E) I, II and III

Text 2

Now, it is clear that the decline of a language must ultimately have political and economic causes: it is not due
simply to the bad influence of this or that individual writer. But an effect can become a cause, reinforcing the
original cause and producing the same effect in an intensified form, and so on indefinitely. A man may take to drink
because he feels himself to be a failure, and then fail all the more completely because he drinks. It is rather the same
thing that is happening to the English language. It becomes ugly and inaccurate because our thoughts are foolish, but
the slovenliness of our language makes it easier for us to have foolish thoughts.
Adapted from: Politics And The English Language, George Orwell

03. The example of the man who takes to drink is 04. The author would most likely agree that
used to illustrate which of the following ideas in
the paragraph? (A) individual writers can never have a bad
influence on the English language
(A) foolish thoughts (B) imprecise use of language is likely to make
(B) the slovenliness of language precise thought more difficult
(C) the English language is ugly and inaccurate
(C) political and economic causes
(D) all language declines for political reasons
(D) an effect becoming a cause
(E) failure generally leads to more failure in a
(E) bad influences downward spiral

- 115 - BTA / Ing / S. Intensif 2023


Text 3

The pioneers of the teaching of science imagined that its introduction into education would remove the
conventionality, artificiality, and backward-lookingness which were characteristic of classical studies, but they were
gravely disappointed. So, too, in their time had the humanists thought that the study of the classical authors in the
original would banish at once the dull pedantry and superstition of mediaeval scholasticism. The professional
schoolmaster was a match for both of them, and has almost managed to make the understanding of chemical
reactions as dull and as dogmatic an affair as the reading of Virgil's Aeneid.
The chief claim for the use of science in education is that it teaches a child something about the actual
universe in which he is living, in making him acquainted with the results of scientific discovery, and at the same
time teaches him how to think logically and inductively by studying scientific method. A certain limited success has
been reached in the first of these aims, but practically none at all in the second. Those privileged members of the
community who have been through a secondary or public school education may be expected to know something
about the elementary physics and chemistry of a hundred years ago, but they probably know hardly more than any
bright boy can pick up from an interest in wireless or scientific hobbies out of school hours.
As to the learning of scientific method, the whole thing is palpably a farce. Actually, for the convenience of
teachers and the requirements of the examination system, it is necessary that the pupils not only do not learn
scientific method but learn precisely the reverse, that is, to believe exactly what they are told and to reproduce it
when asked, whether it seems nonsense to them or not. The way in which educated people respond to such
quackeries as spiritualism or astrology, not to say more dangerous ones such as racial theories or currency myths,
shows that fifty years of education in the method of science in Britain or Germany has produced no visible effect
whatever. The only way of learning the method of science is the long and bitter way of personal experience, and,
until the educational or social systems are altered to make this possible, the best we can expect is the production of a
minority of people who are able to acquire some of the techniques of science and a still smaller minority who are
able to use and develop them.
Adapted from: The Social Function of Science, John D Bernal (1939)

05. The author implies that the 'professional 08. The author blames all of the following for the
schoolmaster' (paragraph 1) has failure to impart scientific method through the
education system except
(A) no interest in teaching science
(B) thwarted attempts to enliven education (A) poor teaching
(C) aided true learning
(B) examination methods
(D) supported the humanists
(E) been a pioneer in both science and (C) lack of direct experience
humanities.
(D) the social and education systems
06. The author’s attitude to secondary and public (E) lack of interest on the part of students
school education in the sciences is

(A) ambivalent 09. If the author were to study current education in


science to see how things have changed since he
(B) neutral wrote the piece, he would probably be most
(C) supportive interested in the answer to which of the
(D) satirical following questions?
(E) contemptuous
(A) Do students know more about the world
about them?
07. The word ‘palpably’ (paragraph 3) most nearly
means (B) Do students spend more time in
laboratories?
(A) empirically (C) Can students apply their knowledge
(B) obviously logically?
(C) tentatively (D) Have textbooks improved?
(D) markedly (E) Do they respect their teachers?
(E) ridiculously

- 116 - BTA / Ing / S. Intensif 2023


10. Astrology (paragraph 3) is mentioned as an (A) at the time of writing, not all children
example of received a secondary school education
(B) the author finds chemical reactions
(A) a science that needs to be better understood interesting
(B) a belief which no educated people hold (C) science teaching has imparted some
(C) something unsupportable to those who have knowledge of facts to some children
absorbed the methods of science (D) the author believes that many teachers are
(D) the gravest danger to society authoritarian
(E) an acknowledged failure of science (E) it is relatively easy to learn scientific
method.
11. All of the following can be inferred from the text
except

Text 4

Paragraph one
All the sound reasons ever given for conserving other natural resources apply to the conservation of wildlife and
with three-fold power. When a spendthrift squanders his capital it is lost to him and his heirs; yet it goes somewhere
else. When a nation allows any one kind of natural resource to be squandered it must suffer a real, positive loss; yet
substitutes of another kind can generally be found. But when wildlife is squandered it does not go elsewhere, like
squandered money; it cannot possibly be replaced by any substitute, as some inorganic resources are: it is simply an
absolute, dead loss, gone beyond even the hope of recall.

Paragraph two
The public still has a hazy idea that Nature has an overflowing sanctuary of her own, somewhere or other, which
will fill up the gaps automatically. The result is that poaching is commonly regarded as a venial offence, poachers
taken red-handed are rarely punished, and willing ears are always lent to the cry that rich sportsmen are trying to
take the bread out of the poor settler's mouth. The poor settler does not reflect that he himself, and all other classes
alike, really have a common interest in the conservation of any wildlife that does not conflict with legitimate human
development.
Both adapted from: Animal Sanctuaries in Labrador, W Wood (1911)

12. The author of paragraph one probably uses the 14. Both paragraphs apparently imply that
expression three-fold power
(A) there is no source from which wildlife, once
(A) because there are three-times as many reasons exterminated, can be replaced
for conserving wildlife (B) poachers must be punished
(B) to be more dramatic that saying double-power (C) wildlife has much in common with other
(C) to emphasize the contrast between loss of natural resources
money, loss of other resources, and loss of (D) conservation is in conflict with human
wildlife development
(D) to stress the need for saving money, resources (E) preserving wildlife is expensive
and time
(E) to indicate the magnitude of the problem
without intending the expression to be taken 15. It can be inferred that the spendthrift in paragraph
literally one and the poor settler mentioned in paragraph
two are alike in that they are
13. From the context, the word venial in paragraph two
most nearly means (A) in conflict with the aims of conservation
(B) inclined to waste natural resources
(A) major (C) more concerned with the present than the
(B) criminal future
(C) frequent (D) unable to control their spending
(D) trivial (E) unaware of conservation
(E) natural

- 117 - BTA / Ing / S. Intensif 2023


Text 5
A stout old lady was walking with her basket down the middle of a street in Petrograd to the great
confusion of the traffic and with no small peril to herself. It was pointed out to her that the pavement was the place
for pedestrians, but she replied: 'I'm going to walk where I like. We've got liberty now.' It did not occur to the dear
old lady that if liberty entitled the pedestrian to walk down the middle of the road, then the end of such liberty would
be universal chaos. Everybody would be getting in everybody else's way and nobody would get anywhere.
Individual liberty would have become social anarchy.
There is a danger of the world getting liberty-drunk in these days like the old lady with the basket, and it is
just as well to remind ourselves of what the rule of the road means. It means that in order that the liberties of all may
be preserved, the liberties of everybody must be curtailed. When the policeman, say, at Piccadilly Circus steps into
the middle of the road and puts out his hand, he is the symbol not of tyranny, but of liberty. You may not think so.
You may, being in a hurry, and seeing your car pulled up by this insolence of office, feel that your liberty has been
outraged. How dare this fellow interfere with your free use of the public highway? Then, if you are a reasonable
person, you will reflect that if he did not interfere with you, he would interfere with no one, and the result would be
that Piccadilly Circus would be a maelstrom that you would never cross at all. You have submitted to a curtailment
of private liberty in order that you may enjoy a social order which makes your liberty a reality.
Liberty is not a personal affair only, but a social contract. It is an accommodation of interests. In matters
which do not touch anybody else's liberty, of course, I may be as free as I like. If I choose to go down the road in a
dressing-gown who shall say me nay? You have liberty to laugh at me, but I have liberty to be indifferent to you.
And if I have a fancy for dyeing my hair, or waxing my moustache (which heaven forbid), or
wearing an overcoat and sandals, or going to bed late or getting up early, I shall follow my fancy and ask no man's
permission. I shall not inquire of you whether I may eat mustard with my mutton. And you will not ask me whether
you may follow this religion or that, whether you may prefer Ella Wheeler Wilcox to Wordsworth, or champagne to
shandy.
In all these and a thousand other details you and I please ourselves and ask no one's leave. We have a whole
kingdom in which we rule alone, can do what we choose, be wise or ridiculous, harsh or easy, conventional or odd.
But directly we step out of that kingdom, our personal liberty of action becomes qualified by other people's liberty. I
might like to practice on the trombone from midnight till three in the morning. If I went on to the top of Everest to
do it, I could please myself, but if I do it in my bedroom my family will object, and if I do it out in the streets the
neighbors will remind me that my liberty to blow the trombone must not interfere with their liberty to sleep in quiet.
There are a lot of people in the world, and I have to accommodate my liberty to their liberties.
We are all liable to forget this, and unfortunately we are much more conscious of the imperfections of
others in this respect than of our own. A reasonable consideration for the rights or feelings of others is the
foundation of social conduct.
It is in the small matters of conduct, in the observance of the rule of the road, that we pass judgment upon
ourselves, and declare that we are civilized or uncivilized. The great moments of heroism and sacrifice are rare. It is
the little habits of commonplace intercourse that make up the great sum of life and sweeten or make bitter the
journey.
Adapted from an essay by George Orwell

16. The author might have stated his rule of the road 18. The sentence It means....curtailed (paragraph 2) is
as an example of
(A) do not walk in the middle of the road (A) hyperbole
(B) follow the orders of policemen (B) cliche
(C) do not behave inconsiderately in public (C) simile
(D) do what you like in private (D) paradox
(E) liberty is more important than anarchy (E) consonance

17. The author’s attitude to the old lady in paragraph 19. Which sentence best sums up the author’s main
one is point?
(A) condescending (A) There is a danger....(paragraph 2)
(B) intolerant (B) A reasonable.... (paragraph 5)
(C) objective (C) It is in the small matters....(paragraph 6)
(D) sardonic (D) The great moments....(paragraph 6)
(E) supportive (E) It is the little.... (paragraph 6)

- 118 - BTA / Ing / S. Intensif 2023


20. A situation analogous to the insolence of office 22. The author assumes that he may be as free as he
described in paragraph 2 would be likes in

(A) a teacher correcting grammar errors (A) all matters of dress and food
(B) an editor shortening the text of an article (B) any situation which does not interfere with the
(C) a tax inspector demanding to see someone’s liberty of others
accounts (C) anything that is not against the law
(D) an army office giving orders to a soldier (D) his own home
(E) a gaoler locking up a prisoner (E) public places as long as no one sees him

21. Qualified (paragraph 4) most nearly means 23. In the sentence We are all liable.... (paragraph 5)
the author is
(A) accredited
(B) improved (A) pointing out a general weakness
(C) limited (B) emphasizing his main point
(D) stymied (C) countering a general misconception
(E) educated
(D) suggesting a remedy
(E) modifying his point of view

Text 6
I have yet to meet a poetry-lover under thirty who was not an introvert, or an introvert who was not
unhappy in adolescence. At school, particularly, maybe, if, as in my own case, it is a boarding school, he sees the
extrovert successful, happy, and good and himself unpopular or neglected; and what is hardest to bear is not
unpopularity, but the consciousness that it is deserved, that he is grubby and inferior and frightened and dull.
Knowing no other kind of society than the contingent, he imagines that this arrangement is part of the eternal
scheme of things, that he is doomed to a life of failure and envy. It is not till he grows up, till years later he runs
across the heroes of his school days and finds them grown commonplace and sterile, that he realizes that the
introvert is the lucky one, the best adapted to an industrial civilization the collective values of which are so infantile
that he alone can grow, who has educated his fantasies and learned how to draw upon the resources of his inner life.
At the time, however, his adolescence is unpleasant enough. Unable to imagine a society in which he would feel at
home, he turns away from the human to the nonhuman: homesick he will seek, not his mother, but mountains or
autumn woods, and the growing life within him will express itself in a devotion to music and thoughts upon
mutability and death. Art for him will be something infinitely precious, pessimistic, and hostile to life. If it speaks of
love it must be love frustrated, for all success seems to him noisy and vulgar; if it moralizes, it must counsel a stoic
resignation, for the world he knows is well content with itself and will not change.
Deep as first love and wild with all regret,
O death in life, the days that are no more.
Now more than ever seems it sweet to die
To cease upon the midnight with no pain.
That to the adolescent is the authentic poetic note and whoever is the first in his life to strike it, whether
Tennyson, Keats, Swinburne, Housman or another, awakens a passion of imitation and an affectation which no
subsequent refinement or sophistication of his taste can entirely destroy. In my own case it was Hardy in the summer
of 1923; for more than a year I read no one else and I do not think that I was ever without one volume or another or
the beautifully produced Wessex edition in my hands: I smuggled them into class, carried them about on Sunday
walks, and took them up to the dormitory to read in the early morning, though they were far too unwieldy to be read
in bed with comfort. In the autumn of 1924 there was a palace revolution after which he had to share his kingdom
with Edward Thomas, until finally they were both defeated by Elliot at the battle of Oxford in 1926. Besides serving
as the archetype of the Poetic, Hardy was also an expression of the contemporary scene. He was both my Keats and
my Sandburg. To begin with, he looked like my father: that broad unpampered moustache, bald forehead, and
deeply lined sympathetic face belonged to that other world of feeling and sensation. Here was a writer whose
emotions, if sometimes monotonous and sentimental in expression, would be deeper and more faithful than my own,
and whose attachment to the earth would be more secure and observant.
Adapted from an article written by W H Auden

- 119 - BTA / Ing / S. Intensif 2023


24. According to the author, poetry lovers under thirty 29. The author uses all of the following to make his
generally point except

(A) have a strong sense of their own inferiority (A) metaphor


during school years (B) personal experience
(B) are always products of boarding schools (C) generalization
(C) have an unhappy home life (D) classical allusions
(D) are outgoing as adolescents (E) comparison
(E) long to return to early childhood
30. The poetry quoted (italicized) is most likely
25. The author’s main purpose is apparently to included as

(A) describe what lead to his being an introvert (A) extracts from the author’s own poetry
(B) explore the reasons for his early taste in poetry (B) extracts from Hardy’s poetry
(C) explain what lead to his becoming a poet (C) examples of poetry that appeals to the
(D) account for the unhappy adolescent’s aesthetic unhappy adolescent
sense (D) the type of poetry much admired by all poetry
(E) criticize a system that makes young people lovers
feel unhappy and neglected (E) examples of schoolboy poetry

26. The author regards the introverted adolescent as 31. It can be inferred that Edward Thomas
ultimately lucky because he has
(A) was once held in high esteem by the author
(A) become financially successful in an (B) was a better poet than Hardy
industrialized society (C) was writing in 1924
(B) ceased to envy others (D) had views opposed to Eliot
(C) cultivated inner resources that he will need in (E) wrote poetry similar to that of Hardy
modern society
(D) a better general education than those who were 32. The author mentions Sandburg (last paragraph) as
envied in school
(E) learned to appreciate nature (A) an example of a modern poet
(B) an example of a traditional figure
27. To the adolescent the authentic poetic note is one (C) having a poetic appearance
of (D) a poet to appeal to young people
(E) resembling his father
(A) pain and affirmation
(B) hostility and vulgarity 33. The author qualifies his appreciation of Hardy by
(C) contentment and peace pointing out that Hardy’s poetic techniques were
(D) purity and love
(E) melancholy and acceptance (A) sometimes unmoving
(B) not always deeply felt
28. It can be inferred that, for the author, the poetry of (C) occasionally lacking in variety
Hardy is (D) always emotional
(E) irrelevant to certain readers
(A) something with which he is not entirely
comfortable 34. The author feels that Hardy’s physical appearance
suggested
(B) a temporary interest soon supplanted by other
poetry
(A) deep and lasting feelings
(C) a secret obsession that he is reluctant to
(B) paternal values
confess
(C) careworn old age
(D) his first poetic love that time has not entirely
erased (D) a contemporary writer
(E) a childlike passion (E) fatherly concern

- 120 - BTA / Ing / S. Intensif 2023


Text 7

In regard to propaganda the early advocates of universal literacy and a free press envisaged only two
possibilities: the propaganda might be true, or it might be false. They did not foresee what in fact has happened,
above all in our Western capitalist democracies - the development of a vast mass communications industry,
concerned in the main neither with the true nor the false, but with the unreal, the more or less totally irrelevant. In a
word, they failed to take into account man's almost infinite appetite for distractions.
In the past most people never got a chance of fully satisfying this appetite. They might long for distractions,
but the distractions were not provided. Christmas came but once a year, feasts were "solemn and rare," there were
few readers and very little to read, and the nearest approach to a neighborhood movie theater was the parish
church, where the performances, though infrequent, were somewhat monotonous. For conditions even remotely
comparable to those now prevailing we must return to imperial Rome, where the populace was kept in good humor
by frequent, gratuitous doses of many kinds of entertainment - from poetical dramas to gladiatorial fights, from
recitations of Virgil to all-out boxing, from concerts to military reviews and public executions. But even in Rome
there was nothing like the non-stop distraction now provided by newspapers and magazines, by radio, television and
the cinema. In Brave New World non-stop distractions of the most fascinating nature (the feelies, orgy-porgy,
centrifugal bumblepuppy) are deliberately used as instruments of policy, for the purpose of preventing people from
paying too much attention to the realities of the social and political situation. The other world of religion is different
from the other world of entertainment; but they resemble one another in being most decidedly "not of this world."
Both are distractions and, if lived in too continuously, both can become, in Marx's phrase, "the opium of the people"
and so a threat to freedom. Only the vigilant can maintain their liberties, and only those who are constantly and
intelligently on the spot can hope to govern themselves effectively by democratic procedures. A society, most of
whose members spend a great part of their time, not on the spot, not here and now and in the calculable future, but
somewhere else, in the irrelevant other worlds of sport and soap opera, of mythology and metaphysical fantasy,
will find it hard to resist the encroachments of those who would manipulate and control it.
In their propaganda today's dictators rely for the most part on repetition, suppression and rationalization –
the repetition of catchwords which they wish to be accepted as true, the suppression of facts which they wish to be
ignored, the arousal and rationalization of passions which may be used in the interests of the Party or the State. As
the art and science of manipulation come to be better understood, the dictators of the future will doubtless learn to
combine these techniques with the non-stop distractions which, in the West, are now threatening to drown in a sea of
irrelevance the rational propaganda essential to the maintenance of individual liberty and the survival of democratic
institutions.
Adapted from the introduction to Brave New World, A Huxley (1931)

35. The author would be most likely to agree that 37. The author refers to Brave New World (paragraph
propaganda 2) as a fictional example of a society in which

(A) can serve a vital function in democracy (A) non-stop distractions are the main instrument
(B) is concerned mainly with the irrelevant of government policy
(C) is now combined with entertainment (B) people are totally unaware of political realities
(C) entertainment is used to keep people from full
(D) is universally recognized as a danger
awareness of social realities
(E) needs constant vigilance to avoid (D) entertainment resembles religion in its effects
on the masses
36. The early advocates of universal literacy (line 1) (E) non-stop entertainment is provided as it was in
are mentioned as Rome

(A) advocates of propaganda 38. By intelligently on the spot (paragraph 2) the


(B) opponents of an idea that the author thinks is author apparently means
correct
(C) proponents of an idea that the author wishes to (A) alert to the dangers of propaganda
counter (B) in a particular society at a particular time
(D) people who made wrong predictions about
(C) in a specific time and place
freedom of the press
(E) social commentators unaware of man’s (D) conscious of political and social realities
appetite for distractions (E) deeply aware of current trends

- 121 - BTA / Ing / S. Intensif 2023


SAT Identification of Sentence Errors
Source: majortests.com

01. A number of trainees who take this course every year find that their knowledge of mathematics is inadequate.
No error.

02. Either of the solutions you have proposed are acceptable to the union, whose members are willing to
compromise. No error.

03. The last man on earth will abandon his ruined house for a cave, and his woven clothes for an animal's skin. No
error.

04. The station was a hive of bustling activity, the arrival of the train was the most important event of the day in
that remote place. No error.

05. My grandmother's legacy is substantial, especially if the value of the rare stamps are taken into consideration.
No error.

06. Neither Bradley, nor more recent critics who have written on Shakespeare's tragedies, has been able to give a
convincing explanation for the timing of events in Othello. No error.

07. The bridal gown was most unique: the bridegroom designed it and his mother provided the lace fabric. No error.

08. For a successful career as a beautician, one must be prepared to dissemble: you must not tell your client the
unvarnished truth about his or her appearance. No error.

09. When Russell Wallace and Darwin independently proposed similar theories, Darwin had already accumulated
extensive evidence with which to support his ideas. No error.

10. Everyone who visits Singapore is impressed by its cleanliness, which is mainly a result of rigorous
implementation of their strict laws. No error.

11. She wondered whether the city has changed a lot since she had left to go to university. No error.

12. The company bowed to pressure, now it has removed the offensive advertisement from the hoarding. No error.

13. I will not object to his delivering the lecture as long as he is told not to make personal attacks on his critics. No
error.

14. While he thinks the phenomenon is the result of enzyme action, I believe it is caused by a shortage of a
neurotransmitter. No error.

15. Cynthia argued vehemently with her mother over her refusal to attend the school concert. No error.

- 122 - BTA / Ing / S. Intensif 2023

You might also like